SlideShare a Scribd company logo
1 of 253
Question 1: Robert Mugabe has ruled Zimbabwe since the _______
achieved independence in 1980.
A. national B. nation C. nationality D. nationally
Kiến thức: Từ loại
Giải thích:
Xét cá đáp án:
A. national (a): quốc gia B. Nation (n): đất nước
C. nationality (n): quốc tịnh D. Nationally (adv):
quốc gia
- Sau mạo từ the + danh từ, có 2 đáp án B, C là danh từ
Đáp án đúng là B
Tạm dịch: Robert Mugabe đã thống trị Zimbabwe kể từ khi đất nước
giành được độc lập vào năm1980.
Question 2: It is time to begin, _______?
A. isn’t it B. are they C. do I D. is it
Kiến thức: Câu hỏi đuôi
Giải thích:
- câu hỏi đuôi, It is là khẳng định => đuôi là phủ định
Đáp án đúng là A
Tạm dịch: Đã đến lúc bắt đầu rồi, đúng không?
Question 3: At first, I give a(n) _______ impression that you are
too anxious for a settlement.
A. wrong B. false C. improper D. incorrect
Kiến thức: Thành ngữ
Giải thích:
Ta có thành ngữ: give a false impression (collocation): nhầm
tưởng
Đáp án đúng là B
Tạm dịch: Lúc đầu, tôi nhầm tưởng rằng bạn lo lắng cho bản thỏa
thuận.
Question 4: Grinder, a 35-year military veteran, was named to the
post _______ Thursday, Jan. 13
A. on B. in C. of D. with
Kiến thức: Giới từ
Giải thích:
Ta có, trước ngày tháng năm + on
Đáp án đúng là A
Tạm dịch: Grinder, một quân nhân 35 năm kinh nghiệm, được cử
đến doanh trại quân đội vào thứ năm ngày 13/1.
Question 5: Poteet will receive a $100 cash _______ and $200 for
his school to purchase books.
A. bonus B. award C. gift D. prize
Kiến thức: Từ vựng
Giải thích:
Xét các đáp án:
- prize : giải thưởng, phần thưởng (ít trang trọng, dùng cho người có
hành động tốt)
- award: giải thưởng, phần thưởng (dùng cho người nào đó làm
hành động lớn lao )
- gift: món quà
- bonus: phần thưởng thêm
Đáp án đúng là D
Tạm dịch: Poteet sẽ nhận được một phần thưởng tiền mặt 200 đô
và 200 đô cho trường anh ấy để mua sách.
Question 6: Kids without guidance got into trouble _______ there
was nothing else to do.
A. because B. though C. because of D. despite
Kiến thức: Liên từ
Giải thích:
Xét các đáp án:
- because + clause: bởi vì - because of + N: bởi vì
- though + clause: mặc dù - despite + N: mặc dù
Đáp án đúng là A
Tạm dịch: Những đứa trẻ không có sự hướng dẫn sẽ gặp phiền toái
bởi vì không có cái gì khác để làm.
Question 7: Tonight, Hana wore a _______ dress at the party.
A. gorgeous red Chinese B. gorgeous Chinese red
C. red Chinese gorgeous D. Chinese red gorgeous
Kiến thức: Trật tự tính từ
Giải thích:
- Theo quy tắc trật tự tính từ trong câu: OSASCOMP
- gorgeous : tính từ chỉ quan điểm
- red: màu sắc
- Chinese: nguồn gốc
Đáp án đúng là A
Tạm dịch: Tối nay, Hana mặc 1 chiếc váy lộng lẫy, màu đỏ , bắt nguồn
từ Trung Hoa đến buổi tiệc.
Question 8: Don't _______ when he is telling the story.
A. stop in B. stop off C. break in D. break into
Kiến thức: Cụm động từ
Giải thích:
Xét các đáp án:
- stop in: ở nhà - stop off: ngừng chuyến đi/hành trình
- break in: chen lời, ngắt lời - break into: đột nhập
Tạm dịch: Đừng có ngắt lời khi anh ấy đang kể 1 câu chuyện.
Question 9: My father _______ TV when my friend suddenly
came to my house.
A. has watched B. watches C. watched
D. was watching
Kiến thức: Thì quá khứ tiếp diễn
Giải thích:
Ta thấy mệnh đề trạng ngữ chỉ thời gian bắt đầu bằng WHEN và
động từ CAME ở quá khứ đơn nên động từ ở mệnh đề chính ở thì
quá khứ tiếp diễn (một hành động đang diễn ra thì bị một hành
động khác xen vào).
Công thức: S1 + was/ were + V1-ing + when + S2 + V2-simple past
Đáp án đúng là D
Tạm dịch: Khi ba tôi đang xem TV, bạn tôi bất thình lình ghé nhà
tôi.
Question 10: _______, I will give him your regards.
A. When I see him B. When I saw him
C. When I was seeing him D. When I had seen him
Kiến thức: Sự phối hợp các thì (Tương lai & hiện tại)
Giải thích:
- Sau mệnh đề chỉ thời gian chia hiện tại, vế còn lại chia tương lai.
(mang nghĩa tương lai)
Đáp án đúng là A
Tạm dịch: Khi tôi gặp anh ấy, tôi sẽ chuyển lời hỏi thăm của bạn.
Question 11: Could you _______ me a favour and pick up Sam from
school today?
A. make B. do C. put D. go
Kiến thức: Cụm từ cố định
Giải thích:
- Ta có cụm từ cố định: do sb a favour: giúp ai đó
Đáp án đúng là B
Tạm dịch: Bạn có thể giúp tôi đón Sam đi học về hôm nay không?
Question 12: The doctor said that she might never be able to
_______ children.
A. care B. tolerate C. bear D. feed
Kiến thức: Từ vựng
Giải thích:
Xét các đáp án:
- bear (v): chịu đựng/sinh con- tolerate (v): chịu đựng
- care (v): quan tâm - feed (v): cho ai ăn
Đáp án đúng là C
Tạm dịch: Bác sĩ nói rằng cô ấy có khả năng không thể sinh con.
Question 13: This house _______ in the 18th century.
A. is building B. were build C. was built D. is built
Kiến thức: Câu bị động
Giải thích:
Ta thấy, trạng ngữ chỉ thời gian “in the 18th century” – nên động từ
chia ở quá khứ và chủ ngữ là “this house” nên động từ ở thể bị động.
- Bị động quá khứ đơn: was/ were+ V3/ V-ed
Đáp án đúng là C
Tạm dịch: Ngôi nhà này được xây dựng thế kỉ 18.
Question 14: _______ her homework, she watched her favourite
movie.
A. Having finished B. finish
C. being finished D. she finishes
Kiến thức : Rút gọn mệnh đề trạng ngữ
Giải thích:
- Khi hai mệnh đề có cùng chủ ngữ có thể rút gọn về: having +V3/ V-ed
(quá khứ phân từ)
Đáp án đúng là A
Tạm dịch: Hoàn thành xong bài tập về nhà, cô ấy xem bộ phim yêu
thích nhất
Question 15: The sooner JK take your medicine, _______ he will feel.
A. better B. the better C. the good D. well
Kiến thức: So sánh kép
Giải thích:
- So sánh đồng tiến: The + so sánh hơn S+V, The + so sánh hơn S+V:
càng… càng
Đáp án đúng là B
Tạm dịch: JK càng uống thuốc sớm, anh ta càng cảm thấy khỏe
Question 16: Ann and Peter are doing homework together.
- Ann: “Could you lend me that book”
- Peter: “_______”
A. No, I’m busy. B. That’s what I think.
C. Why did you say that? D. Of course, here it is.
Kiến thức: Tình huống giao tiếp (Đáp lại lời đề nghị)
Giải thích:
Tình huống giao tiếp:
- Ann: Bạn có thể cho tôi mượn quyển sách đó?
- “_______”
Xét các đáp án:
- A. Không, tôi bận - B. Đó là những gì tôi nghĩ
- C. Tại sao bạn nói như vậy? - Dĩ nhiên rồi. Đây nè.
Question 17: Linda is thanking Daniel for his birthday present.
- Linda: “Thanks for the book. I’ve been looking for it for
months.”
- Daniel: “_______”
A. You can say that again. B. Thank you for looking at it.
C. I like reading books. D. I’m glad you like it.
Kiến thức: Tình huống giao tiếp
Giải thích:
Tình huống giao tiếp: Linda đang cảm ơn Daniel về món quà sinh
nhật.
- Linda: “Thanks for the book. I’ve been looking for it for months.”
- Daniel: “_______”
Xét các đáp án:
A. Tôi hoàn toàn đồng ý. B. Cảm ơn vì đã nhìn nó.
C. Tôi thích đọc sách. D. Tôi nghĩ bạn thích nó.
Question 18: A. permission B. suggestion C. refusal
D. possible
Kiến thức: Trọng âm của từ 3 âm tiết
Giải thích:
A. permission /pəˈmɪʃn/: Trọng âm rơi vào âm tiết thứ 2 vì theo quy tắc
đuôi _sion trọng âm rơi vào âm tiết trước nó.
B. suggestion /səˈdʒestʃən/: Trọng âm rơi vào âm tiết thứ 2 vì theo quy
tắc đuôi _tion trọng âm rơi vào âm tiết trước nó.
C. refusal /rɪˈ uːzl/: Trọng âm rơi vào âm tiết thứ 2 vì theo quy tắc hậu
tố không thay đổi trọng âm chính
D. possible /ˈpɒsəbl/: Trọng âm rơi vào âm tiết thứ nhất vì theo quy tắc
từ 3 âm tiết mà cả 3 âm tiết đều chứa nguyên âm ngắn thì trọng âm rơi
vào âm tiết thứ nhất.
Question 19: A. forest B. succeed C. homeless D. nation
Kiến thức: Trọng âm của từ 2 âm tiết
Giải thích:
A. forest /ˈfɒrɪst/ (n): Trọng âm rơi vào âm tiết thứ nhất vì theo quy tắc
danh từ, tính từ 2 âm tiết trọng âm thường rơi vào âm tiết thứ nhất.
B. succeed /səkˈsiːd/ (v): Trọng âm rơi vào âm tiết thứ hai vì theo quy
tắc động từ 2 âm tiết trọng âm thường rơi vào âm tiết thứ hai.
C. homeless /ˈhəʊmləs/ (adj): Trọng âm rơi vào âm tiết thứ nhất vì
theo quy tắc danh từ, tính từ 2 âm tiết trọng âm thường rơi vào âm
tiết thứ nhất.
D. nation /ˈneɪʃn/ (n): Trọng âm rơi vào âm tiết thứ nhất vì theo quy
tắc danh từ, tính từ 2 âm tiết trọng âm thường rơi vào âm tiết thứ
nhất.
Question 20: A. stopped B. wanted C. decided D. visited
Kiến thức : Cách phát âm đuôi -ED
Giải thích:
– Phát âm là /t/ khi từ có tận cùng bằng các phụ âm vô thanh: /θ/,
/p/, /k/, /f/, /s/, /ʃ/, /tʃ/
– Phát âm là /id/ khi từ có tận cùng là các âm: /t/, /d/
– Phát âm là /d/ khi các từ có tận cùng là nguyên âm và các phụ âm
hữu thanh còn lại
Xét các đáp án:
A. stopped /t/ B. wanted /id/
C. decided /id/ D. visited /id/
Question 21: A. head B. please C. heavy D.
measure
Kiến thức: Cách phát âm của nguyên âm
Giải thích:
Xét các đáp án:
A. head /hed/ B. please /pliːz/
C. heavy /ˈhevi/ D. measure /ˈmeʒə(r)/
Ta thấy các phương án A, C, D – phần gạch chân được phát âm là
/e/ còn phương án B được phát âm là /i:/
Question 22: They tell us she isn't allowed to leave unless she gets
the doctor's permission.
A. disapproval B. objection
C. agreement D. refusal
Kiến thức: Đồng nghĩa (từ đơn)
Giải thích:
Ta có: permission (n) sự cho phép
Xét các phương án:
A. disapproval (n) sự không tán thành B. objection (n)
sự phản đối
C. agreement (n) sự đồng ý D. refusal (n) từ chối
- permission (n) sự cho phép = agreement (n) sự đồng ý
Vậy đáp án đúng là C
Tạm dịch: Họ bảo với chúng tôi rằng cô ấy không được phép rời khỏi
nếu không có sự đồng ý của bác sĩ.
Question 23: Before the 1950s, urbanization mainly occurred in more
economically developed countries (MEDCs).
A. happened B. take place C. concentrated D. expanded
Kiến thức: Đồng nghĩa (từ đơn)
Giải thích:
Ta có: occur (v): xảy ra, diễn ra
Xét các đáp án:
A. happen (v): diễn ra, xảy ra
B. take place (phr.v): diễn ra, xảy ra => Ngữ cảnh đang chia ở quá khứ
nên đáp án này không phù hợp về thì.
C. concentrate /ˈkɑːnsntreɪt/ (v): tập trung
D. expand /ɪkˈspænd/ (v): mở rộng
=>Do đó: occurred ~ happened
Question 24: I would love to go back to college, but unfortunately,
that ship has sailed.
A. it was late B. it was timely
C. it was inconvenient D. it was traditional
Kiến thức: Trái nghĩa (cụm từ hoặc thành ngữ)
Giải thích:
Ta có: That ship has sailed (idiom): đã quá muộn
Xét các đáp án:
A. It was late: đã muộn, trễ B. It was timely: kịp lúc
C. It was inconvenient: bất tiện D. It was traditional:
theo truyền thống
That ship has sailed (idiom): đã quá muộn >< It was timely: kịp lúc
Vậy đáp án đúng là B
Tạm dịch: Tôi muốn quay lại trường nhưng thật không may, đã quá
muộn.
Question 25: A comfortable working environment will increase
productivity.
A. promote B. raise C. decrease D. go up
Kiến thức: Trái nghĩa (từ đơn)
Giải thích:
Ta có: increase (v): tăng
- promote (v) thúc đẩy, thăng chức
- raise (v) = go up = increase: tăng>< decrease (v) giảm
Vậy đáp án đúng là C
Tạm dịch: Một môi trường làm việc thoải mái sẽ tang năng suất.
Question 26: It was not until they had reached Dak Lak that they realized how far
they had gone.
A. Not until they had reached Dak Lak did they realize how far they had gone.
B. Not until had they reached Dak Lak, they realized how far they had gone.
C. Not until they reached Dak Lak had they realized how far they had gone.
D. Not until they realized how far they had gone had they reached Dak Lak.
Kiến thức: Kết hợp câu – đảo ngữ
Giải thích:
Cấu trúc đảo ngữ: Not until + clause, aux +S+V: chỉ tới khi mà…….thì…..
It was not until they had reached Dak Lak that they realized how far they had gone.
= A. Not until they had reached Dak Lak did they realize how far they had gone.
Tạm dịch: Chỉ tới khi họ đến Đăk Lăk, họ mới nhận ra họ đã đi được bao xa.
Question 27: They have a lot of homework today.
A. They wish they had not a lot of homework today.
B. They wish they didn’t have a lot of homework today.
C. They wish they had had little homework today.
D. They wish they wouldn’t have a lot of homework today.
Kiến thức: Kết hợp câu – câu ước
Giải thích:
- câu ước muốn loại 2: S+wish+ S + V2/V-ed
Câu đề bài: They have a lot of homework today.
= B. They wish they didn’t have a lot of homework today.
Tạm dịch: Họ có rất nhiều bài tập hôm nay
=>Họ ước họ không có nhiều bài tập hôm nay
Question 28: Let's start by reviewing what we do with those
friends last week.
A. start B. reviewing C. do D. friends
Kiến thức: Lỗi sai – Thì của động từ
Giải thích:
- Sai thì của động từ vì trong câu có last week nên chia động từ ở thì
quá khứ.
Vậy đáp án đúng là C
Sửa lỗi: do → did
Tạm dịch: Hãy bắt đầu xem xét lại những gì chúng ta làm với những
người bạn vào tuần trước.
Question 29: Lily and her mother went to hospital last week to check
our health.
A. her B. went C. last D. our health
Kiến thức: Lỗi sai – Đại từ nhân xưng/ tính từ sở hữu
Giải thích:
- Sai tính từ sở hữu vì chủ ngữ Lily and her mother nên tính từ sở
hữu thay thế là THEIR
Vậy đáp án đúng là D
Sửa lỗi: our → their
Tạm dịch: Lily và mẹ cô ấy đi bệnh viện tuần trước để kiểm tra sức
khỏe.
Question 30: I must complement you on your handling of a very
difficult situation.
A. must B. complement C. handling
D. very
Kiến thức: Lỗi sai – Từ vựng
Giải thích:
- complement: thêm vào
- compliment sb on: khen ai đó
Dựa vào nghĩa, đáp án đúng là B
Sửa lỗi: complement → compliment
Tạm dịch: Tôi phải khen bạn vì đã giải quyết 1 tình huống cực kì khó
khăn
Question 31: She advised him to go to the dentist immediately.
A. He would go to the dentist immediately.
B. She should go to the dentist immediately.
C. He should go to the dentist immediately.
D. He must go to the dentist immediately.
Kiến thức: Câu đồng nghĩa – Động từ khuyết thiếu
Giải thích:
Câu đề bài: She advised him to go to the dentist immediately.
(Cô ấy khen anh ta đi nha sĩ ngay lập tức.) (khuyên bảo)
= C. He should go to the dentist immediately.
(Anh ấy nên đi nha sĩ ngay lập tức.)
Question 32: She said, “Don’t laugh, Jessica. Be serious.”
A. She said not to laugh and be serious.
B. She said Jessica not to laugh and be serious.
C. She told Jessica not to laugh and asked him to be serious.
D. She told Jessica do not laugh and be serious.
Kiến thức: Câu đồng nghĩa – Câu tường thuật
Giải thích:
Câu đề bài: She said, “Don’t laugh, Jessica. Be serious.”
(Cô ấy nói : “ Đừng cười, Jessica. Hãy nghiêm túc.”)
- A,B, D. Sai cấu trúc ngữ pháp
- tell sb (not) to do sth: bảo ai làm gì và ask sb to do sth: yêu cầu ai làm gì.
Vậy đáp án đúng là C
C. She told Jessica not to laugh and asked him to be serious.
(Cô ấy bảo Jessica không cười và yêu cầu anh ấy nghiêm túc.)
Question 33: The last time I played the piano was 10 years ago.
A. I have not played the piano for 10 years ago.
B. I have not played the piano for 10 years.
C. I have not played the piano since 10 years.
D. I did not play the piano since 10 years.
Kiến thức: Câu đồng nghĩa – Thì của động từ
Giải thích:
Ta có công thức:
S + haven’t/ hasn’t + V- pp + O + for + time
= S + (last) + V-simple past + O + time + ago
= It’s + time + since + S + (last) + V-simple past + O
= The last time + S + V-simple past + O + was + time + ago
Vậy đáp án đúng là B
The last time I played the piano was 10 years ago.
= B. I have not played the piano for 10 years.
Tạm dịch: Lần cuối cùng tôi chơi piano là cách đây 10 năm.
Question 34: A. many B. each C. much D. little
Plants and animals will find it difficult to escape from or adjust to
the effect of global warming. Scientists have already observed
shifts in the life cycles of (34) _______ plants and animals,
Kiến thức: Đọc điền từ
Giải thích:
Xét các đáp án:
A. Many + danh từ số nhiều đếm được: nhiều B. Each
+ danh từ số ít: mỗi, một
C. Much + danh từ không đếm được D. Little + danh từ
không đếm được
Question 35: A. increasing B. blooming
C. growing D. swelling
such as flowers (35) _______ earlier and birds hatching earlier in the
spring.
Kiến thức: Đọc điền từ
Giải thích:
Xét các đáp án:
A. increasing : tăng B. blooming: nở hoa
C. growing: phát triển D. swelling: sưng lên
Dựa vào nghĩa, đáp án đúng là B
Tạm dịch: Các nhà khoa học đã quan sát thấy sự thay đổi trong chu kỳ
sống của nhiều loài thực vật và động vật, chẳng hạn như hoa nở sớm
hơn và chim nở sớm hơn vào mùa xuân.
Question 36: A. that B. what C. which D. where
Many species have begun shifting (36) _______ they live or their
annual migration patterns due to warmer temperatures.
Kiến thức: Đọc điền từ
Giải thích:
Xét các đáp án:
A. that: thay thế cho danh từ chỉ người và vật có chức năng làm chủ
ngữ hoặc tân ngữ trong mệnh đề quan hệ xác định.
B. what: cái gì (không phải đại từ quan hệ)
C. which: thay thế cho danh từ chỉ vật có chức năng làm chủ ngữ hoặc
tân ngữ trong mệnh đề quan hệ.
D. where: thay thế cho danh từ chỉ nơi chốn và có chức năng làm
trạng từ chỉ nơi chốn trong mệnh đề quan hệ.
Ta thấy danh từ mà đại từ quan hệ thay thế “shifting” và sau đại từ
quan hệ là “they live” nên đáp án đúng là D (where)
Question 37:
A. because B. but C. and D. although
Species living in unique ecosystems, such as those found in polar
and mountaintop regions, are especially at risk (37) _______
migration to new habitats is not possible.
Kiến thức: Đọc điền từ
Giải thích:
Xét các đáp án:
A. because (+ clause): bởi vì B. but : nhưng
C. and : và D. although: mặc dù
Tạm dịch: Các loài sống trong các hệ sinh thái độc đáo, chẳng hạn
như các loài được tìm thấy ở các vùng cực và đỉnh núi, đặc biệt gặp
rủi ro vì không thể di cư đến các môi trường sống mới.
Question 38:
A. decreasing B. falling C. deepening D. dwindling
For example, polar bears and marine mammals in the Arctic are
already threatened by (38) _______ sea ice but have nowhere
farther to go.
Kiến thức: Đọc điền từ
Giải thích:
Xét các đáp án:
A. decreasing: giảm B. falling: rơi
C. deepening: trở nên sâu hơn D. dwindling:
tan, thu nhỏ dần
Dựa vào nghĩa, đáp án đúng là D
Tạm dịch: Ví dụ, gấu Bắc Cực và động vật biển có vú ở Bắc Cực đã bị
đe dọa bởi băng biển đang tan dần nhưng không còn nơi nào xa hơn
để đi.
Question 39: What is the passage mainly about?
A. A New Year’s Eve to remember.
B. Preparations before the event.
C. Activities during the ceremony.
D. A great party.
Kiến thức: Đọc hiểu
Giải thích: Đoạn văn này chủ yếu nói về cái gì?
A. Một đêm giao thừa đáng nhớ (tác giả kể lại trải nghiệm đón năm
mới)
B. Sự chuẩn bị trước sự kiện (đây chỉ là 1 ý trong đoạn vì có đề cập
đến trong 5 ngày ở đầu đoạn 1)
C. Sự chuẩn bị trong quá trình lễ (có trước và trong lễ): thông tin
chưa đủ
D. Một bữa tiệc tuyệt vời (trong bài đề cập đến năm mới không phải
là tiệc)
Question 40: Which of the following is NOT mentioned in paragraph 4 as
preparations before the event?
A. People decorated their houses. B. People bought new clothes and jewelry.
C. Women made tasty candies. D. Women went to pagodas to pray for
health.
Kiến thức: Đọc hiểu
Giải thích:
Điều nào sau đây KHÔNG được đề cập trong đoạn 4 như là sự chuẩn bị trước sự
kiện?
Xét các đáp án:
A. Mọi người trang trí nhà cửa. B. Mọi người mua quần áo và đồ
trang sức mới.
C. Phụ nữ làm ra những viên kẹo ngon. D. Phụ nữ đi lễ chùa để cầu
sức khỏe.
Thông tin: People cleaned their homes and painted wonderful designs called
rangolis on the walls and floors. They bought beautiful new clothes and jewelry
to wear during the festival, and women made delicious sweets …:
Tạm dịch: Mọi người lau dọn nhà cửa và sơn những mẫu thiết kế tuyệt vời được
gọi là rangolis trên tường và cửa chính. Họ mua quần áo mới và trang sức để mặc
trong lễ hội và phụ nữ làm những món kẹo rất ngon…
=> đáp án không có thông tin trong bài là D
Question 41: According to paragraph 2, which of the following can be true about the "Festival
of Lights”?
A. It takes place annually.
B. Its aim is to celebrate the victory of evil over good.
C. It lasts for 2 weeks.
D. It takes place in the second week of November.
Question 41: Đáp án A
Kiến thức: Đọc hiểu
Giải thích:
Theo đoạn 2, câu nào sau đây là đúng về “Lễ hội ánh sáng”
A. Nó diễn ra hàng năm
B. Mục đích của nó là tổ chức sự chiến thắng cái ác đối với cái thiện (sai)
C. Nó kéo dài 2 tuần
D. Nó diễn ra vào tuần thứ 2 của tháng 11
Thông tin 1: the "Festival of Lights" which takes place every year to celebrate the victory of
good over evil.
Tạm dịch: Lễ hội ánh sáng diễn ra hàng năm để tổ chức chiến thắng cái thiện đối với cái ác.
Thông tin 2: the "Festival of Lights" which takes place every year to celebrate the victory of
good over evil.
Tạm dịch: Lễ hội ánh sáng diễn ra hàng năm để tổ chức chiến thắng cái thiện đối với cái ác.
Thông tin 3: This festival lasts for five days
Tạm dịch: Lễ hội kéo dài 5 ngày
Thông tin 4: at the end of October or in the first week of November:
Tạm dịch: cuối tháng 10 hoặc tuần đầu tháng 11
Question 42: The word glittered in paragraph 1 is closest in meaning
to _________.
A. sparkled B. lighted C. pressed D. rubbed
Kiến thức: Đọc hiểu
Giải thích: Từ glittered trong đoạn 1 gần nghĩa nhất với _________.
Xét các đáp án:
A. sparkled: lấp lánh B. lighted: sáng
C. pressed : ép D. rubbed: cọ xát
- glitter : long lanh = sparkle: lấp lánh
Vậy đáp án đúng là A
Thông tin: After dark, the whole neighbourhood glittered with
thousands of tiny lights, as though in a fairy tale.
Tạm dịch: Sau khi trời tối, cả khu phố lấp lánh với hàng nghìn ngọn
đèn nhỏ như trong truyện cổ tích.
Question 43: The word others in paragraph 4 refers to _________.
A. people B. women C. sweets D. clothes
Kiến thức: Đọc hiểu
Giải thích: Từ others trong đoạn 4 đề cập đến _________.
A. người B. phụ nữ
C. đồ ngọt D. quần áo
Thông tin: They bought beautiful new clothes and jewelry to wear
during the festival, and women made delicious sweets which were
better than any others I have ever tasted.
Tạm dịch: Họ mua những bộ quần áo mới và đồ trang sức đẹp để
mặc trong lễ hội, và những người phụ nữ làm những món đồ ngọt
ngon tuyệt hơn bất kỳ loại đồ ngọt nào mà tôi từng nếm.
Question 44: Which of the following is the best title for the passage?
A. The disappearance of food species.
B. Food for the Future.
C. Ways to increase the number of food species.
D. How to increase the food production.
Kiến thức: Đọc hiểu
Giải thích: Cái nào sau đây là tiêu đề tốt nhất của đoạn văn?
A. Sự biến mất của các loài thực phẩm B. Thực phẩm cho
tương lai
C. Các cách để tăng số lượng loài thực phẩmD. Cách tang sản
lượng thực phẩm
Ta thấy, đoạn 1, 2 tác giả nói về thực trạng của việc trồng trọt, canh
tác các loài của người nông dân. Do nhu cầu của con người dẫn đến
nhiều loài có nguy cơ tuyệt chủng.
Ở đoạn 3, 4, tác giả nói về giải pháp cho thực phẩm cho tương lai.
Question 45: According to paragraph 1, why does the writer mention Ireland?
A. To give an example of why it is dangerous to depend on a single crop species.
B. To explain how worldwide interest in crop varieties first developed.
C. To describe how Irish researchers are searching for a new crop variety.
D. To illustrate how important it is to preserve potato plants.
Kiến thức: Đọc hiểu
Giải thích: Theo như đoạn 1, tại sao tác giả lại đề cập Ireland?
A. Để đưa ra một ví dụ về lý do tại sao phụ thuộc vào một loài cây trồng duy nhất là
nguy hiểm.
B. Để giải thích mối quan tâm trên toàn thế giới về các giống cây trồng lần đầu tiên
phát triển như thế nào.
C. Để mô tả cách các nhà nghiên cứu Ireland đang tìm kiếm một giống cây trồng
mới.
D. Để minh họa tầm quan trọng của việc bảo quản cây khoai tây.
Thông tin: But in Ireland, in 1845, people depended almost solely on the potato for
food. The death of one species caused a terrible famine.
Tạm dịch: Nhưng ở Ireland, vào năm 1845, mọi người hầu như chỉ phụ thuộc vào
khoai tây để làm thực phẩm. Cái chết của một loài đã gây ra nạn đói khủng khiếp.
Question 46: The word struck in paragraph 1 is closest in meaning to
_________.
A. approachedB. touched C. damaged D. reached
Kiến thức: Đọc hiểu
Giải thích:
Từ struck trong đoạn 1 gần nghĩa nhất với _________.
A. dính vào B. chạm vào
C. gây tổn hại D. đạt tới
- strike = damage: gây tổn hại, tổn thất
Vậy đáp án đúng là C
Thông tin: In 1845, a deadly disease struck the farms of Ireland, killing
all the Lumper potato plants.
Tạm dịch: Năm 1845, một căn bệnh chết người đã gây tổn hại các
trang trại của Ireland, giết chết tất cả các cây khoai tây Lumper.
Question 47: The word them in the final paragraph refers to
_________.
A. people B. seeds C. foods D. years
Kiến thức: Đọc hiểu
Giải thích: Từ them trong đoạn văn cuối cùng đề cập đến
_________.
A. con người B. hạt giống
C. thực phẩm D. năm
Thông tin: But the people at Heritage Farm don’t just store the
seeds; they plant them.
Tạm dịch: Nhưng những người ở Trang trại Di sản không chỉ lưu trữ
hạt giống; họ trồng chúng.
Question 48: The word flavor in the final paragraph is closest in
meaning to _________.
A. smell B. taste C. sense D. feeling
Kiến thức: Đọc hiểu
Giải thích: Từ flavor trong đoạn văn cuối cùng gần nghĩa nhất với
_________.
A. smell: ngửi B. Taste: nếm
C. sense: giác quan D. feeling: cảm giác
- flavor = taste: hương vị
Vậy đáp án đúng là B
Thông tin: These food species are not just special in terms of
appearance or flavor.
Tạm dịch: Những loài thực phẩm này không chỉ đặc biệt về hình thức
hoặc hương vị.
Question 49. Which of the following is NOT true according to the passage?
A. Nikolay Vavilov make suggestions of collecting plant seeds.
B. In Ireland, in 1845, there was a serious shortage of food.
C. Nikolay Vavilov has worked in many seed banks around the world.
D. Nikolay Vavilov was one of the pioneers to research a new crop.
Kiến thức: Đọc hiểu
Giải thích: Câu nào sau đây là không đúng theo đoạn văn?
A. Nikolay Vavilov đưa ra đề nghị tập hợp những hạt giống
B. Ở Ireland, năm 1845, thực phẩm thiếu trầm trọng
C. Nikolay Vavilov đã làm việc tại nhiều ngân hàng hạt giống trên khắp thế giới. (Không có
thông tin)
D. Nikolay Vavilov là một trong những nhà tiên phong trong nghiên cứu vụ mùa mới.
Thông tin 1: One solution to this problem is to collect and preserve the seeds of as many
different plant varieties as we can before they disappear. The idea was first suggested by
Russian scientist Nikolay Vavilov:
Tạm dịch: Một giải pháp cho vấn đề này là thu thập và bảo quản hạt giống của càng nhiều
giống cây trồng khác nhau càng tốt trước khi chúng biến mất. Ý tưởng lần đầu tiên được đề
xuất bởi nhà khoa học người Nga Nikolay Vavilov.
Thông tin 2. The death of one species caused a terrible famine.
Tạm dịch: Cái chết của một loài đã gây ra nạn đói khủng khiếp.
Question 50. Which of the following can be inferred from Daine Ott Whealy’s
opinion?
A. Historic seeds are usually difficult to reintroduce to the marketplace.
B. It’s important to not only store seeds but also plant them.
C. Food grown from historic and new seeds generally have a similar taste.
D. Storing the seeds is likely to be time-consuming.
Kiến thức: Đọc hiểu
Giải thích: Điều nào sau đây có thể được suy ra từ ý kiến của Daine Ott Whealy?
A. Hạt giống lịch sử thường khó được giới thiệu lại trên thị trường.
B. Điều quan trọng là không chỉ lưu trữ hạt giống mà còn phải trồng chúng.
C. Thức ăn được trồng từ hạt cũ và hạt mới thường có mùi vị tương tự nhau.
D. Bảo quản hạt giống sẽ tốn nhiều thời gian.
Thông tin: By doing this, they are reintroducing foods into the marketplace that
haven’t been grown for years.
Tạm dịch: Bằng cách làm này, họ đang đưa các loại thực phẩm vào thị trường đã
không được trồng trong nhiều năm.
Question 1. In some ways, financial planning for singles is more
________ than for couples.
A. Importable B. importance C. important D. importantly
Kiến thức: Từ loại
Giải thích:
Xét các đáp án:
A. importable : (không có từ này) B. importance (n):
tầm quan trọng
C. important (a): quan trọng D. importantly (adv):
một cách quan trọng
Vị trí chỗ trống này cần một tính từ, vì phía trước có động từ
“to be”. Vì vậy chọn đáp án A.
Tạm dịch: Trong một số trường hợp, kế hoạch tài chính cho
những người độc thân thì quan trọng hơn là đối với các cặp
vợ chồng.
Question 2. It is your homework, _______?
A. isn’t it B. didn’t you C. don’t you D. do you
Kiến thức: Câu hỏi đuôi
Giải thích:
- Chủ ngữ là “it” , phần câu trần thuật động từ chia ở phủ định của thì
quá khứ đơn nên câu hỏi đuôi chia ở khẳng định.
Dịch: Nó là bài tập về nhà của bạn phải không?
Question 3. Cuba can't _________ business in America because
of a nearly 50-year-old trade embargo.
A. set B. do C. stay D. have
Kiến thức: Từ vựng
Giải thích:
Xét các đáp án:
A. Set up in business: mở kinh doanh
B. Do business: làm ăn, kinh doanh
C. Stay business: duy trì kinh doanh
D. Have business: có quyền
Vậy đáp án đúng là B
Tạm dịch: Cu Ba không thể kinh doanh tại Mỹ do việc cấm vận
thương mại trong gần 50 năm qua.
Question 4. Thai Ngo Hieu’ s brave action has
contributed_________beautifying the image of police officers in the
people’s hearts.
A. with B. to C. at D. for
Kiến thức: Giới từ
Giải thích:
- Động từ “ contribute to”: nghĩa là : góp phần vào. Vì vậy nên
đáp án B đúng.
Tạm dịch: Hành động dũng cảm của Thai Ngo Hieu đã góp
phần vào việc làm đẹp hình ảnh người công an trong lòng
nhân dân..
Question 5. I cannot think who had_______ the gaff, but it
seems everyone knows that Nicole and I are planning to get
married.
A. burst B. blown C. split D. banged
Kiến thức: Thành ngữ
Giải thích:
Xét các đáp án :
A. burts (PII) : đốt cháy, thiêu cháy B. blown (PII) : thổi
C. split (PII) : chia ra từng phần/ chia rẽ/ . D. banged (PII) :
đánh mạnh/ đập mạnh
Ta có thành ngữ: - Blow the gaff: tiết lộ bí mật
Vậy đáp án đúng là B
Tạm dịch: Tôi không thể nghĩ là ai đã tiết lộ bí mật , nhưng có vẻ như
mọi người đều biết rằng Nicole và tôi đang chuẩn bị kết hôn.
Question 6. At 84 he's still quite active ______ he walks with the aid
of a stick.
A. because B. because of C. despite D. although
Kiến thức: Liên từ
Giải thích:
Xét các đáp án:
A. Because (+ clause): bởi vì B. Because of (+ V-
ing): bởi vì
C. Despite (+ V-ing/ N.P): mặc dù D. Although (+ clause):
mặc dù
Dựa vào nghĩa, đáp án D là đúng
Tạm dịch: Ở tuổi 84, ông ta khá là năng động mặc dù ông ấy
vẫn phải chống gậy khi đi lại.
Question 7. When I was young, I used to live in
a____________house in the central of Vietnam.
A. small wooden simple B. wooden simple small
C. simple wooden small D. simple small wooden
Kiến thức: Trật tự tính từ
Giải thích: Theo quy tắc trật tự tính từ trong câu: OSASCOMP
simple – opinion; small – size; wooden – Material
Tạm dịch: Khi tôi còn trẻ, tôi đã từng sống trong một ngôi nhà
bằng gỗ nhỏ, giản dijowr miền Trung Việt Nam.
Question 8. Food is likely to __________ if its wrapping is
damaged, since even the tiniest of holes can allow bacteria in.
A. keep onB. put off C. leave out D. go off
Kiến thức: Cụm động từ
Giải thích:
Xét các đáp án:
A. keep on; tiếp tục B. put off:
hoãn
C. leave out: không bao gồm D. go off: nổ, reo, ôi
thiu, ngừng thích
Dựa vào nghĩa và ngữ cảnh, đáp án D là đúng
Tạm dịch: Thức ăn có thể bị ôi thiu nếu phần đóng gói của nó
bị hư hỏng , bởi vì thậm chí những cái lỗ nhỏ nhất cũng có thể
cho phép vi khuẩn xâm nhập.
Question 9. I ________a presentation in front of 500 people
when the microphone stopped working.
A. made B. was making C. had made D. make
Kiến thức: Thì quá khứ tiếp diễn
Giải thích:
Căn cứ vào mệnh đề trạng ngữ chỉ thời gian: S + was/ were +
V-ing when +S + Ved. Diễn tả hành động đang xảy ra thì hành
động khác xen vào.
Vậy vế đầu chia ở thì quá khứ tiếp diễn.
Tạm dịch: Tôi đang phát biểu trước 500 người thì chiếc mic bị
hỏng.
Question 10. We will go on holiday in Nha Trang ___________.
A. as soon as our son finishes the final examination
B. before our son finished the final examination
C. when our son will finish the final examination
D. after our son had finished the final examination
Kiến thức: Sự phối hợp các thì (Tương lai & hiện tại)
Giải thích: Căn cứ vào công thức của mệnh đề trạng ngữ chỉ
thời gian:
Động từ ở mệnh đề chính chia ở thì tương lai + as
soon as + S + V(s/es).
Tạm dịch: Chúng tôi sẽ đi du lịch Nha Trang ngay khi con trai
chúng tôi thi tốt nghiệp xong.
Question 11. The closure of the school is a slap in the________to the
local community.
A. head B. feet C. mouth D. face
Kiến thức: Cụm từ cố định
Giải thích: a slap in the face: sự sỉ nhục/ sự lăng mạ
Tạm dịch: Sự đóng cửa trường học là một sự sỉ nhục đối với
cộng đồng địa phương
Question 12. Researchers fear the virus could ________ and
become transmissible between humans.
A. vanish B. fade C. mutate D. rotate
Kiến thức: Từ vựng
Giải thích:
Xét các đáp án :
A. vanish (verb): biến mất B. fade (verb):
mờ dần; bạc màu
C. mutate (verb): đột biến, biến đổi D. rotate
(verb): làm xoay quanh; luân phiên nhau
Dựa nghĩa và ngữ cảnh, đáp án đúng là C
Tạm dịch : Các nhà nghiên cứu đang lo sợ vi rút có thể biến đổi và trở
nên dễ dàng lây nhiễm trên con người
Question 13. Miss Vietnam Do Thi Ha __________ in the list of 13
finalists in the Top Model competition at the Miss World 2021 beauty
pageant on December 7.
A. was namedB. named C. was naming D. names
Kiến thức: Câu bị động
Giải thích:
Căn cứ vào nghĩa, ta cân chia động từ ở thể bị động vì “Đỗ Thị
Hà” “đã được gọi tên” chứ không thể tự mình gọi tên được. Do đó,
động từ chia ở bị động và ở “ dạng “was + PII”
Tạm dịch: Hoa Hậu Đỗ Thị Hà đã được gọi tên vào danh sách 13 thí
sinh xuất săc trong phần thi Siêu Mẫu tại cuộc thi Hoa Hậu Thế Giới
vào ngày 7thangs 12.
Question 14________ every major tennis title, Alan retired from
international competition.
A. Having won B. Winning C. When he won D. On
winning
Kiến thức : Rút gọn mệnh đề trạng ngữ
Giải thích:
Khi hai mệnh đề cùng chủ ngữ, và câu muốn nhấn mạnh hành
động phía trước được hoàn thành xong trước rồi hành động
phía sau mới xảy ra thì chúng ta dùng công thức:
Having + P2, S+Ved.
Đáp án đúng là A.
Tạm dịch: Sau khi giành được danh hiệu quan trọng trong môn
quần vợt , Alan đã rời khỏi các cuộc thi đấu quốc tê .
Question 15. The harder he studies, ______ results he gets.
A. the best results he gets B. his results get better
C. the better do his results get D. the better
Kiến thức: So sánh kép
Giải thích:
Ta có công thức so sánh kép:
The comparative + S1 + V1, the comparative + S2 + V2: càng …., càng
….
Xét các đáp án:
A. the best results he gets: sai B. his results get better :
sai
C. the better do his results get: sai D. the better : đúng
Tạm dịch: Anh ấy càng học hành chăm chỉ, anh ấy càng đạt được kết
quả tốt.
Question 16. The hotel receptionist is asking the customer his name.
Receptionist: “Could you please say your name again please?”
Customer: “__________”
A. How can you say so? B. Why do I have to?
C. No, I don’t remember. D. Sure, that’s Robert.
Kiến thức: Tình huống giao tiếp (Đáp lại lời đề nghị)
Giải thích:
Nhân viên lễ tân khách sạn đang hỏi khách hàng về tên của anh ta.
- Nhân viên lễ tân: " Ông/anh làm ơn có thể nói lại tên của mình được
không?"
- Khách hàng: "_______."
Xét các đáp án:
A. Sao ông có thể nói thế B. Sao tôi phải làm vậy
C. Không, tôi không nhớ D. Chắc chắn rồi, đó là Robert
Question 17. Jolie is reading a magazine on famous people, asking her
friend.
Jolie: “Do you think celebrities today tend to focus more on wealth rather
than achievements?”
Jolie's friend: “__________________. And this sets bad examples for
young people.”
A. Not at all B. You can say that again
C. I think they focus more on achievements D. It's out of the question
Kiến thức: Tình huống giao tiếp (Hỏi về quan điểm của người khác)
Giải thích:
Jolie đang đọc một cuốn tạp chí về người nổi tiếng, hỏi bạn cô ấy.
- Jolie: "Bạn có nghĩ những người nổi tiếng ngày nay có xu hướng tập trung vào tiền
bạc hơn về những thành tích không?."
- Bạn của Jolie: "_______, và điều này tạo ra những tấm gương xấu cho các bạn trẻ"
Xét các đáp án:
A. Hoàn toàn không. B. Tôi đồng ý.
C. Tôi nghĩ họ tập trung nhiều hơn về những thành tích D. Nó là vấn đề
không thể bàn đến
Question 18.
A. publicity B. chemistry C. attention D. romantic
Kiến thức: Trọng âm của từ 3 âm tiết trở lên
Giải thích:
A. publicity /pʌbˈlɪsəti/ (n): Trọng âm rơi vào âm tiết thứ hai
vì đuôi –ITY – trọng âm rơi vào âm tiết liền trước.
B. chemistry /ˈkemɪstri/ (n): Trọng âm rơi vào âm tiết thứ
nhất
C. attention /əˈtenʃn/ (n): Trọng âm rơi vào âm tiết thứ hai vì
đuôi – TION- trong âm rơi vào âm liền trước
D. romantic /rəʊˈmæntɪk/ (a): Trọng âm rơi vào âm tiết thứ
hai vì đuôi – IC - trong âm rơi vào âm liền trước
Question 19.
A. attract B. discuss C. follow D. confide
Kiến thức: Trọng âm của từ 2 âm tiết
Giải thích:
- Đáp án C có trọng âm rơi vào âm tiết thứ 1. Các đáp án còn
lại có trọng âm rơi vào âm tiết thứ 2.
A. attract /əˈtrækt/ (v): B. discus /dɪˈskʌs/ (v):
C. follow /dɪˈskʌs/ (v): D. confide /kənˈfaɪd/ (v):
* Note - Động từ 2 âm tiết – trọng âm thường rơi vào âm tiết
thứ hai. Trừ những động từ có tận cùng là : er/ ow / en / ish
trong âm rơi vào âm tiết số 1
Question 20.
A. accessed B. searched C. recorded D. developed
Question 20. Đáp án C
Kiến thức : Cách phát âm đuôi -ED
Giải thích:
– Phát âm là /t/ khi từ có tận cùng bằng các phụ âm vô thanh:
/θ/, /p/, /k/, /f/, /s/, /ʃ/, /tʃ/
– Phát âm là /id/ khi từ có tận cùng là các âm: /t/, /d/
– Phát âm là /d/ khi các từ có tận cùng là nguyên âm và các
phụ âm hữu thanh còn lại
Xét các đáp án:
A. accessed B. searched
C. recorded D. developed
Question 21.
A. mouth B. founder C. around D. country
Kiến thức: Cách phát âm của nguyên âm
Giải thích:
A. mouth /maʊθ/ B. founder /ˈfaʊndə(r)/
C. around /əˈraʊnd/ D. country /ˈkʌntri/
Ta thấy: Các phương án A, B, C – phần gạch chân được phát âm là
/aʊ/
Phương án D – phần gạch chân được phát âm là /ʌ/
Question 22. Some students only cram for tests when there is little
time left, so their results are not satisfactory.
A. prepare in a short period B. prepare in a long time
C. prepare well D. prepare badly
Kiến thức: Đồng nghĩa (từ đơn)
Giải thích:
cram (v): học gạo ( học trong khoảng thời gian ngắn để thi)
Xét các đáp án:
A. prepare in a short period: chuẩn bị trong thời gian ngắn
B. prepare in a long time: chuẩn bị trong thời gian dài
C. prepare well: chuẩn bị tốt
D. prepare badly: chuẩn bị không tốt
Question 23. Please, you are so nervous, do try to contain
your anger.
A. hold back B. consult C. consume D. contact
Kiến thức: Đồng nghĩa (từ đơn)
Giải thích:
Contain (v) : chứa đựng/ kiềm chế/ ngăn lại
Xét các đáp án:
A. hold back: ngăn lại/ giữ lại/kìm lại
B. consult(v) hỏi ý kiến/ tham khảo
C. consume(v) tiêu thụ/ tiêu dùng D. contact (v) liên
lạc
Vậy đáp án đúng là A
Tạm dịch: Bạn đang rất căng thẳng , làm ơn cố gắng kiềm chế
cơn tức giận.
Question 24 The principle of use and disuse states that those
parts of organisms' bodies that are used grown larger while
those parts that are not tend to wither away.
A. split B. swell C. perish D. shrink
Kiến thức: Trái nghĩa (cụm từ hoặc thành ngữ)
Giải thích:
Ta có: - wither away: trở nên yếu đi/ nhỏ lại
Xét các đáp án:
A. split (v) chia/ tách ra B. swell (v) phồng lên/ to lên
C. perish (v) diệt vong/ bỏ mạng D. shrink (v) nhỏ lại
Vậy đáp án đúng là B
Wither away >< swell
Question 25. He is from such an unemotional family, he will
never learn to unleash his feelings.
A. describe B. conceal C. release D. extend
Kiến thức: Trái nghĩa (từ đơn)
Giải thích:
Ta có: unleash (v): buông lỏng/ giải phóng/ thể hiện ( cảm xúc).
Xét các đáp án:
A. describe (v) miêu tả B. conceal (v) che dấu
C. release (v) thả/ phóng thích/làm nhẹ bớt D. extend (v)
mở rộng
Question 26. My eldest sister graduated from university. She soon
started working as a freelance journalist.
A. Hardly had my eldest started working as a freelance journalist
when she graduated from university.
B. No sooner had my eldest sister graduated from university than
she started working as a freelance journalist.
C. No sooner had my eldest sister started working as a freelance
journalist than she graduated from university.
D. After my eldest sister graduated from university, she had started
working as a freelance journalist.
Kiến thức: Kết hợp câu – đảo ngữ
Giải thích:
Ta có công thức đảo ngữ:
No sooner + had + S + PII than + S + Ved
Câu đề bài: Chị gái tôi tốt nghiệp đại học. Chị ấy sớm bắt đầu làm
một nhà báo tự do.
Question 27. You didn’t give me a chance to tell you the truth. I regret that.
A. If you gave me a chance, I could tell you the truth.
B. I wish you had given me a chance to tell you the truth.
C. I wish you didn’t give me a chance to tell you the truth.
D. If only you hadn’t given me a chance to tell you the truth.
Kiến thức: Kết hợp câu – câu điều ước
Giải thích:
Giải thích: Câu đề bài: Bạn không cho tôi cơ hội nói cho bạn biết sự thật. Tôi thấy
tiếc vì điều đó
Ta thấy câu này diễn đạt hành động trong quá khứ, nên cần có đáp án diễn tả điều
ước ở quá khứ
A. Nếu bạn cho tôi cơ hội, tôi đã có thể nói cho bạn sự thật.=> sai cấu trúc ( câu
điều kiện loại 2- hiện tại)
B. Tôi ước bạn đã cho tôi cơ hội để nói sự thật => đúng ( câu ước ở quá khứ)
C. Tôi ước bạn đã không cho tôi cơ hội để nói sự thật.=> sai nghĩa + sai cấu trúc (
câu ước ở hiện tại)
D. Tôi ước bạn đã không cho tôi cơ hội để nói sự thật. => sai nghĩa
Question 28. We have a very happy party at Tom’s house last
weekend.
A. have B. very C. party D. Tom’s
Kiến thức: Lỗi sai – Thì của động từ
Giải thích:
Ta thấy thời gian: last weekend nên động từ phải chia ở thì quá
khứ.
Vậy đáp án đúng là A
Sửa lỗi: have → had
Tạm dịch: Chúng tôi đã có một bữa tiệc rất vui tại nhà Tom cuối tuần
trước.
Question 29. Everyone has its own ideas about the best way to bring
up children.
A. its B. about C. way D. children
Kiến thức: Lỗi sai – Đại từ nhân xưng/ tính từ sở hữu
Giải thích:
Ta thấy Everyone là đại từ bất định chỉ người nên tính từ sở hữu
tương ứng là their hoặc his/her.
Vậy đáp án đúng là A
Sửa lỗi: its → their, his/her
Tạm dịch: Mỗi người có ý tưởng riêng của mình về cách tốt nhất để
nuôi nấng con cái.
Question 30. Young people of an impressive age can easily be
manipulated.
A. Yong B. impressive C. easily D. manipulated
Kiến thức: Lỗi sai – Từ vựng
Giải thích: impressive (a) = gây ấn tượng sâu sắc/gây xúc động ;
impressionable (a) = nhạy cảm/ dễ bị ảnh hưởng
Sửa lỗi: impressive → impressionable
Tạm dịch: Các bạn trẻ đang ở độ tuổi nhạy cảm có thể dễ dàng bị dụ
dỗ/ lôi kéo.
Question 31. I am sure he said something terrible to her because of her red eyes.
A. He must have said something terrible to her because I could see it from her red
eyes
B. He could have said something terrible to her because I could see it from her red
eyes
C. He should have said something terrible to her because I could see it from her red
eyes
D. He may have said something terrible to her because I could see it from her red
eyes
Kiến thức: Câu đồng nghĩa – Động từ khuyết thiếu
Giải thích:
Sử dụng cấu trúc “động từ khuyết thiếu + (not) + have + Vp2” để đưa ra dự đoán, lời
khuyên,… cho các hành động trong quá khứ.
Câu đề bài: Tôi chắc chắn là anh ta đã nói điều gì đó rất tệ với cô ta vì mắt cô ấy rất đỏ.
Các phương án:
A. Anh ta chắc chắn đã nói điều gì đó rất tệ với cô ta bởi vì tôi có thể thấy điều đó từ đôi mắt
đỏ của cô ta.
B. Anh ta lẽ ra đã có thể nói điều gì đó rất tệ với cô ta bởi vì tôi có thể thấy điều đó từ đôi mắt
đỏ của cô ta.
C. Anh ta lẽ ra đã nên nói điều gì đó rất tệ với cô ta bởi vì tôi có thể thấy điều đó từ đôi mắt đỏ
của cô ta.
D. Anh ta có lẽ là đã nói điều gì đó rất tệ với cô ta bởi vì tôi có thể thấy điều đó từ đôi mắt đỏ
của cô ta.
Phù hợp với nghĩa câu gốc nhất là đáp án A.
Question 32. “What have you done to your hair?” she said to her
son.
A. She asked her son what to do to his hair.
B. She wanted her son to know what he had done to his hair.
C. She wanted to know what did her son do to his hair.
D. She asked her son what he had done to his hair.
Kiến thức: Câu đồng nghĩa – Câu tường thuật
Giải thích:
Đề bài: "Con đã làm gì với tóc của con vậy", Cô ta nói với con trai mình.
Ta nhận thấy đây là một câu hỏi ở thì hiện tại hoàn thành - lùi thì về quá
khứ hoàn thành
A. Cô ta hỏi con trai phải làm gì với mái tóc của nó. => sai nghĩa
B. Cô ta muốn con trai biết nó đã làm gì với mái tóc của nó => Sai nghĩa
C. Cô ta muốn biết con trai cô ta đã làm gì với mái tóc của nó.=> Đáp án
này sai vì chưa lùi thì và đảo ngữ
D. Cô ta muốn biết con trai cô ta đã làm gì với mái tóc của nó. => Đáp án
này đúng.
Question 33. It’s almost nine months since I last subscribed to
that magazine.
I haven’t subscribed to that magazine for almost nine months.
I have subscribed to that magazine almost nine months ago.
I subscribed to that magazine almost for nine months .
I have subscribed to that magazine for almost nine months, but
now I stopped.
Kiến thức: Câu đồng nghĩa – Thì của động từ
Giải thích:
Câu đề bài: Gần chín tháng kể từ lần cuối cùng tôi đăng ký tạp chí
đó.
= A. Tôi không đăng ký tạp chí đó được gần chín tháng rồi.
Cấu trúc: S + haven’t/ hasn’t + Vp2 + for + time = It ‘s + time + since
+ S + (last) + Vp1 + .
Question 34: A. making B. providing C. allowing D.
holding
Countries all over the world are now beginning to enter the
sector of eLearning, (34) ______ people to have a broader
access to learning opportunities that weren't otherwise
possible in the past.
Kiến thức: Đọc điền từ
Giải thích:
A. make sb + Vnt: bảo ai làm gì B. provide sb with st:
cung cấp cho ai cái gì
C. allow sb to + Vnt: cho phép ai làm gì D. hold sb/ st: cầm /
nắm …
Ta thấy chỗ trống cần một động từ + tân ngữ + to Vnt
Question 35: A. who B. whom C. when D. why
The reason why this industry has become so popular over the
last few years is the convenience it offers to those (35)
______ are interested in it.
Kiến thức: Đọc điền từ
Giải thích:
A. who: Thay thế cho danh từ người có chức năng làm chủ ngữ và
tân ngữ trong mệnh đề quan hệ.
B. whom: Thay thế cho danh từ người có chức năng làm tân ngữ
trong mệnh đề quan hệ.
C. when: Thay thế cho trạng từ chỉ thời gian có chức năng làm bổ
ngữ trong mệnh đề quan hệ
C. why: Thay thế cho trạng từ chỉ lý do có chức năng làm bổ ngữ
trong mệnh đề quan hệ
Ta có cấu trúc : those = those people => cần đại từ quan hệ thay thế
cho người làm chủ ngữ
Question 36: A. constitution B. institution C. restitution D.
destitution
Traditionally, if a person wanted to learn something or that can
teach you to get a degree, they would have to go to a university,
a college, or any (36) ______ giving you a certification at the
end.
Kiến thức: Từ vựng
Giải thích:
A. constitution (n) hiến pháp B. institution (n) cơ sở giáo dục
C. restitution (n) sự hoàn lại D. destitution (n) sự cơ cự , thiếu thốn
Tạm dịch: Theo truyền thống, nếu một người nào đó muốn học một
thứ gì đó hoặc có thể dạy bạn để lấy bằng, họ phải học đại học , cao
đẳng hoặc bất kỳ một cơ sở giáo dục nào mà cấp chứng chỉ cho bạn.
Question 37:
A. Because B. However C. Therefore D. Even though
(37)_________there are several benefits to developing
eLearning sectors in countries, there are still a few nations that
are moving relatively backward in this industry.
Kiến thức: Đọc điền từ
Giải thích:
Xét các đáp án:
A. Because: bởi vì B. However: tuy nhiên
C. Therefore: cho nên D. Eventhough: mặc dù
Tạm dịch: Mặc dù có một số lợi ích đối với việc phát triển các khu
vực eLearning ở các quốc gia, nhưng vẫn còn một vài quốc gia đang
khá là lạc hậu đối với nghành công nghiệp này
Question 38: A. each B. others C. every D. another
There are, of course, countries that have already gone far ahead
of (38)_________, setting up more and more eLearning
platforms.
Kiến thức: Đọc điền từ
Giải thích:
Xét các đáp án:
A. each (+ N –số ít ) : mỗi một
B. others : những người khác
C. every ( + N-số ít) : mọi/ mỗi….
D. another ( + N-số ít) : một cái/người khác nữa
Tạm dịch: Tất nhiên có những quốc gia vượt xa các quốc gia khác, thiết
lập càng nhiều nền tảng eLearning
Question 39. What could be the best title for the passage?
A. legal limit for driving B. driving ability
C. drinking and driving D. standard of drinks
Kiến thức: Đọc hiểu
Giải thích:
Câu nào có thể là tiêu đề hay nhất cho đoạn văn.
A. Giới hạn hợp pháp cho việc lái xe B. Khả năng lái xe
C. Uống rượu và lái xeD. Tiêu chuẩn của việc uống rượu
Bài viết nói về mối liên quan giữa việc uống rượu và lái xe
Question 40. The word “breathalyzer” in paragraph 5 means ______ .
A. a device used by the police to measure the amount of alcohol in a
driver’s breath.
B. a device used to analyse driver’s blood type.
C. a person who measures the amount of alcohol in a driver’s breath.
D. a person who has to take a breath test
Kiến thức: Đọc hiểu
Giải thích:
Từ “ breathalyser” trong đoạn 5 có nghĩa là:
A. một thiết bị được sử dụng bởi cảnh sát để đo nồng độ cồn trong
hơi thở của người lái xe
B. một thiết bị được sử dụng để phân tích nhóm máu của người lái xe
C. một người mà đo nồng độ cồn trong hơi thở của người lái xe
D. một người mà phải kiểm tra hơi thở
-“ breathalyser”: dụng cụ kiểm tra lượng rượu trong hơi thở.
Question 41. The word “they” in paragraph 4 refers to ______ .
A. professional drivers B. moveable posts
C. alcoholic drinks D. tests
Kiến thức: Đọc hiểu
Giải thích:
Từ “they” trong đoạn 4 là chỉ:
A. Những người lái xe chuyên nghiệp B. Những cột mốc có thể
di chuyển được
C. Những đồ uống có cồn D. những bài
kiểm tra
Thông tin: “In a test with professional drivers, the more alcoholic
drinks they had had, the more certain they were that they could
drive a test course through a set of moveable posts … and the less
able they were to do it!
Question 42. According to the passage, which of the following is TRUE about
the amount of alcohol a person can drink before reaching the legal limit ?
A. 800 milligrams of pure alcohol B. approximately
three standard drinks
C. exactly proportional to body weight D. different for different
people
Kiến thức: Đọc hiểu
Giải thích:
Lượng rượu mà một người có thể uống trước khi đạt đến giới hạn
pháp lý là .
A. 800 mg rượu nguyên chất B. tỷ lệ chính xác với trọng lượng cơ thể
C. khoảng ba ly thông thường D. khác nhau với những người
khác nhau
Thông tin: But there is no sure way of telling how much you can drink
before you reach this limit. It varies with each person depending on
your weight, your sex, if you‟ve just eaten and what sort of drinks
you‟ve had.
Question 43. When might you be taken to court by the police for drinking and
driving?
A. When the police think that you have been drinking from the way you are
driving.
B. When you have driven a vehicle after drinking any alcohol at all.
C. Only when tests show that you have 80mg of alcohol in 100 ml of blood.
D. When you have drunk at least three drinks before driving.
Kiến thức: Đọc hiểu
Giải thích:
Khi nào bạn có thể bị cảnh sát đưa ra tòa vì uống rượu và lái xe?
A. Khi cảnh sát nhận ra rằng bạn đã uống rượu từ cách bạn lái xe.
B. Chỉ khi xét nghiệm cho thấy bạn có 80mg cồn trong 100 ml máu.
C. Khi bạn lái xe sau khi uống bất kỳ chút đồ uống có cồn nào.
D. Khi bạn đã uống ít nhất ba ly trước khi lái xe.
Thông tin: Even if you‟re below the legal limit, you could still be taken to
court if a police officer thinks your driving has been affected by alcohol.
Question 44. Alternative forms of health care are becoming more
popular because _______.
A. they have been used for a very long time
B. they are able to cure patients much more quickly
C. other ways aren't able to get rid of the symptoms as well as them
D. medical costs are on the upswing in many developed countries
Kiến thức: Đọc hiểu
Giải thích:
Các hình thức chăm sóc sức khỏe thay thế đang trở nên phổ biến
hơn vì _______.
A. chúng đã được sử dụng trong một thời gian rất dài
B. chúng có thể cứu chữa bệnh nhân nhanh hơn nhiều
C. những cách khác không thể loại bỏ các triệu chứng cũng như
chúng
D. chi phí y tế đang tăng lên ở nhiều nước phát triển
Thông tin: As health care costs continually rise in most developed
countries, many people are looking for alternative forms of health
therapy. Two of the most popular forms of alternative therapies in the
West today are acupuncture and herbal medicine,…
Question 45. The word "crux" in paragraph 1 mostly means
_______.
A. basis B. relief C. symptom D. application
Kiến thức: Đọc hiểu – từ vựng
Giải thích:
Từ “crux” trong đoạn 1 thì hầu hết có nghĩa là ________.
A. basis: cơ sở, căn cứ
B. relief: cứu trợ
C. symptom: triệu chứng
D. application: ứng dụng
=> crux (n): mấu chốt, điểm then chốt = basis
Question 46. The word "solely" in paragraph 3 is CLOSEST in
meaning to _______.
A. safely B. barely C. intentionally D. exclusively
Kiến thức: Đọc hiểu – từ vựng
Giải thích:
Từ “solely” trong đoạn 3 thì gần nghĩa nhất với ______.
A. safely (adv): an toàn
B. barely (adv): vừa đủ
C. intentionally (adv): cố ý
D. exclusively (adv): duy nhất (for only one particular person, group
or use)
=> solely (adv): only; not involving somebody/something else (chỉ
….) = exclusively
Thông tin: This technique is involved solely with the ear and its
numerous activation points.
Tạm dịch: Kỹ thuật này chỉ liên quan đến tai và nhiều điểm kích hoạt
của nó.
Question 47. It can be inferred from paragraph 3 that the ear _______.
A. has a healthy blood supply, which relieves the pain from needles
B. contains a small number of the body's special activation points
C. is a beneficial acupuncture area because it contains a myriad of nerves
D. is the source of dependency issues such as alcoholism and drug use
Kiến thức: Đọc hiểu – suy luận
Giải thích:
Có thể suy ra từ đoạn 3 rằng tai _______.
A. có nguồn cung cấp máu lành mạnh, giúp giảm đau do kim tiêm
B. chứa một số lượng nhỏ các điểm kích hoạt đặc biệt của cơ thể
C. là vùng huyệt đạo có lợi vì chứa vô số dây thần kinh
D. là nguồn gốc của các vấn đề phụ thuộc như nghiện rượu và sử dụng ma
túy
Thông tin: This technique is involved solely with the ear and its numerous
activation points. The ear is considered a highly sensitive area of the human
body because it contains a strong, healthy flow of blood as well as numerous
nerve points that connect it with the rest of the human body. Acupuncturists
place tiny needles in the surface of the skin at certain points, depending on the
desired result. In general, ear acupuncture is considered an excellent therapy,
especially for treating individuals with psychological or physical dependency
issues such as eating disorders and drug or alcohol abuse.
Question 48. The word "it" in paragraph 3 refers to _______.
A. the ear B. the activation point C. the nerve point D.
the flow of blood
Kiến thức: Đọc hiểu – từ thay thế
Giải thích:
Từ “it” trong đoạn 3 đề cập đến _______.
A. tai
B. 1 điểm kích hoạt
C. 1 điểm thần kinh
D. dòng chảy của máu
Thông tin: The ear is considered a highly sensitive area of the human body
because it contains a strong, healthy flow of blood as well as numerous
nerve points that connect it with the rest of the human body.
Tạm dịch: Tai được coi là khu vực nhạy cảm cao của cơ thể con người vì nó
(tai) chứa một dòng chảy của máu lưu thông mạnh mẽ, khỏe mạnh cũng
như nhiều điểm thần kinh mà kết nối nó (tai) với phần còn lại của cơ thể
con người.
Question 49. The author's description of herbal therapy mentions all of the following
EXCEPT _______.
A. the reaction time is faster for acupuncture than for herbal therapy
B. herbs are an excellent way of increasing a person's endurance
C. it is used as a preventative form of therapy for healthy people
D. it is beneficial to the circulatory system and blood pressure problems
Kiến thức: Đọc hiểu – chi tiết
Giải thích:
Mô tả của tác giả về liệu pháp thảo dược đề cập đến tất cả những điều sau NGOẠI
TRỪ _______.
A. thời gian phản ứng đối với châm cứu nhanh hơn so với liệu pháp thảo dược
B. các loại thảo mộc là một cách tuyệt vời để tăng sức bền của một người
C. nó được sử dụng như một hình thức trị liệu phòng ngừa cho những người khỏe
mạnh
D. nó có lợi cho hệ tuần hoàn và các vấn đề về huyết áp
Thông tin:
- Traditionally, herbal teas are imbibed to boost the immune system and prevent
illnesses from being able to enter the body.
- Herbs also have a more direct influence than acupuncture on the body's physical
system, as they can aid in normalizing blood pressure.
- Other benefits attributed to the use of traditional Chinese herbs are that they build
stamina and are good for digestive purposes.
Question 50. What is the passage mainly about?
A. Acupuncture – the best therapy. B. Using plants for
healing the injuries.
C. Two forms of alternative therapies in the West. D. Western
medical techniques.
Kiến thức: Đọc hiểu – chi tiết
Giải thích: Đoạn văn này chủ yếu nói về điều gì?
A. Châm cứu – liệu pháp chữa bệnh tốt nhất
B. Việc sử dụng thảo dược để chữa trị các vết thương.
C. Hai liệu pháp chữa bệnh thay thế ở phương Tây
D. Kỹ thuật y học ở phương tây
Thông tin:
“ As health care costs continually rise in most developed countries,
many people are looking for alternative forms of health therapy. Two of
the most popular forms of alternative therapies in the West today are
acupuncture and herbal medicine, both of which have been used for
centuries in Asia and are rooted in ancient Chinese medicine.”
Question 1. If you don't have anything _______ to say, it's
better to say nothing.
A. construct B. construction C. constructive
D. constructor
Kiến thức: Từ loại
Giải thích:
Chỗ trống cần 1 tính từ vì tính từ đứng sau các đại từ bất định
như : something, anything, somebody, anybody …
Xét các đáp án :
A. construct /kənˈstrʌkt/ (v): xây dựng
B. construction /kənˈstrʌkʃn/ (n): sự xây dựng
C. constructive /kənˈstrʌktɪv/ (a): có tính xây dựng
D. constructor /kənˈstrʌktə(r)/ (n): kỹ sư xây dựng
Question 2. Neither of the boys came to school yesterday, _______?
A. didn’t they B. does they C. did they
D. doesn’t they
Kiến thức: Câu hỏi đuôi
Giải thích:
Ta có công thức của câu hỏi đuôi:
S + V + O..., TRỢ ĐỘNG TỪ + ĐẠI TỪ?
Chủ ngữ của mệnh đề chính là “the boys” nên đại từ ở câu hỏi đuôi phải chuyển
thành “they”
Động từ “came” được chia ở thì quá khứ đơn thì trợ động từ ở câu hỏi đuôi là
“did” → loại B và D
Tuy nhiên trong mệnh đề chính không xuất hiện từ not mang nghĩa phủ định
nhưng có một từ mang nghĩa phủ định đó là Neither (không cái nào/người nào
trong hai) → trợ động từ câu hỏi đuôi phải ở dạng khẳng định → loại A và chọn
được C.
* Note:
Với những câu hỏi về câu hỏi đuôi nếu mệnh đề chính không có NOT nhưng xuất
hiện các từ mang nghĩa phủ định như never, little, seldom, neither, hardly, scarely,
... thì trợ động từ câu hỏi đuôi ở dạng khẳng định.
Question 3. It's a little _______ that no one knows where he was at
the time of the murder.
A. doubtful B. suspicious C. skeptical D. dubious
Kiến thức: Từ vựng
Giải thích:
It's a little ______ that no one knows where he was at the time
of the murder.
A. doubtful B. suspicious C. skeptical D. Dubious
Question 4. Television can make things memorable for the
reason that it presents information _______ an effective way.
A. over B. with C. in D. on
Kiến thức: Giới từ
Giải thích:
Khi muốn nói theo một cách nào đó ta dùng cụm giới từ in
....way. Cụ thể trong câu này là theo một cách hiệu quả. Các
phương án còn lại không dùng với nghĩa này nên đáp án là C.
In an effective way: một cachs hiệu quả
Tạm dịch: Truyền hình có thể khiến mọi thứ trở nên đáng nhớ
vì nó trình bày thông tin theo một cách hiệu quả.
Question 5. Someone stopped me on the street and offered to
sell me a gold watch for five dollars. I could _______ a rat.
A. taste B. see C. think D. smell
Kiến thức: Thành ngữ
Giải thích:
Ta có thành ngữ: smell a rat: tỏ ý nghi ngờ có một điều gì đó
Vậy đáp án đúng là D
Tạm dịch: Ai đó đã chặn tôi lại trên phố và đề nghị bán cho tôi một
chiếc đồng hồ vàng với giá năm đô la. Tôi có thể nghi ngờ có gì đó sai
sai.
Question 6. All activities have stopped _______ he weather has
been terrible since yesterday.
A. although B. because of C. because D. despite
Kiến thức: Liên từ
Giải thích:
Xét các đáp án:
A. Although (+ clause): mặc dù B. Because of (+ V-ing): bởi vì
C. Because (+ clause): bởi vì D. Despite (+ V-ing/ N.P): mặc dù
Dựa vào nghĩa, đáp án đúng là C
Tạm dịch: Mọi hoạt động đã dừng lại vì thời tiết từ hôm qua trở nên
kinh khủng.
Question 7. Her husband bought her a _______ scarf when he
went on holiday in Singapore last week.
A. beautiful silk yellow B. yellow beautiful silk
C. beautiful yellow silk D. silk beautiful yellow
Kiến thức: Trật tự tính từ
Giải thích:
Vị trí của tính từ trước danh từ: Opinion (quan điểm) –
beautiful + Color (màu sắc) – yellow + Material (chất liệu) – silk
+ Noun (danh từ)
Vậy đáp án đúng là C
Tạm dịch: Chồng cô đã mua cho cô chiếc khăn lụa màu vàng
tuyệt đẹp khi anh đi nghỉ ở Singapore tuần trước.
Question 8. There are other problems of city life which I don't
propose to _______ at the moment.
A. go into B. go around C. go for D. go up
Kiến thức: Cụm động từ
Giải thích:
A. go into (phr.v): bắt đầu làm gì; thảo luận, kiểm chứng, mô tả, giải
thích một cách chi tiết và thận trọng
B. go around (phr.v): đủ cho mọi người trong nhóm; đến thăm ai; cư
xử tệ
C. go for (phr.v): chọn, say mê, cố gắng
D. go up (phr.v): tăng lên
Tạm dịch: Còn nhiều vấn đề khác về cuộc sống thành thị cái mà tôi
không muốn đi vào sâu để thảo luận kỹ ngay lúc này.
Question 9. When Carol called last night, I _______ my favorite
show on television.
A. was watching B. watched C. have watched
D. watch
Kiến thức: Thì quá khứ tiếp diễn
Giải thích:
Trong quá khứ, một sự việc đang diễn ra thì sự việc khác xen
vào:
+ Sự việc đang diễn ra chia ở thì quá khứ tiếp diễn: S +
was/were + V.ing => was watching
+ Sự việc xen vào chia ở thì quá khứ đơn: S + V.ed => called
Tạm dịch: Tối qua, khi Carol gọi thì tôi đang xem chương
trình yêu thích của mình trên tivi.
Question 10. _______, I will remind him to call for you.
A. when he was turning up B. until he turned up
C. after had turned up D. When he turns up
Kiến thức: Sự phối hợp các thì (Tương lai & hiện tại)
Giải thích:
Ta thấy, động từ của mệnh đề chính ở tương lai đơn nên động từ của
mệnh đề trạng ngữ chỉ thời gian chia ở hiện tại.
Vậy, đáp án D đúng.
Tạm dịch: Khi anh ấy đến, tôi sẽ nhắc anh ấy gọi cho bạn.
Question 11. Researchers have to _______ the conclusion that
your personality is affected by your genes.
A. arrived B. come C. got D. reached
Kiến thức: Cụm từ cố định
Giải thích:
Ta có cụm từ cố định: Come to conclusion: đưa ra kết luận
Vậy đáp án đúng là B
Tạm dịch: Các nhà nghiên cứu phải đưa ra kết luận rằng tính cách của
bạn bị ảnh hưởng bởi gen của bạn.
Question 12. Closer _______ of the documents revealed a
number of interesting and important facts.
A. scrutiny B. inception C. peculiarity D. Suspicion
Kiến thức: Từ vựng
Giải thích:
Xét các đáp án :
A. scrutiny : xem xét kỹ lưỡng B. inception : khởi đầu
C. peculiarity : đặc biệt D. suspicion : nghi ngờ
Ta có cụm từ : close scrutiny : xem xét kĩ lưỡng
Vậy đáp án đúng là A
Tạm dịch: Việc xem xét kỹ hơn các tài liệu đã tiết lộ một số sự kiện
thú vị và quan trọng.
Question 13. It is claimed that new nuclear power plants designed
to be safer than the current ones _______.
A. will build B. build C. are soon built D. built
Kiến thức: Câu bị động
Giải thích:
Căn cứ vào nghĩa, “new nuclear power plants” - những nhà
máy điện nguyên tử thì động từ phải chia ở thể bị động. Do đó
loại A,B, D
Vậy đáp án đúng: C
Tạm dịch: Người ta cho rằng các nhà máy điện hạt nhân mới
được thiết kế để an toàn hơn các nhà máy hiện tại sẽ sớm
được xây dựng.
Question 14. _______ water resources irresponsibly, the
authorities had to deal with water shortages in the region.
A. Managed B. Having managed
C. To manage D. Being managed
Kiến thức : Rút gọn mệnh đề trạng ngữ
Giải thích:
Khi hai mệnh đề cùng chủ ngữ, và câu muốn nhấn mạnh hành
động phía trước được hoàn thành xong trước rồi hành động
phía sau mới xảy ra thì chúng ta dùng công thức:
Having + V-p2, S+Ved.
Đáp án đúng là B.
Tạm dịch: Quản lý tài nguyên nước thiếu trách nhiệm, các cơ quan
chức năng đã phải đối mặt với tình trạng thiếu nước trong khu vực.
Question 15. The harder you study for these exams, _______ you
will do.
A. the better B. the much C. the best D. the more good
Kiến thức: So sánh kép
Giải thích:
Xét các đáp án:
A. the better → cấu trúc so sánh kép: The more/ -er + S + V,
the more/ -er + S + V
B. the much → sai cấu trúc so sánh kép
C. the best → sai cấu trúc so sánh kép
D. the more good → sai cấu trúc so sánh hơn (hard là từ 1
âm tiết, good – bất quy tắc)
Đáp án đúng là A
Tạm dịch: Bạn càng học chăm chỉ cho những bài thi này, bạn làm bài
càng tốt.
Question 16. John was in Hanoi and wanted to send a parcel to his parents.
He asked a local passer-by the way to the post-office.
- John: "Can you show me the way to the nearest post office,
please?"
- Passer-by: "_______"
A. Look it up in a dictionary! B. Just round the corner over there.
C. There's no traffic near here. D. Not way, sorry.
Kiến thức: Tình huống giao tiếp (Đáp lại lời đề nghị)
Giải thích:
Tình huống giao tiếp:
John đang ở Hà Nội và muốn gửi một bưu kiện cho bố mẹ. Anh hỏi đường đến bưu
điện một người qua đường địa phương.
- John: "Bạn có thể chỉ cho tôi đường đến bưu điện gần nhất được không?"
- Người qua đường: "_______"
Xét các đáp án:
A. Tra từ điển! B. Chỉ quanh góc đằng kia.
C. Gần đây không có giao thông. D. Không được, xin lỗi.
Dựa vào nghĩa, đáp án đúng là B
Question 17. Ann was thanking Kate for coming to dinner.
- Ann: “Thank you for taking the time to come here in
person.”
- Kate: “_______”
A. I’d love to come. What time? B. I haven’t got a clue.
C. It’s my pleasure. D. That’s all for now.
Kiến thức: Tình huống giao tiếp (Đáp lại lời cảm ơn)
Giải thích:
Tình huống giao tiếp:
Ann cảm ơn Kate vì đã đến ăn tối.
- Ann: "Cảm ơn bạn đã dành thời gian đến đây trực tiếp."
- Kate: ‘_______’
Xét các đáp án:
A. Tôi rất muốn đến. Mấy giờ? B. Tôi không có manh mối.
C. Đó là niềm vui của tôi. D. Đó là tất cả cho bây giờ.
Question 18.
A. survival B. industry C. endanger D. commercial
Kiến thức: Trọng âm của từ 3 âm tiết
Giải thích:
A. survival / səˈvaɪvl / (n): Trọng âm rơi vào âm tiết thứ 2 vì từ
gốc là động từ và hậu tố không thay đổi trọng âm chính.
B. industry / ˈɪndəstri / (n): Trọng âm rơi vào âm tiết thứ nhất vì
đuôi –try trọng âm rơi vào âm tiết thứ 3 tính từ cuối.
C. endanger / ɪnˈdeɪndʒ/ (adj): Trọng âm rơi vào âm tiết thứ 2
vì từ gốc là danh từ và tiền tố không thay đổi trọng âm chính.
D. commercial / kəˈmɜːʃl / (adj): Trọng âm rơi vào âm tiết thứ 2
vì đuôi –cial trọng âm rơi vào âm tiết trước nó.
Vậy đáp án đúng là B
Question 19. A. cartoon B. answer C. reason D. paper
Kiến thức: Trọng âm của từ 2 âm tiết
Giải thích:
A. cartoon /kɑːˈtuːn/: Từ này trọng âm rơi vào âm tiết thứ hai. Vì theo
quy tắc các từ tận cùng bằng đuôi -oon trọng âm nhấn ở chính đuôi
này.
B. answer /ˈɑːnsə(r)/: Từ này trọng âm rơi vào âm tiết thứ nhất. Vì
theo quy tắc trọng âm không rơi vào âm /ə/.
C. reason / 'ri:zn /: Từ này trọng âm rơi vào âm tiết thứ nhất. Vì theo
quy tắc trọng âm, thì danh từ 2âm tiết trọng âm thường rơi vào âm
tiết thứ nhất.
D. paper /ˈpeɪpə(r)/: Từ này trọng âm rơi vào âm tiết thứ nhất. Vì
theo quy tắc trọng âm không rơi vào âm /ə/.
=> Phương án A trọng âm rơi vào âm tiết thứ hai, các phương án còn
lại trọng âm rơi vào âm tiết thứ nhất.
Question 20.
A. visited B. played C. hated D. needed
Kiến thức : Cách phát âm đuôi -ED
Giải thích:
– Phát âm là /t/ khi từ có tận cùng bằng các phụ âm vô thanh:
/θ/, /p/, /k/, /f/, /s/, /ʃ/, /tʃ/
– Phát âm là /id/ khi từ có tận cùng là các âm: /t/, /d/
– Phát âm là /d/ khi các từ có tận cùng là nguyên âm và các
phụ âm hữu thanh còn lại
Question 21. A. hand B. save C. take D. face
Kiến thức: Cách phát âm của nguyên âm
Giải thích:
Xét các đáp án:
A. hand /hӕnd/ B. save /seiv/
C. take /teik) D. face /feis/
Question 22. Emissions from factories and exhaust fumes from
vehicles can have detrimental effects on our health.
A. beneficial B. neutral C. needy D. harmful
Kiến thức: Đồng nghĩa (từ đơn)
Giải thích:
Xét các đáp án:
- beneficial (adj): có lợi - neutral (adj): trung tính, trung lập
- needy (adj): nghèo túng, nghèo đói - harmful (adj): có hại
- detrimental (adj): có hại, bất lợi cho = harmful
Vậy đáp án đúng là D
Tạm dịch: Khí thải từ các nhà máy và các phương tiện đi lại có thể tác
động có hại đến sức khỏe của chúng ta.
Question 23. Dozens of valuable works of art disappeared
during shipment to the US.
A. authority B. security C. activity D. delivery
Kiến thức: Đồng nghĩa (từ đơn)
Giải thích:
Xét các đáp án:
A. authority (n.): chính quyềnB. security (n.): an ninh
C. activity (n.): hoạt động D. delivery (n.): sự giao hàng
Ta có: shipment: sự giao hàng
shipment = delivery
Vậy đáp án đúng là D
Tạm dịch: Hàng chục tác phẩm nghệ thuật có giá trị đã biến mất
trong quá trình vận chuyển đến Mỹ.
Question 24. Tom may get into hot water when driving at full
speed after drinking wine.
A. get into trouble B. stay safe
C. fall into disuse D. keep calm
Kiến thức: Trái nghĩa (cụm từ hoặc thành ngữ)
Giải thích:
- get into trouble: gặp rắc rối >< stay safe: giữ an toàn
- get into hot water: gặp rắc rối - stay safe: giữ an toàn
- fall into disuse: bỏ đi, không dùng đến - keep calm: giữ bình
tĩnh
Vậy đáp án đúng là B
Tạm dịch: Tom có thể gặp rắc rối khi lái xe quá tốc độ sau khi uống
rượu.
Question 25. Because of her conservative views, the professor
never accepts anything not related to traditional values and the
status.
A. conservational B. progressive C. modern D. economic
Kiến thức: Trái nghĩa (từ đơn)
Giải thích:
Xét các đáp án:
A. conservational: bảo tồn B. progressive: tiến bộ
C. modern: hiện đại D. economic: kinh tế
Ta có: conservative: bảo thủ, cố chấp
Vậy: conservative >< progressive
Tạm dịch: Vì quan điểm bảo thủ của mình, giáo sư không bao giờ chấp
nhận bất cứ điều gì không liên quan đến các giá trị truyền thống và
địa vị.
Question 26. His wife gave birth to their first child. He understood what true
responsibility meant.
A. Never has he understood true responsibility before he became a parent
himself.
B. Were his first child not to be born, he wouldn’t understand true
responsibility.
C. Hardly had he understood true responsibility when their first child was born.
D. Not until he became a parent did he understand what true responsibility
meant.
Kiến thức: Kết hợp câu – đảo ngữ
Giải thích:
Tạm dịch: Vợ anh ấy sinh đứa con đầu tiên của họ. Anh ta hiểu được trách
nhiệm thực sự là gì.
= D. Mãi cho đến khi vợ anh ta sinh đứa con đầu tiên của họ thì anh ta mới
hiểu được trách nhiệm thực sự là gì.
Cấu trúc: Not until + S + V + O + trợ động từ + S + V (nguyên thể): Mãi cho
đến khi …
Chọn D
Các phương án khác:
A. Sai “has” => “had” (hành động trở thành bố đã ở quá khứ => hành động
trước khi trở thành bố chia ở thì quá khứ hoàn thành)
B. Câu điều kiện loại 2 => sai
Question 27. Peter moved abroad for a fresh start. He regrets it now.
A. Peter wishes he hadn’t moved abroad for a fresh start.
B. If Peter moved abroad for a fresh start, he would regret it.
C. Peter regrets not having moved abroad for a fresh start.
D. If only Peter had moved abroad for a fresh start.
Kiến thức: Kết hợp câu – câu ước
Giải thích:
Dịch đề bài: Peter chuyển ra nước ngoài cho một khởi đầu mới.
Bây giờ anh ấy đang hối hận.
A. Peter ước anh ấy đã không chuyển ra nước ngoài cho một
khởi đầu mới. (Đúng nghĩa, đúng cấu trúc)
B. Nếu Peter chuyển ra nước ngoài cho một khởi đầu mới, anh
ấy sẽ hối hận. (Sai nghĩa)
C. Peter hối tiếc vì đã không chuyển ra nước ngoài cho một khởi
đầu mới. (Sai nghĩa – sự thật là đã chuyển ra nước ngoài)
D. Giá như Peter đã chuyển ra nước ngoài cho một khởi đầu
mới. (Sai nghĩa – như C)
Question 28. The composer Verdi has written the opera Aida
to celebrate the opening of the Suez Canal,
but the opera was not performed until 1871.
A. has written B. to celebrate
C. opening of D. was not performed
Kiến thức: Lỗi sai – Thì của động từ
Giải thích:
Xét mốc thời gian “until 1871” thì hành động đều đã xảy ra vào
thời điểm xác định trong quá khứ. Nên sai ở đáp án A.
Sửa lỗi: has wri en → wrote
Tạm dịch: Nhà soạn nhạc Verdi đã viết vở opera Aida để kỷ niệm
việc mở kênh đào Suez, nhưng vở opera đã không được biểu diễn
cho đến năm 1871.
Question 29. Those of us who have a family history of heart
disease should make yearly appointments with their doctors.
A. who B. should make C. yearly D. their
Kiến thức: Lỗi sai – Đại từ nhân xưng/ tính từ sở hữu
Giải thích:
Những người trong số chúng ta đặt lịch hẹn bác sỹ của chúng ta chứ
không thể là đặt lịch hẹn bác sỹ của họ.
Vậy đáp án đúng là D
Sửa lỗi: their → our
Question 30. Modern office buildings have false floors under
which computer and phone wires can be lain.
A. office buildings B. false floors C. which D. can be lain
Kiến thức: Lỗi sai – Từ vựng
Giải thích:
lay - laid - laid : đặt, xếp thứ gì đó nằm ở vị trí tĩnh
lie – lay - lain: tựa lên, nằm nghỉ trên một vị trí bằng phẳng
Vậy đáp án đúng là D
Sửa: can be lain → can be laid
Question 31. It’s possible that Joanna didn’t receive my message.
A. Joanna shouldn’t have received my message.
B. Joanna needn’t have received my message.
C. Joanna mightn’t have received my message.
D. Joanna can’t have received my message.
Kiến thức: Câu đồng nghĩa – Động từ khuyết thiếu
Giải thích:
Dịch câu gốc: Có thể là Joanna đã không nhận được tin nhắn của tôi
Dịch các phương án.
A. Joanna đáng lẽ không nên nhận được tin nhắn của tôi
B. Joanna đáng lẽ ra không cẩn nhận được tin nhắn của tôi
C. Joanna có lẽ đã không nhận được tin nhắn của tôi.
D. Joanna đã không thể nhận được tin nhắn của tôi
shouldn’t have P 2 : không nên làm gì trong quá khứ (nhưng đã làm)
needn’t have P 2 : không cần làm gì trong quá khứ (nhưng đã làm)
mightn’t have P 2 : có thể có lẽ đã không
can’t have P 2 : không thể nào đã làm
Phù hợp nhất với nghĩa gốc là đáp án C
Question 32. She said, "John, I'll show you round my city
when you're here."
A. She made a trip round her city with John.
B. She planned to show John round her city.
C. She said to John that she would show John round her
city when he was there..
D. She organized a trip round her city for John.
Kiến thức: Câu đồng nghĩa – Câu tường thuật
Giải thích:
Vậy đáp án đúng là C
Đề bài: Cô ấy nói: "John, tớ sẽ dẫn bạn đi thăm thành phố của tớ
trong lúc bạn ở đây = C. Cô ấy hứa sẽ dẫn John đi thăn thành phố
của cô ấy.
Question 33. It is over twenty years since I last got in touch with
them.
A. I can’t help keeping getting in touch with them for over 20
years.
B. I haven’t gotten in touch with them for over 20 years.
C. I used to get in touch with them for over 20 years.
D. I have been getting in touch with them for over 20 years.
Kiến thức: Câu đồng nghĩa – Thì của động từ
Giải thích:
Câu đề bài: Đã hơn hai mươi năm kể từ lần cuối tôi liên lạc với họ.
= B. Tôi đã không liên lạc với họ được hơn 20 năm.
Công thức: S + haven’t/ hasn’t + Vp2 + for + time = It’s + time + since
+ S + (last) Vp1
Question 34. A. much B. many C. every D. a lot
Every day on radio, on TV, and in the newspaper, we hear, see
or read about (34) _______ problems in the world, for example,
pollution problems.
Kiến thức: Đọc điền từ
Giải thích:
Xét các đáp án:
A. much + N-không đếm được B. many + N-số nhiều
C. every + N-số ít D. a lot + (of + N-số nhiều/ không
đếm được)
Question 35. A. released B. launched C. escaped D. freed
Air pollution is the first kind. It mostly comes from fumes (35)
_______ from motorbikes, cars, airplanes, trains and poisonous
gases emitted from factories.
Kiến thức: Đọc điền từ
Giải thích:
Xét các đáp án:
A. release (v): thải ra/ phát hành B. launch (v): phóng ra
C. escape (v): trốn thoát D. free (v): giải phóng
Tạm dịch: Ô nhiễm không khí là loại đầu tiên. Nó chủ yếu đến từ khói
thải ra từ xe máy, ô tô, máy bay, tàu hỏa và khí độc thải ra từ các nhà
máy.
Question 36. A. that B. which C. who D. where
Also, waste is dumped anywhere, even in the city (36) _______
many people are living.
Kiến thức: Đọc điền từ
Giải thích:
Xét các đáp án:
A. that: Thay thế cho danh từ chỉ người và vật có chức năng làm
chủ ngữ và tân ngữ trong mệnh đề quan hệ.
B. which: Thay thế cho danh từ chỉ vật, sự vật hiện tượng có
chức năng làm chủ ngữ và tân ngữ trong mệnh đề quan hệ.
C. who: Thay thế cho danh từ chỉ người có chức năng làm chủ
ngữ và tân ngữ trong mệnh đề quan hệ.
D. where: Thay thế cho danh từ chỉ nơi chốn có chức năng làm
trạng ngữ chỉ nơi chốn trong mệnh đề quan hệ.
Question 37. A. However B. So C. Therefore D. But
. Many people earn their living from fishing in the sea, and the
fish they catch feed many people. (37) _______, the sea has
become so polluted from oil spills and factory wastes that the
fish are dying.
Kiến thức: Đọc điền từ
Giải thích:
Xét các đáp án:
A. However: tuy nhiên B. So: vì vậy
C. Therefore: cho nên D. But: nhưng
Tạm dịch: Nhiều người mưu sinh bằng nghề đánh bắt trên biển, và
những con cá họ bắt được đã nuôi sống nhiều người. Tuy nhiên, biển
đã trở nên ô nhiễm do dầu tràn và chất thải của nhà máy đến nỗi cá
đang chết dần.
Question 38. A. method B. solution C. road D. idea
This problem is growing more difficult every day. We must find a
good (38) _______ that makes the world a better place to live.
Kiến thức: Đọc điền từ
Giải thích:
Xét các đáp án:
A. method: phương pháp B. solution: giải pháp
C. road: con đường D. idea: ý kiến
Tạm dịch: Vấn đề này mỗi ngày một khó hơn. Chúng ta phải tìm ra
một giải pháp tốt để làm cho thế giới trở thành một nơi tốt đẹp hơn
để sống.
Question 39. The best title for this passage could be ________.
A. Vietnam struggling with ageing population
B. Stopping the two-child policy in Vietnam
C. Raising the retirement age in Vietnam
D. How to solve the aging population in Vietnam
Kiến thức : Đọc hiểu
Giải thích: Tiêu đề tốt nhất cho đoạn văn này có thể là
_________.
A. Việt Nam đấu tranh với việc già hóa dân số
B. Chấm dứt chính sách 2 con ở Việt Nam
C. Tăng tuổi nghỉ hưu ở Việt Nam
D. Cách giải quyết vấn đề già hóa dân số ở Việt Nam
Căn cứ vào thông tin toàn bài: “chính sách 2 con; tăng tuổi
nghỉ hưu; cách giải quyết vấn đề già hóa dân số” đều được đề
cập trong bài nhưng chưa bao quát toàn bài. => Đáp án A.
Question 40. Which statement is probably TRUE according to the information in the paragraph
1?
A. In 2017, the elderly take up for one third of the total population, this leads to more
concerns about healthcare, welfare and pensions for the elderly.
B. In 2015, the two-child policy has been officially tightened and succeeded.
C. The government would promote families to have two children to compensate for the
ageing population within the next 20 years.
D. In the next 15-17 years, Vietnam's economy will need a large labor force to integrate
with global economy.
Giải thích: Phát biểu nào có lẽ là ĐÚNG theo thông tin trong đoạn văn 1?
A. Năm 2017, dân số giả ở Việt Nam chiếm tới 1/3 dân số cả nước, điều này dẫn đến
những lo ngại về chăm sóc sức khỏe, phúc lợi xã hội và lương hưu cho người già.
B. Năm 2015, chính sách 2 con đã chính thức được thắt chặt và đã thành công.
C. Chính phủ khuyến khích các gia đình có 2 con để bù đắp cho việc già hóa dân số
trong 20 năm tới.
D. Trong 15-17 năm tới, nền kinh tế Việt Nam sẽ cần một lực lượng lao động lớn để hội
nhập với nền kinh tế toàn cầu.
Thông tin: "in 15-20 years the elderly will account for one third of the total population. This
raises concerns about healthcare, welfare and pensions for the elderly at a time when
Vietnam is focusing on economic integration and requires a large labor force.
Tạm dịch: ...trong 15- 20 năm nữa, người cao tuổi sẽ chiếm một phần ba tổng dân số.
Điều này làm tăng mối lo ngại về chăm sóc sức khỏe, phúc lợi và lương hưu cho người
cao tuổi tại thời điểm Việt Nam đang tập trung vào hội nhập kinh tế và đòi hỏi một lực
lượng lao động lớn.
Question 41. The word “its" in paragraph 1 refers to ________?
A. two-Child policy B. aging population
C. retirement age D. economic integration
Giải thích: Từ “its” trong đoạn 1 thay thế cho từ _________.
A. chính sách 2 con B. già hóa dân số
C. tuổi nghỉ hưu D. hội nhập kinh tế
Thông tin: By ending the two-child policy the government
expects to make up for the ageing population within the next
20 years. But its effect could be creating an uncontrollable
boom in the Vietnamese population.
Thông tin: While the policy is beneficial in utilizing the work
experience of the elderly while creating savings in the pension
budget, it also means fewer job prospects and promotion
opportunities for younger generations.
Question 42. In the 2rd paragraph, the writer suggests that ________.
A. The Ministry of Labor has applied raising the retirement age in May 2017.
B. Raising the retirement age can reduce job opportunities for younger
generations.
C. The elderly whose age is 50 would be dangerous if they continued to work
D. Raising the retirement age and stopping two-child policy can be considered
as long-term and effective solutions.
Kiến thức : Đọc hiểu
Giải thích: Trong đoạn 2, tác giả gợi ý rằng _________.
A. Bộ Lao động đã áp dụng việc nâng tuổi nghỉ hưu từ tháng 5 năm 2017
B. Nâng tuổi nghỉ hưu có thể làm giảm cơ hội việc làm cho thế hệ trẻ.
C. Những người 50 tuổi có thể sẽ gặp nguy hiểm nếu tiếp tục làm việc
D. Nâng tuổi nghỉ hưu và dừng chính sách 2 con được coi là những giải pháp
lâu dài và hiệu quả.
Thông tin: While the policy is beneficial in utilizing the work experience of the
elderly while creating savings in the pension budget, it also means fewer job
prospects and promotion opportunities for younger generations.
Question 43. The word "temporary" in paragraph 2 means _____.
A. constant B. permanent C. short-term D. long-term
Kiến thức : Đọc hiểu
Giải thích: Từ “ temporatory” trong đoạn 2 có nghĩa là ______.
A. consistant (a): kiên trì B. permanent (a): lâu dài
C. short-term (a): ngắn hạn D. long-term (a): dài hạn
Temporatory = short-term
Thông tin: Despite these drawbacks, raising the retirement age
is still considered by policymakers as one of the key solutions
to the ageing population problem in Vietnam. But these are
only temporary solutions.
Tạm dịch: Bất chấp những hạn chế này, việc tăng tuổi nghỉ
hưu vẫn được các nhà hoạch định chính sách coi là một trong
những giải pháp chính cho vấn đề dân số giờ ở Việt Nam.
Nhưng đây chỉ là những giải pháp tạm thời.
Question 44. Which of the following could best reflect the main purpose of the
author in the passage?
A. To prove that coral reefs are animals.
B. To provide the facts about coral reefs.
C. To explain that coral reefs are the most diverse ecosystems in the
ocean.
D. To distinguish coral reefs with other animals.
Kiến thức : Đọc hiểu
Giải thích: Câu nào sau đây thể hiện chính xác nhất mục đích chính của tác
giả trong bài đọc?
A. Nhằm chứng minh rằng dải san hô ngầm là động vật.
B. Để cung cấp các thông tin thật về dải san hô ngầm.
C. Để giải thích rằng dải san hô ngầm là hệ sinh thái đa dạng nhất dưới đại
dương.
D. Để phân biệt dải san hô ngầm và các động vật khác.
Căn cứ vào thông tin toàn bài:
Bài đọc cung cấp cho chúng ta toàn bộ thông tin về loài san hô, từ cấu tạo
(đoạn 2, 3), nơi sinh sống (đoạn 4) và các loại san hô (đoạn cuối).
DE PHAT TRIEN THEO CAU TRUC DE MINH HOA 2022 MON TIENG ANH DE 21 25.pdf
DE PHAT TRIEN THEO CAU TRUC DE MINH HOA 2022 MON TIENG ANH DE 21 25.pdf
DE PHAT TRIEN THEO CAU TRUC DE MINH HOA 2022 MON TIENG ANH DE 21 25.pdf
DE PHAT TRIEN THEO CAU TRUC DE MINH HOA 2022 MON TIENG ANH DE 21 25.pdf
DE PHAT TRIEN THEO CAU TRUC DE MINH HOA 2022 MON TIENG ANH DE 21 25.pdf
DE PHAT TRIEN THEO CAU TRUC DE MINH HOA 2022 MON TIENG ANH DE 21 25.pdf
DE PHAT TRIEN THEO CAU TRUC DE MINH HOA 2022 MON TIENG ANH DE 21 25.pdf
DE PHAT TRIEN THEO CAU TRUC DE MINH HOA 2022 MON TIENG ANH DE 21 25.pdf
DE PHAT TRIEN THEO CAU TRUC DE MINH HOA 2022 MON TIENG ANH DE 21 25.pdf
DE PHAT TRIEN THEO CAU TRUC DE MINH HOA 2022 MON TIENG ANH DE 21 25.pdf
DE PHAT TRIEN THEO CAU TRUC DE MINH HOA 2022 MON TIENG ANH DE 21 25.pdf
DE PHAT TRIEN THEO CAU TRUC DE MINH HOA 2022 MON TIENG ANH DE 21 25.pdf
DE PHAT TRIEN THEO CAU TRUC DE MINH HOA 2022 MON TIENG ANH DE 21 25.pdf
DE PHAT TRIEN THEO CAU TRUC DE MINH HOA 2022 MON TIENG ANH DE 21 25.pdf
DE PHAT TRIEN THEO CAU TRUC DE MINH HOA 2022 MON TIENG ANH DE 21 25.pdf
DE PHAT TRIEN THEO CAU TRUC DE MINH HOA 2022 MON TIENG ANH DE 21 25.pdf
DE PHAT TRIEN THEO CAU TRUC DE MINH HOA 2022 MON TIENG ANH DE 21 25.pdf
DE PHAT TRIEN THEO CAU TRUC DE MINH HOA 2022 MON TIENG ANH DE 21 25.pdf
DE PHAT TRIEN THEO CAU TRUC DE MINH HOA 2022 MON TIENG ANH DE 21 25.pdf
DE PHAT TRIEN THEO CAU TRUC DE MINH HOA 2022 MON TIENG ANH DE 21 25.pdf
DE PHAT TRIEN THEO CAU TRUC DE MINH HOA 2022 MON TIENG ANH DE 21 25.pdf
DE PHAT TRIEN THEO CAU TRUC DE MINH HOA 2022 MON TIENG ANH DE 21 25.pdf
DE PHAT TRIEN THEO CAU TRUC DE MINH HOA 2022 MON TIENG ANH DE 21 25.pdf
DE PHAT TRIEN THEO CAU TRUC DE MINH HOA 2022 MON TIENG ANH DE 21 25.pdf
DE PHAT TRIEN THEO CAU TRUC DE MINH HOA 2022 MON TIENG ANH DE 21 25.pdf
DE PHAT TRIEN THEO CAU TRUC DE MINH HOA 2022 MON TIENG ANH DE 21 25.pdf
DE PHAT TRIEN THEO CAU TRUC DE MINH HOA 2022 MON TIENG ANH DE 21 25.pdf
DE PHAT TRIEN THEO CAU TRUC DE MINH HOA 2022 MON TIENG ANH DE 21 25.pdf
DE PHAT TRIEN THEO CAU TRUC DE MINH HOA 2022 MON TIENG ANH DE 21 25.pdf
DE PHAT TRIEN THEO CAU TRUC DE MINH HOA 2022 MON TIENG ANH DE 21 25.pdf
DE PHAT TRIEN THEO CAU TRUC DE MINH HOA 2022 MON TIENG ANH DE 21 25.pdf
DE PHAT TRIEN THEO CAU TRUC DE MINH HOA 2022 MON TIENG ANH DE 21 25.pdf
DE PHAT TRIEN THEO CAU TRUC DE MINH HOA 2022 MON TIENG ANH DE 21 25.pdf
DE PHAT TRIEN THEO CAU TRUC DE MINH HOA 2022 MON TIENG ANH DE 21 25.pdf
DE PHAT TRIEN THEO CAU TRUC DE MINH HOA 2022 MON TIENG ANH DE 21 25.pdf
DE PHAT TRIEN THEO CAU TRUC DE MINH HOA 2022 MON TIENG ANH DE 21 25.pdf
DE PHAT TRIEN THEO CAU TRUC DE MINH HOA 2022 MON TIENG ANH DE 21 25.pdf
DE PHAT TRIEN THEO CAU TRUC DE MINH HOA 2022 MON TIENG ANH DE 21 25.pdf
DE PHAT TRIEN THEO CAU TRUC DE MINH HOA 2022 MON TIENG ANH DE 21 25.pdf
DE PHAT TRIEN THEO CAU TRUC DE MINH HOA 2022 MON TIENG ANH DE 21 25.pdf
DE PHAT TRIEN THEO CAU TRUC DE MINH HOA 2022 MON TIENG ANH DE 21 25.pdf
DE PHAT TRIEN THEO CAU TRUC DE MINH HOA 2022 MON TIENG ANH DE 21 25.pdf
DE PHAT TRIEN THEO CAU TRUC DE MINH HOA 2022 MON TIENG ANH DE 21 25.pdf
DE PHAT TRIEN THEO CAU TRUC DE MINH HOA 2022 MON TIENG ANH DE 21 25.pdf
DE PHAT TRIEN THEO CAU TRUC DE MINH HOA 2022 MON TIENG ANH DE 21 25.pdf
DE PHAT TRIEN THEO CAU TRUC DE MINH HOA 2022 MON TIENG ANH DE 21 25.pdf
DE PHAT TRIEN THEO CAU TRUC DE MINH HOA 2022 MON TIENG ANH DE 21 25.pdf
DE PHAT TRIEN THEO CAU TRUC DE MINH HOA 2022 MON TIENG ANH DE 21 25.pdf
DE PHAT TRIEN THEO CAU TRUC DE MINH HOA 2022 MON TIENG ANH DE 21 25.pdf
DE PHAT TRIEN THEO CAU TRUC DE MINH HOA 2022 MON TIENG ANH DE 21 25.pdf
DE PHAT TRIEN THEO CAU TRUC DE MINH HOA 2022 MON TIENG ANH DE 21 25.pdf
DE PHAT TRIEN THEO CAU TRUC DE MINH HOA 2022 MON TIENG ANH DE 21 25.pdf
DE PHAT TRIEN THEO CAU TRUC DE MINH HOA 2022 MON TIENG ANH DE 21 25.pdf
DE PHAT TRIEN THEO CAU TRUC DE MINH HOA 2022 MON TIENG ANH DE 21 25.pdf
DE PHAT TRIEN THEO CAU TRUC DE MINH HOA 2022 MON TIENG ANH DE 21 25.pdf
DE PHAT TRIEN THEO CAU TRUC DE MINH HOA 2022 MON TIENG ANH DE 21 25.pdf
DE PHAT TRIEN THEO CAU TRUC DE MINH HOA 2022 MON TIENG ANH DE 21 25.pdf
DE PHAT TRIEN THEO CAU TRUC DE MINH HOA 2022 MON TIENG ANH DE 21 25.pdf
DE PHAT TRIEN THEO CAU TRUC DE MINH HOA 2022 MON TIENG ANH DE 21 25.pdf
DE PHAT TRIEN THEO CAU TRUC DE MINH HOA 2022 MON TIENG ANH DE 21 25.pdf
DE PHAT TRIEN THEO CAU TRUC DE MINH HOA 2022 MON TIENG ANH DE 21 25.pdf
DE PHAT TRIEN THEO CAU TRUC DE MINH HOA 2022 MON TIENG ANH DE 21 25.pdf
DE PHAT TRIEN THEO CAU TRUC DE MINH HOA 2022 MON TIENG ANH DE 21 25.pdf
DE PHAT TRIEN THEO CAU TRUC DE MINH HOA 2022 MON TIENG ANH DE 21 25.pdf
DE PHAT TRIEN THEO CAU TRUC DE MINH HOA 2022 MON TIENG ANH DE 21 25.pdf
DE PHAT TRIEN THEO CAU TRUC DE MINH HOA 2022 MON TIENG ANH DE 21 25.pdf
DE PHAT TRIEN THEO CAU TRUC DE MINH HOA 2022 MON TIENG ANH DE 21 25.pdf
DE PHAT TRIEN THEO CAU TRUC DE MINH HOA 2022 MON TIENG ANH DE 21 25.pdf
DE PHAT TRIEN THEO CAU TRUC DE MINH HOA 2022 MON TIENG ANH DE 21 25.pdf
DE PHAT TRIEN THEO CAU TRUC DE MINH HOA 2022 MON TIENG ANH DE 21 25.pdf
DE PHAT TRIEN THEO CAU TRUC DE MINH HOA 2022 MON TIENG ANH DE 21 25.pdf
DE PHAT TRIEN THEO CAU TRUC DE MINH HOA 2022 MON TIENG ANH DE 21 25.pdf
DE PHAT TRIEN THEO CAU TRUC DE MINH HOA 2022 MON TIENG ANH DE 21 25.pdf
DE PHAT TRIEN THEO CAU TRUC DE MINH HOA 2022 MON TIENG ANH DE 21 25.pdf
DE PHAT TRIEN THEO CAU TRUC DE MINH HOA 2022 MON TIENG ANH DE 21 25.pdf
DE PHAT TRIEN THEO CAU TRUC DE MINH HOA 2022 MON TIENG ANH DE 21 25.pdf
DE PHAT TRIEN THEO CAU TRUC DE MINH HOA 2022 MON TIENG ANH DE 21 25.pdf
DE PHAT TRIEN THEO CAU TRUC DE MINH HOA 2022 MON TIENG ANH DE 21 25.pdf
DE PHAT TRIEN THEO CAU TRUC DE MINH HOA 2022 MON TIENG ANH DE 21 25.pdf
DE PHAT TRIEN THEO CAU TRUC DE MINH HOA 2022 MON TIENG ANH DE 21 25.pdf
DE PHAT TRIEN THEO CAU TRUC DE MINH HOA 2022 MON TIENG ANH DE 21 25.pdf
DE PHAT TRIEN THEO CAU TRUC DE MINH HOA 2022 MON TIENG ANH DE 21 25.pdf
DE PHAT TRIEN THEO CAU TRUC DE MINH HOA 2022 MON TIENG ANH DE 21 25.pdf
DE PHAT TRIEN THEO CAU TRUC DE MINH HOA 2022 MON TIENG ANH DE 21 25.pdf
DE PHAT TRIEN THEO CAU TRUC DE MINH HOA 2022 MON TIENG ANH DE 21 25.pdf
DE PHAT TRIEN THEO CAU TRUC DE MINH HOA 2022 MON TIENG ANH DE 21 25.pdf
DE PHAT TRIEN THEO CAU TRUC DE MINH HOA 2022 MON TIENG ANH DE 21 25.pdf
DE PHAT TRIEN THEO CAU TRUC DE MINH HOA 2022 MON TIENG ANH DE 21 25.pdf
DE PHAT TRIEN THEO CAU TRUC DE MINH HOA 2022 MON TIENG ANH DE 21 25.pdf
DE PHAT TRIEN THEO CAU TRUC DE MINH HOA 2022 MON TIENG ANH DE 21 25.pdf
DE PHAT TRIEN THEO CAU TRUC DE MINH HOA 2022 MON TIENG ANH DE 21 25.pdf
DE PHAT TRIEN THEO CAU TRUC DE MINH HOA 2022 MON TIENG ANH DE 21 25.pdf
DE PHAT TRIEN THEO CAU TRUC DE MINH HOA 2022 MON TIENG ANH DE 21 25.pdf
DE PHAT TRIEN THEO CAU TRUC DE MINH HOA 2022 MON TIENG ANH DE 21 25.pdf
DE PHAT TRIEN THEO CAU TRUC DE MINH HOA 2022 MON TIENG ANH DE 21 25.pdf
DE PHAT TRIEN THEO CAU TRUC DE MINH HOA 2022 MON TIENG ANH DE 21 25.pdf
DE PHAT TRIEN THEO CAU TRUC DE MINH HOA 2022 MON TIENG ANH DE 21 25.pdf
DE PHAT TRIEN THEO CAU TRUC DE MINH HOA 2022 MON TIENG ANH DE 21 25.pdf
DE PHAT TRIEN THEO CAU TRUC DE MINH HOA 2022 MON TIENG ANH DE 21 25.pdf
DE PHAT TRIEN THEO CAU TRUC DE MINH HOA 2022 MON TIENG ANH DE 21 25.pdf
DE PHAT TRIEN THEO CAU TRUC DE MINH HOA 2022 MON TIENG ANH DE 21 25.pdf
DE PHAT TRIEN THEO CAU TRUC DE MINH HOA 2022 MON TIENG ANH DE 21 25.pdf
DE PHAT TRIEN THEO CAU TRUC DE MINH HOA 2022 MON TIENG ANH DE 21 25.pdf
DE PHAT TRIEN THEO CAU TRUC DE MINH HOA 2022 MON TIENG ANH DE 21 25.pdf
DE PHAT TRIEN THEO CAU TRUC DE MINH HOA 2022 MON TIENG ANH DE 21 25.pdf
DE PHAT TRIEN THEO CAU TRUC DE MINH HOA 2022 MON TIENG ANH DE 21 25.pdf
DE PHAT TRIEN THEO CAU TRUC DE MINH HOA 2022 MON TIENG ANH DE 21 25.pdf

More Related Content

What's hot

Starter Toeic
Starter ToeicStarter Toeic
Starter ToeicVõ Phúc
 
Đề thi thử và đáp án chi tiết môn Tiếng Anh số 4 - Megabook.vn
Đề thi thử và đáp án chi tiết môn Tiếng Anh số 4 - Megabook.vnĐề thi thử và đáp án chi tiết môn Tiếng Anh số 4 - Megabook.vn
Đề thi thử và đáp án chi tiết môn Tiếng Anh số 4 - Megabook.vnMegabook
 
Dịch nghĩa và giải thích chi tiết starter toeic unit 2
Dịch nghĩa và giải thích chi tiết starter toeic unit 2Dịch nghĩa và giải thích chi tiết starter toeic unit 2
Dịch nghĩa và giải thích chi tiết starter toeic unit 2my nguyễn
 
Đề Tiếng Anh 12 cơ bản unit 8 có đáp án - VipLam.Net
Đề Tiếng Anh 12 cơ bản unit 8 có đáp án - VipLam.NetĐề Tiếng Anh 12 cơ bản unit 8 có đáp án - VipLam.Net
Đề Tiếng Anh 12 cơ bản unit 8 có đáp án - VipLam.NetThùy Linh
 
19 enterovirus rotavirus - da
19 enterovirus   rotavirus - da19 enterovirus   rotavirus - da
19 enterovirus rotavirus - daLe Tran Anh
 
Dịch nghĩa và giải thích chi tiết starter toeic unit 4
Dịch nghĩa và giải thích chi tiết starter toeic unit 4Dịch nghĩa và giải thích chi tiết starter toeic unit 4
Dịch nghĩa và giải thích chi tiết starter toeic unit 4my nguyễn
 
Dịch nghĩa và giải thích chi tiết starter toeic unit 3
Dịch nghĩa và giải thích chi tiết starter toeic unit 3Dịch nghĩa và giải thích chi tiết starter toeic unit 3
Dịch nghĩa và giải thích chi tiết starter toeic unit 3my nguyễn
 
40 ĐỀ THI THỬ TỐT NGHIỆP THPT - MÔN TIẾNG ANH - NĂM 2023 - SOẠN CHUẨN CẤU TR...
40 ĐỀ THI THỬ TỐT NGHIỆP THPT - MÔN TIẾNG ANH - NĂM 2023 - SOẠN CHUẨN CẤU TR...40 ĐỀ THI THỬ TỐT NGHIỆP THPT - MÔN TIẾNG ANH - NĂM 2023 - SOẠN CHUẨN CẤU TR...
40 ĐỀ THI THỬ TỐT NGHIỆP THPT - MÔN TIẾNG ANH - NĂM 2023 - SOẠN CHUẨN CẤU TR...Nguyen Thanh Tu Collection
 
De 3-ta-ts-hien
De 3-ta-ts-hienDe 3-ta-ts-hien
De 3-ta-ts-hienHuong Van
 
Cách dùng modal perfect và bài tập thực hành
Cách dùng modal perfect và bài tập thực hànhCách dùng modal perfect và bài tập thực hành
Cách dùng modal perfect và bài tập thực hànhthuonglindo
 
Đề thi thử và đáp án chi tiết môn Tiếng Anh số 3 - Megabook.vn
Đề thi thử và đáp án chi tiết môn Tiếng Anh số 3 - Megabook.vnĐề thi thử và đáp án chi tiết môn Tiếng Anh số 3 - Megabook.vn
Đề thi thử và đáp án chi tiết môn Tiếng Anh số 3 - Megabook.vnMegabook
 
50 ĐỀ PHÁT TRIỂN THEO CẤU TRÚC ĐỀ MINH HỌA BGD NGÀY 22-3-2024 KỲ THI TỐT NGHI...
50 ĐỀ PHÁT TRIỂN THEO CẤU TRÚC ĐỀ MINH HỌA BGD NGÀY 22-3-2024 KỲ THI TỐT NGHI...50 ĐỀ PHÁT TRIỂN THEO CẤU TRÚC ĐỀ MINH HỌA BGD NGÀY 22-3-2024 KỲ THI TỐT NGHI...
50 ĐỀ PHÁT TRIỂN THEO CẤU TRÚC ĐỀ MINH HỌA BGD NGÀY 22-3-2024 KỲ THI TỐT NGHI...Nguyen Thanh Tu Collection
 
Đề Tiếng Anh 12 cơ bản unit 2 có đáp án - VipLam.Net
Đề Tiếng Anh 12 cơ bản unit 2 có đáp án - VipLam.NetĐề Tiếng Anh 12 cơ bản unit 2 có đáp án - VipLam.Net
Đề Tiếng Anh 12 cơ bản unit 2 có đáp án - VipLam.NetThùy Linh
 
Đề Tiếng Anh 12 cơ bản unit 12 có đáp án - VipLam.Net
Đề Tiếng Anh 12 cơ bản unit 12 có đáp án - VipLam.NetĐề Tiếng Anh 12 cơ bản unit 12 có đáp án - VipLam.Net
Đề Tiếng Anh 12 cơ bản unit 12 có đáp án - VipLam.NetThùy Linh
 
TỔNG ÔN 32 CHUYÊN ĐỀ NGỮ PHÁP TIẾNG ANH 9+ (ÔN THI HSG, VÀO 10, TỐT NGHIỆP TH...
TỔNG ÔN 32 CHUYÊN ĐỀ NGỮ PHÁP TIẾNG ANH 9+ (ÔN THI HSG, VÀO 10, TỐT NGHIỆP TH...TỔNG ÔN 32 CHUYÊN ĐỀ NGỮ PHÁP TIẾNG ANH 9+ (ÔN THI HSG, VÀO 10, TỐT NGHIỆP TH...
TỔNG ÔN 32 CHUYÊN ĐỀ NGỮ PHÁP TIẾNG ANH 9+ (ÔN THI HSG, VÀO 10, TỐT NGHIỆP TH...Nguyen Thanh Tu Collection
 
Bai giai chi tiet mon anh cao dang khoi a1 d 2013
Bai giai chi tiet mon anh cao dang khoi a1 d 2013Bai giai chi tiet mon anh cao dang khoi a1 d 2013
Bai giai chi tiet mon anh cao dang khoi a1 d 2013Tommy Bảo
 
Đề Tiếng Anh 12 cơ bản unit 6 có đáp án - VipLam.Net
Đề Tiếng Anh 12 cơ bản unit 6 có đáp án - VipLam.NetĐề Tiếng Anh 12 cơ bản unit 6 có đáp án - VipLam.Net
Đề Tiếng Anh 12 cơ bản unit 6 có đáp án - VipLam.NetThùy Linh
 
Lý thuyết câu bị động
Lý thuyết câu bị độngLý thuyết câu bị động
Lý thuyết câu bị độngTươi Sama
 
Bo de thi lop 10 chuyen anh co dap an
Bo de thi lop 10 chuyen anh co dap anBo de thi lop 10 chuyen anh co dap an
Bo de thi lop 10 chuyen anh co dap anTommy Bảo
 
Đề Tiếng Anh 12 cơ bản unit 3 có đáp án - VipLam.Net
Đề Tiếng Anh 12 cơ bản unit 3 có đáp án - VipLam.NetĐề Tiếng Anh 12 cơ bản unit 3 có đáp án - VipLam.Net
Đề Tiếng Anh 12 cơ bản unit 3 có đáp án - VipLam.NetThùy Linh
 

What's hot (20)

Starter Toeic
Starter ToeicStarter Toeic
Starter Toeic
 
Đề thi thử và đáp án chi tiết môn Tiếng Anh số 4 - Megabook.vn
Đề thi thử và đáp án chi tiết môn Tiếng Anh số 4 - Megabook.vnĐề thi thử và đáp án chi tiết môn Tiếng Anh số 4 - Megabook.vn
Đề thi thử và đáp án chi tiết môn Tiếng Anh số 4 - Megabook.vn
 
Dịch nghĩa và giải thích chi tiết starter toeic unit 2
Dịch nghĩa và giải thích chi tiết starter toeic unit 2Dịch nghĩa và giải thích chi tiết starter toeic unit 2
Dịch nghĩa và giải thích chi tiết starter toeic unit 2
 
Đề Tiếng Anh 12 cơ bản unit 8 có đáp án - VipLam.Net
Đề Tiếng Anh 12 cơ bản unit 8 có đáp án - VipLam.NetĐề Tiếng Anh 12 cơ bản unit 8 có đáp án - VipLam.Net
Đề Tiếng Anh 12 cơ bản unit 8 có đáp án - VipLam.Net
 
19 enterovirus rotavirus - da
19 enterovirus   rotavirus - da19 enterovirus   rotavirus - da
19 enterovirus rotavirus - da
 
Dịch nghĩa và giải thích chi tiết starter toeic unit 4
Dịch nghĩa và giải thích chi tiết starter toeic unit 4Dịch nghĩa và giải thích chi tiết starter toeic unit 4
Dịch nghĩa và giải thích chi tiết starter toeic unit 4
 
Dịch nghĩa và giải thích chi tiết starter toeic unit 3
Dịch nghĩa và giải thích chi tiết starter toeic unit 3Dịch nghĩa và giải thích chi tiết starter toeic unit 3
Dịch nghĩa và giải thích chi tiết starter toeic unit 3
 
40 ĐỀ THI THỬ TỐT NGHIỆP THPT - MÔN TIẾNG ANH - NĂM 2023 - SOẠN CHUẨN CẤU TR...
40 ĐỀ THI THỬ TỐT NGHIỆP THPT - MÔN TIẾNG ANH - NĂM 2023 - SOẠN CHUẨN CẤU TR...40 ĐỀ THI THỬ TỐT NGHIỆP THPT - MÔN TIẾNG ANH - NĂM 2023 - SOẠN CHUẨN CẤU TR...
40 ĐỀ THI THỬ TỐT NGHIỆP THPT - MÔN TIẾNG ANH - NĂM 2023 - SOẠN CHUẨN CẤU TR...
 
De 3-ta-ts-hien
De 3-ta-ts-hienDe 3-ta-ts-hien
De 3-ta-ts-hien
 
Cách dùng modal perfect và bài tập thực hành
Cách dùng modal perfect và bài tập thực hànhCách dùng modal perfect và bài tập thực hành
Cách dùng modal perfect và bài tập thực hành
 
Đề thi thử và đáp án chi tiết môn Tiếng Anh số 3 - Megabook.vn
Đề thi thử và đáp án chi tiết môn Tiếng Anh số 3 - Megabook.vnĐề thi thử và đáp án chi tiết môn Tiếng Anh số 3 - Megabook.vn
Đề thi thử và đáp án chi tiết môn Tiếng Anh số 3 - Megabook.vn
 
50 ĐỀ PHÁT TRIỂN THEO CẤU TRÚC ĐỀ MINH HỌA BGD NGÀY 22-3-2024 KỲ THI TỐT NGHI...
50 ĐỀ PHÁT TRIỂN THEO CẤU TRÚC ĐỀ MINH HỌA BGD NGÀY 22-3-2024 KỲ THI TỐT NGHI...50 ĐỀ PHÁT TRIỂN THEO CẤU TRÚC ĐỀ MINH HỌA BGD NGÀY 22-3-2024 KỲ THI TỐT NGHI...
50 ĐỀ PHÁT TRIỂN THEO CẤU TRÚC ĐỀ MINH HỌA BGD NGÀY 22-3-2024 KỲ THI TỐT NGHI...
 
Đề Tiếng Anh 12 cơ bản unit 2 có đáp án - VipLam.Net
Đề Tiếng Anh 12 cơ bản unit 2 có đáp án - VipLam.NetĐề Tiếng Anh 12 cơ bản unit 2 có đáp án - VipLam.Net
Đề Tiếng Anh 12 cơ bản unit 2 có đáp án - VipLam.Net
 
Đề Tiếng Anh 12 cơ bản unit 12 có đáp án - VipLam.Net
Đề Tiếng Anh 12 cơ bản unit 12 có đáp án - VipLam.NetĐề Tiếng Anh 12 cơ bản unit 12 có đáp án - VipLam.Net
Đề Tiếng Anh 12 cơ bản unit 12 có đáp án - VipLam.Net
 
TỔNG ÔN 32 CHUYÊN ĐỀ NGỮ PHÁP TIẾNG ANH 9+ (ÔN THI HSG, VÀO 10, TỐT NGHIỆP TH...
TỔNG ÔN 32 CHUYÊN ĐỀ NGỮ PHÁP TIẾNG ANH 9+ (ÔN THI HSG, VÀO 10, TỐT NGHIỆP TH...TỔNG ÔN 32 CHUYÊN ĐỀ NGỮ PHÁP TIẾNG ANH 9+ (ÔN THI HSG, VÀO 10, TỐT NGHIỆP TH...
TỔNG ÔN 32 CHUYÊN ĐỀ NGỮ PHÁP TIẾNG ANH 9+ (ÔN THI HSG, VÀO 10, TỐT NGHIỆP TH...
 
Bai giai chi tiet mon anh cao dang khoi a1 d 2013
Bai giai chi tiet mon anh cao dang khoi a1 d 2013Bai giai chi tiet mon anh cao dang khoi a1 d 2013
Bai giai chi tiet mon anh cao dang khoi a1 d 2013
 
Đề Tiếng Anh 12 cơ bản unit 6 có đáp án - VipLam.Net
Đề Tiếng Anh 12 cơ bản unit 6 có đáp án - VipLam.NetĐề Tiếng Anh 12 cơ bản unit 6 có đáp án - VipLam.Net
Đề Tiếng Anh 12 cơ bản unit 6 có đáp án - VipLam.Net
 
Lý thuyết câu bị động
Lý thuyết câu bị độngLý thuyết câu bị động
Lý thuyết câu bị động
 
Bo de thi lop 10 chuyen anh co dap an
Bo de thi lop 10 chuyen anh co dap anBo de thi lop 10 chuyen anh co dap an
Bo de thi lop 10 chuyen anh co dap an
 
Đề Tiếng Anh 12 cơ bản unit 3 có đáp án - VipLam.Net
Đề Tiếng Anh 12 cơ bản unit 3 có đáp án - VipLam.NetĐề Tiếng Anh 12 cơ bản unit 3 có đáp án - VipLam.Net
Đề Tiếng Anh 12 cơ bản unit 3 có đáp án - VipLam.Net
 

Similar to DE PHAT TRIEN THEO CAU TRUC DE MINH HOA 2022 MON TIENG ANH DE 21 25.pdf

đáP án và giải thích đề 25
đáP án và giải thích đề 25đáP án và giải thích đề 25
đáP án và giải thích đề 25Huynh ICT
 
đáP án và giải thích đề 20
đáP án và giải thích đề 20đáP án và giải thích đề 20
đáP án và giải thích đề 20Huynh ICT
 
đáP án và giải thích đề 24
đáP án và giải thích đề 24đáP án và giải thích đề 24
đáP án và giải thích đề 24Huynh ICT
 
đáP án và giải thích đề 23
đáP án và giải thích đề 23đáP án và giải thích đề 23
đáP án và giải thích đề 23Huynh ICT
 
Đề thi thử Ôn thi Tiếng Anh vào Cao Đẳng , Đại học năm 2013 - Đề 110
Đề thi thử  Ôn thi Tiếng Anh vào Cao Đẳng , Đại học năm 2013 - Đề 110Đề thi thử  Ôn thi Tiếng Anh vào Cao Đẳng , Đại học năm 2013 - Đề 110
Đề thi thử Ôn thi Tiếng Anh vào Cao Đẳng , Đại học năm 2013 - Đề 110phamnhakb
 
đáP án và giải thích đề 4
đáP án và giải thích đề 4đáP án và giải thích đề 4
đáP án và giải thích đề 4Huynh ICT
 
đáP án và giải thích đề 15
đáP án và giải thích đề 15đáP án và giải thích đề 15
đáP án và giải thích đề 15Huynh ICT
 
đáP án và giải thích đề 9
đáP án và giải thích đề 9đáP án và giải thích đề 9
đáP án và giải thích đề 9Huynh ICT
 
đáP án và giải thích đề 17
đáP án và giải thích đề 17đáP án và giải thích đề 17
đáP án và giải thích đề 17Huynh ICT
 
đáP án và giải thích đề 31
đáP án và giải thích đề 31đáP án và giải thích đề 31
đáP án và giải thích đề 31Huynh ICT
 
đáP án và giải thích đề 26
đáP án và giải thích đề 26đáP án và giải thích đề 26
đáP án và giải thích đề 26Huynh ICT
 
đáP án và giải thích đề 16
đáP án và giải thích đề 16đáP án và giải thích đề 16
đáP án và giải thích đề 16Huynh ICT
 
đáP án và giải thích đề 14
đáP án và giải thích đề 14đáP án và giải thích đề 14
đáP án và giải thích đề 14Huynh ICT
 
đáP án và giải thích đề 19
đáP án và giải thích đề 19đáP án và giải thích đề 19
đáP án và giải thích đề 19Huynh ICT
 
đáP án và giải thích đề 32
đáP án và giải thích đề 32đáP án và giải thích đề 32
đáP án và giải thích đề 32Huynh ICT
 
đáP án và giải thích đề 8
đáP án và giải thích đề 8đáP án và giải thích đề 8
đáP án và giải thích đề 8Huynh ICT
 
đáP án và giải thích đề 6
đáP án và giải thích đề 6đáP án và giải thích đề 6
đáP án và giải thích đề 6Huynh ICT
 
đáP án và giải thích đề 7
đáP án và giải thích đề 7đáP án và giải thích đề 7
đáP án và giải thích đề 7Huynh ICT
 
đáP án và giải thích đề 13
đáP án và giải thích đề 13đáP án và giải thích đề 13
đáP án và giải thích đề 13Huynh ICT
 
đáP án và giải thích đề 5
đáP án và giải thích đề 5đáP án và giải thích đề 5
đáP án và giải thích đề 5Huynh ICT
 

Similar to DE PHAT TRIEN THEO CAU TRUC DE MINH HOA 2022 MON TIENG ANH DE 21 25.pdf (20)

đáP án và giải thích đề 25
đáP án và giải thích đề 25đáP án và giải thích đề 25
đáP án và giải thích đề 25
 
đáP án và giải thích đề 20
đáP án và giải thích đề 20đáP án và giải thích đề 20
đáP án và giải thích đề 20
 
đáP án và giải thích đề 24
đáP án và giải thích đề 24đáP án và giải thích đề 24
đáP án và giải thích đề 24
 
đáP án và giải thích đề 23
đáP án và giải thích đề 23đáP án và giải thích đề 23
đáP án và giải thích đề 23
 
Đề thi thử Ôn thi Tiếng Anh vào Cao Đẳng , Đại học năm 2013 - Đề 110
Đề thi thử  Ôn thi Tiếng Anh vào Cao Đẳng , Đại học năm 2013 - Đề 110Đề thi thử  Ôn thi Tiếng Anh vào Cao Đẳng , Đại học năm 2013 - Đề 110
Đề thi thử Ôn thi Tiếng Anh vào Cao Đẳng , Đại học năm 2013 - Đề 110
 
đáP án và giải thích đề 4
đáP án và giải thích đề 4đáP án và giải thích đề 4
đáP án và giải thích đề 4
 
đáP án và giải thích đề 15
đáP án và giải thích đề 15đáP án và giải thích đề 15
đáP án và giải thích đề 15
 
đáP án và giải thích đề 9
đáP án và giải thích đề 9đáP án và giải thích đề 9
đáP án và giải thích đề 9
 
đáP án và giải thích đề 17
đáP án và giải thích đề 17đáP án và giải thích đề 17
đáP án và giải thích đề 17
 
đáP án và giải thích đề 31
đáP án và giải thích đề 31đáP án và giải thích đề 31
đáP án và giải thích đề 31
 
đáP án và giải thích đề 26
đáP án và giải thích đề 26đáP án và giải thích đề 26
đáP án và giải thích đề 26
 
đáP án và giải thích đề 16
đáP án và giải thích đề 16đáP án và giải thích đề 16
đáP án và giải thích đề 16
 
đáP án và giải thích đề 14
đáP án và giải thích đề 14đáP án và giải thích đề 14
đáP án và giải thích đề 14
 
đáP án và giải thích đề 19
đáP án và giải thích đề 19đáP án và giải thích đề 19
đáP án và giải thích đề 19
 
đáP án và giải thích đề 32
đáP án và giải thích đề 32đáP án và giải thích đề 32
đáP án và giải thích đề 32
 
đáP án và giải thích đề 8
đáP án và giải thích đề 8đáP án và giải thích đề 8
đáP án và giải thích đề 8
 
đáP án và giải thích đề 6
đáP án và giải thích đề 6đáP án và giải thích đề 6
đáP án và giải thích đề 6
 
đáP án và giải thích đề 7
đáP án và giải thích đề 7đáP án và giải thích đề 7
đáP án và giải thích đề 7
 
đáP án và giải thích đề 13
đáP án và giải thích đề 13đáP án và giải thích đề 13
đáP án và giải thích đề 13
 
đáP án và giải thích đề 5
đáP án và giải thích đề 5đáP án và giải thích đề 5
đáP án và giải thích đề 5
 

More from Nguyen Thanh Tu Collection

GIÁO ÁN DẠY THÊM (KẾ HOẠCH BÀI DẠY BUỔI 2) - TIẾNG ANH 6, 7 GLOBAL SUCCESS (2...
GIÁO ÁN DẠY THÊM (KẾ HOẠCH BÀI DẠY BUỔI 2) - TIẾNG ANH 6, 7 GLOBAL SUCCESS (2...GIÁO ÁN DẠY THÊM (KẾ HOẠCH BÀI DẠY BUỔI 2) - TIẾNG ANH 6, 7 GLOBAL SUCCESS (2...
GIÁO ÁN DẠY THÊM (KẾ HOẠCH BÀI DẠY BUỔI 2) - TIẾNG ANH 6, 7 GLOBAL SUCCESS (2...Nguyen Thanh Tu Collection
 
TÀI LIỆU BỒI DƯỠNG HỌC SINH GIỎI KỸ NĂNG VIẾT ĐOẠN VĂN NGHỊ LUẬN XÃ HỘI 200 C...
TÀI LIỆU BỒI DƯỠNG HỌC SINH GIỎI KỸ NĂNG VIẾT ĐOẠN VĂN NGHỊ LUẬN XÃ HỘI 200 C...TÀI LIỆU BỒI DƯỠNG HỌC SINH GIỎI KỸ NĂNG VIẾT ĐOẠN VĂN NGHỊ LUẬN XÃ HỘI 200 C...
TÀI LIỆU BỒI DƯỠNG HỌC SINH GIỎI KỸ NĂNG VIẾT ĐOẠN VĂN NGHỊ LUẬN XÃ HỘI 200 C...Nguyen Thanh Tu Collection
 
BỘ LUYỆN NGHE VÀO 10 TIẾNG ANH DẠNG TRẮC NGHIỆM 4 CÂU TRẢ LỜI - CÓ FILE NGHE.pdf
BỘ LUYỆN NGHE VÀO 10 TIẾNG ANH DẠNG TRẮC NGHIỆM 4 CÂU TRẢ LỜI - CÓ FILE NGHE.pdfBỘ LUYỆN NGHE VÀO 10 TIẾNG ANH DẠNG TRẮC NGHIỆM 4 CÂU TRẢ LỜI - CÓ FILE NGHE.pdf
BỘ LUYỆN NGHE VÀO 10 TIẾNG ANH DẠNG TRẮC NGHIỆM 4 CÂU TRẢ LỜI - CÓ FILE NGHE.pdfNguyen Thanh Tu Collection
 
ĐỀ CHÍNH THỨC KỲ THI TUYỂN SINH VÀO LỚP 10 THPT CÁC TỈNH THÀNH NĂM HỌC 2020 –...
ĐỀ CHÍNH THỨC KỲ THI TUYỂN SINH VÀO LỚP 10 THPT CÁC TỈNH THÀNH NĂM HỌC 2020 –...ĐỀ CHÍNH THỨC KỲ THI TUYỂN SINH VÀO LỚP 10 THPT CÁC TỈNH THÀNH NĂM HỌC 2020 –...
ĐỀ CHÍNH THỨC KỲ THI TUYỂN SINH VÀO LỚP 10 THPT CÁC TỈNH THÀNH NĂM HỌC 2020 –...Nguyen Thanh Tu Collection
 
TÀI LIỆU BỒI DƯỠNG HỌC SINH GIỎI LÝ LUẬN VĂN HỌC NĂM HỌC 2023-2024 - MÔN NGỮ ...
TÀI LIỆU BỒI DƯỠNG HỌC SINH GIỎI LÝ LUẬN VĂN HỌC NĂM HỌC 2023-2024 - MÔN NGỮ ...TÀI LIỆU BỒI DƯỠNG HỌC SINH GIỎI LÝ LUẬN VĂN HỌC NĂM HỌC 2023-2024 - MÔN NGỮ ...
TÀI LIỆU BỒI DƯỠNG HỌC SINH GIỎI LÝ LUẬN VĂN HỌC NĂM HỌC 2023-2024 - MÔN NGỮ ...Nguyen Thanh Tu Collection
 
30 ĐỀ PHÁT TRIỂN THEO CẤU TRÚC ĐỀ MINH HỌA BGD NGÀY 22-3-2024 KỲ THI TỐT NGHI...
30 ĐỀ PHÁT TRIỂN THEO CẤU TRÚC ĐỀ MINH HỌA BGD NGÀY 22-3-2024 KỲ THI TỐT NGHI...30 ĐỀ PHÁT TRIỂN THEO CẤU TRÚC ĐỀ MINH HỌA BGD NGÀY 22-3-2024 KỲ THI TỐT NGHI...
30 ĐỀ PHÁT TRIỂN THEO CẤU TRÚC ĐỀ MINH HỌA BGD NGÀY 22-3-2024 KỲ THI TỐT NGHI...Nguyen Thanh Tu Collection
 
GIÁO ÁN DẠY THÊM (KẾ HOẠCH BÀI DẠY BUỔI 2) - TIẾNG ANH 7 GLOBAL SUCCESS (2 CỘ...
GIÁO ÁN DẠY THÊM (KẾ HOẠCH BÀI DẠY BUỔI 2) - TIẾNG ANH 7 GLOBAL SUCCESS (2 CỘ...GIÁO ÁN DẠY THÊM (KẾ HOẠCH BÀI DẠY BUỔI 2) - TIẾNG ANH 7 GLOBAL SUCCESS (2 CỘ...
GIÁO ÁN DẠY THÊM (KẾ HOẠCH BÀI DẠY BUỔI 2) - TIẾNG ANH 7 GLOBAL SUCCESS (2 CỘ...Nguyen Thanh Tu Collection
 
SÁNG KIẾN ÁP DỤNG CLT (COMMUNICATIVE LANGUAGE TEACHING) VÀO QUÁ TRÌNH DẠY - H...
SÁNG KIẾN ÁP DỤNG CLT (COMMUNICATIVE LANGUAGE TEACHING) VÀO QUÁ TRÌNH DẠY - H...SÁNG KIẾN ÁP DỤNG CLT (COMMUNICATIVE LANGUAGE TEACHING) VÀO QUÁ TRÌNH DẠY - H...
SÁNG KIẾN ÁP DỤNG CLT (COMMUNICATIVE LANGUAGE TEACHING) VÀO QUÁ TRÌNH DẠY - H...Nguyen Thanh Tu Collection
 
30 ĐỀ PHÁT TRIỂN THEO CẤU TRÚC ĐỀ MINH HỌA BGD NGÀY 22-3-2024 KỲ THI TỐT NGHI...
30 ĐỀ PHÁT TRIỂN THEO CẤU TRÚC ĐỀ MINH HỌA BGD NGÀY 22-3-2024 KỲ THI TỐT NGHI...30 ĐỀ PHÁT TRIỂN THEO CẤU TRÚC ĐỀ MINH HỌA BGD NGÀY 22-3-2024 KỲ THI TỐT NGHI...
30 ĐỀ PHÁT TRIỂN THEO CẤU TRÚC ĐỀ MINH HỌA BGD NGÀY 22-3-2024 KỲ THI TỐT NGHI...Nguyen Thanh Tu Collection
 
30 ĐỀ PHÁT TRIỂN THEO CẤU TRÚC ĐỀ MINH HỌA BGD NGÀY 22-3-2024 KỲ THI TỐT NGHI...
30 ĐỀ PHÁT TRIỂN THEO CẤU TRÚC ĐỀ MINH HỌA BGD NGÀY 22-3-2024 KỲ THI TỐT NGHI...30 ĐỀ PHÁT TRIỂN THEO CẤU TRÚC ĐỀ MINH HỌA BGD NGÀY 22-3-2024 KỲ THI TỐT NGHI...
30 ĐỀ PHÁT TRIỂN THEO CẤU TRÚC ĐỀ MINH HỌA BGD NGÀY 22-3-2024 KỲ THI TỐT NGHI...Nguyen Thanh Tu Collection
 
30 ĐỀ PHÁT TRIỂN THEO CẤU TRÚC ĐỀ MINH HỌA BGD NGÀY 22-3-2024 KỲ THI TỐT NGHI...
30 ĐỀ PHÁT TRIỂN THEO CẤU TRÚC ĐỀ MINH HỌA BGD NGÀY 22-3-2024 KỲ THI TỐT NGHI...30 ĐỀ PHÁT TRIỂN THEO CẤU TRÚC ĐỀ MINH HỌA BGD NGÀY 22-3-2024 KỲ THI TỐT NGHI...
30 ĐỀ PHÁT TRIỂN THEO CẤU TRÚC ĐỀ MINH HỌA BGD NGÀY 22-3-2024 KỲ THI TỐT NGHI...Nguyen Thanh Tu Collection
 
30 ĐỀ PHÁT TRIỂN THEO CẤU TRÚC ĐỀ MINH HỌA BGD NGÀY 22-3-2024 KỲ THI TỐT NGHI...
30 ĐỀ PHÁT TRIỂN THEO CẤU TRÚC ĐỀ MINH HỌA BGD NGÀY 22-3-2024 KỲ THI TỐT NGHI...30 ĐỀ PHÁT TRIỂN THEO CẤU TRÚC ĐỀ MINH HỌA BGD NGÀY 22-3-2024 KỲ THI TỐT NGHI...
30 ĐỀ PHÁT TRIỂN THEO CẤU TRÚC ĐỀ MINH HỌA BGD NGÀY 22-3-2024 KỲ THI TỐT NGHI...Nguyen Thanh Tu Collection
 
TỔNG HỢP ĐỀ THI CHÍNH THỨC KỲ THI TUYỂN SINH VÀO LỚP 10 THPT MÔN NGỮ VĂN NĂM ...
TỔNG HỢP ĐỀ THI CHÍNH THỨC KỲ THI TUYỂN SINH VÀO LỚP 10 THPT MÔN NGỮ VĂN NĂM ...TỔNG HỢP ĐỀ THI CHÍNH THỨC KỲ THI TUYỂN SINH VÀO LỚP 10 THPT MÔN NGỮ VĂN NĂM ...
TỔNG HỢP ĐỀ THI CHÍNH THỨC KỲ THI TUYỂN SINH VÀO LỚP 10 THPT MÔN NGỮ VĂN NĂM ...Nguyen Thanh Tu Collection
 
TUYỂN TẬP 20 ĐỀ THI KHẢO SÁT HỌC SINH GIỎI MÔN TIẾNG ANH LỚP 6 NĂM 2020 (CÓ Đ...
TUYỂN TẬP 20 ĐỀ THI KHẢO SÁT HỌC SINH GIỎI MÔN TIẾNG ANH LỚP 6 NĂM 2020 (CÓ Đ...TUYỂN TẬP 20 ĐỀ THI KHẢO SÁT HỌC SINH GIỎI MÔN TIẾNG ANH LỚP 6 NĂM 2020 (CÓ Đ...
TUYỂN TẬP 20 ĐỀ THI KHẢO SÁT HỌC SINH GIỎI MÔN TIẾNG ANH LỚP 6 NĂM 2020 (CÓ Đ...Nguyen Thanh Tu Collection
 
TUYỂN TẬP 25 ĐỀ THI HỌC SINH GIỎI MÔN TIẾNG ANH LỚP 6 NĂM 2023 CÓ ĐÁP ÁN (SƯU...
TUYỂN TẬP 25 ĐỀ THI HỌC SINH GIỎI MÔN TIẾNG ANH LỚP 6 NĂM 2023 CÓ ĐÁP ÁN (SƯU...TUYỂN TẬP 25 ĐỀ THI HỌC SINH GIỎI MÔN TIẾNG ANH LỚP 6 NĂM 2023 CÓ ĐÁP ÁN (SƯU...
TUYỂN TẬP 25 ĐỀ THI HỌC SINH GIỎI MÔN TIẾNG ANH LỚP 6 NĂM 2023 CÓ ĐÁP ÁN (SƯU...Nguyen Thanh Tu Collection
 
BỘ ĐỀ PHÁT TRIỂN THEO CẤU TRÚC ĐỀ MINH HỌA BGD NGÀY 22-3-2024 KỲ THI TỐT NGHI...
BỘ ĐỀ PHÁT TRIỂN THEO CẤU TRÚC ĐỀ MINH HỌA BGD NGÀY 22-3-2024 KỲ THI TỐT NGHI...BỘ ĐỀ PHÁT TRIỂN THEO CẤU TRÚC ĐỀ MINH HỌA BGD NGÀY 22-3-2024 KỲ THI TỐT NGHI...
BỘ ĐỀ PHÁT TRIỂN THEO CẤU TRÚC ĐỀ MINH HỌA BGD NGÀY 22-3-2024 KỲ THI TỐT NGHI...Nguyen Thanh Tu Collection
 
Sáng kiến Dạy học theo định hướng STEM một số chủ đề phần “vật sống”, Khoa họ...
Sáng kiến Dạy học theo định hướng STEM một số chủ đề phần “vật sống”, Khoa họ...Sáng kiến Dạy học theo định hướng STEM một số chủ đề phần “vật sống”, Khoa họ...
Sáng kiến Dạy học theo định hướng STEM một số chủ đề phần “vật sống”, Khoa họ...Nguyen Thanh Tu Collection
 
Sáng kiến “Sử dụng ứng dụng Quizizz nhằm nâng cao chất lượng ôn thi tốt nghiệ...
Sáng kiến “Sử dụng ứng dụng Quizizz nhằm nâng cao chất lượng ôn thi tốt nghiệ...Sáng kiến “Sử dụng ứng dụng Quizizz nhằm nâng cao chất lượng ôn thi tốt nghiệ...
Sáng kiến “Sử dụng ứng dụng Quizizz nhằm nâng cao chất lượng ôn thi tốt nghiệ...Nguyen Thanh Tu Collection
 
30 ĐỀ PHÁT TRIỂN THEO CẤU TRÚC ĐỀ MINH HỌA BGD NGÀY 22-3-2024 KỲ THI TỐT NGHI...
30 ĐỀ PHÁT TRIỂN THEO CẤU TRÚC ĐỀ MINH HỌA BGD NGÀY 22-3-2024 KỲ THI TỐT NGHI...30 ĐỀ PHÁT TRIỂN THEO CẤU TRÚC ĐỀ MINH HỌA BGD NGÀY 22-3-2024 KỲ THI TỐT NGHI...
30 ĐỀ PHÁT TRIỂN THEO CẤU TRÚC ĐỀ MINH HỌA BGD NGÀY 22-3-2024 KỲ THI TỐT NGHI...Nguyen Thanh Tu Collection
 
10 ĐỀ KIỂM TRA + 6 ĐỀ ÔN TẬP CUỐI KÌ 2 VẬT LÝ 11 - KẾT NỐI TRI THỨC - THEO C...
10 ĐỀ KIỂM TRA + 6 ĐỀ ÔN TẬP CUỐI KÌ 2 VẬT LÝ 11 - KẾT NỐI TRI THỨC - THEO C...10 ĐỀ KIỂM TRA + 6 ĐỀ ÔN TẬP CUỐI KÌ 2 VẬT LÝ 11 - KẾT NỐI TRI THỨC - THEO C...
10 ĐỀ KIỂM TRA + 6 ĐỀ ÔN TẬP CUỐI KÌ 2 VẬT LÝ 11 - KẾT NỐI TRI THỨC - THEO C...Nguyen Thanh Tu Collection
 

More from Nguyen Thanh Tu Collection (20)

GIÁO ÁN DẠY THÊM (KẾ HOẠCH BÀI DẠY BUỔI 2) - TIẾNG ANH 6, 7 GLOBAL SUCCESS (2...
GIÁO ÁN DẠY THÊM (KẾ HOẠCH BÀI DẠY BUỔI 2) - TIẾNG ANH 6, 7 GLOBAL SUCCESS (2...GIÁO ÁN DẠY THÊM (KẾ HOẠCH BÀI DẠY BUỔI 2) - TIẾNG ANH 6, 7 GLOBAL SUCCESS (2...
GIÁO ÁN DẠY THÊM (KẾ HOẠCH BÀI DẠY BUỔI 2) - TIẾNG ANH 6, 7 GLOBAL SUCCESS (2...
 
TÀI LIỆU BỒI DƯỠNG HỌC SINH GIỎI KỸ NĂNG VIẾT ĐOẠN VĂN NGHỊ LUẬN XÃ HỘI 200 C...
TÀI LIỆU BỒI DƯỠNG HỌC SINH GIỎI KỸ NĂNG VIẾT ĐOẠN VĂN NGHỊ LUẬN XÃ HỘI 200 C...TÀI LIỆU BỒI DƯỠNG HỌC SINH GIỎI KỸ NĂNG VIẾT ĐOẠN VĂN NGHỊ LUẬN XÃ HỘI 200 C...
TÀI LIỆU BỒI DƯỠNG HỌC SINH GIỎI KỸ NĂNG VIẾT ĐOẠN VĂN NGHỊ LUẬN XÃ HỘI 200 C...
 
BỘ LUYỆN NGHE VÀO 10 TIẾNG ANH DẠNG TRẮC NGHIỆM 4 CÂU TRẢ LỜI - CÓ FILE NGHE.pdf
BỘ LUYỆN NGHE VÀO 10 TIẾNG ANH DẠNG TRẮC NGHIỆM 4 CÂU TRẢ LỜI - CÓ FILE NGHE.pdfBỘ LUYỆN NGHE VÀO 10 TIẾNG ANH DẠNG TRẮC NGHIỆM 4 CÂU TRẢ LỜI - CÓ FILE NGHE.pdf
BỘ LUYỆN NGHE VÀO 10 TIẾNG ANH DẠNG TRẮC NGHIỆM 4 CÂU TRẢ LỜI - CÓ FILE NGHE.pdf
 
ĐỀ CHÍNH THỨC KỲ THI TUYỂN SINH VÀO LỚP 10 THPT CÁC TỈNH THÀNH NĂM HỌC 2020 –...
ĐỀ CHÍNH THỨC KỲ THI TUYỂN SINH VÀO LỚP 10 THPT CÁC TỈNH THÀNH NĂM HỌC 2020 –...ĐỀ CHÍNH THỨC KỲ THI TUYỂN SINH VÀO LỚP 10 THPT CÁC TỈNH THÀNH NĂM HỌC 2020 –...
ĐỀ CHÍNH THỨC KỲ THI TUYỂN SINH VÀO LỚP 10 THPT CÁC TỈNH THÀNH NĂM HỌC 2020 –...
 
TÀI LIỆU BỒI DƯỠNG HỌC SINH GIỎI LÝ LUẬN VĂN HỌC NĂM HỌC 2023-2024 - MÔN NGỮ ...
TÀI LIỆU BỒI DƯỠNG HỌC SINH GIỎI LÝ LUẬN VĂN HỌC NĂM HỌC 2023-2024 - MÔN NGỮ ...TÀI LIỆU BỒI DƯỠNG HỌC SINH GIỎI LÝ LUẬN VĂN HỌC NĂM HỌC 2023-2024 - MÔN NGỮ ...
TÀI LIỆU BỒI DƯỠNG HỌC SINH GIỎI LÝ LUẬN VĂN HỌC NĂM HỌC 2023-2024 - MÔN NGỮ ...
 
30 ĐỀ PHÁT TRIỂN THEO CẤU TRÚC ĐỀ MINH HỌA BGD NGÀY 22-3-2024 KỲ THI TỐT NGHI...
30 ĐỀ PHÁT TRIỂN THEO CẤU TRÚC ĐỀ MINH HỌA BGD NGÀY 22-3-2024 KỲ THI TỐT NGHI...30 ĐỀ PHÁT TRIỂN THEO CẤU TRÚC ĐỀ MINH HỌA BGD NGÀY 22-3-2024 KỲ THI TỐT NGHI...
30 ĐỀ PHÁT TRIỂN THEO CẤU TRÚC ĐỀ MINH HỌA BGD NGÀY 22-3-2024 KỲ THI TỐT NGHI...
 
GIÁO ÁN DẠY THÊM (KẾ HOẠCH BÀI DẠY BUỔI 2) - TIẾNG ANH 7 GLOBAL SUCCESS (2 CỘ...
GIÁO ÁN DẠY THÊM (KẾ HOẠCH BÀI DẠY BUỔI 2) - TIẾNG ANH 7 GLOBAL SUCCESS (2 CỘ...GIÁO ÁN DẠY THÊM (KẾ HOẠCH BÀI DẠY BUỔI 2) - TIẾNG ANH 7 GLOBAL SUCCESS (2 CỘ...
GIÁO ÁN DẠY THÊM (KẾ HOẠCH BÀI DẠY BUỔI 2) - TIẾNG ANH 7 GLOBAL SUCCESS (2 CỘ...
 
SÁNG KIẾN ÁP DỤNG CLT (COMMUNICATIVE LANGUAGE TEACHING) VÀO QUÁ TRÌNH DẠY - H...
SÁNG KIẾN ÁP DỤNG CLT (COMMUNICATIVE LANGUAGE TEACHING) VÀO QUÁ TRÌNH DẠY - H...SÁNG KIẾN ÁP DỤNG CLT (COMMUNICATIVE LANGUAGE TEACHING) VÀO QUÁ TRÌNH DẠY - H...
SÁNG KIẾN ÁP DỤNG CLT (COMMUNICATIVE LANGUAGE TEACHING) VÀO QUÁ TRÌNH DẠY - H...
 
30 ĐỀ PHÁT TRIỂN THEO CẤU TRÚC ĐỀ MINH HỌA BGD NGÀY 22-3-2024 KỲ THI TỐT NGHI...
30 ĐỀ PHÁT TRIỂN THEO CẤU TRÚC ĐỀ MINH HỌA BGD NGÀY 22-3-2024 KỲ THI TỐT NGHI...30 ĐỀ PHÁT TRIỂN THEO CẤU TRÚC ĐỀ MINH HỌA BGD NGÀY 22-3-2024 KỲ THI TỐT NGHI...
30 ĐỀ PHÁT TRIỂN THEO CẤU TRÚC ĐỀ MINH HỌA BGD NGÀY 22-3-2024 KỲ THI TỐT NGHI...
 
30 ĐỀ PHÁT TRIỂN THEO CẤU TRÚC ĐỀ MINH HỌA BGD NGÀY 22-3-2024 KỲ THI TỐT NGHI...
30 ĐỀ PHÁT TRIỂN THEO CẤU TRÚC ĐỀ MINH HỌA BGD NGÀY 22-3-2024 KỲ THI TỐT NGHI...30 ĐỀ PHÁT TRIỂN THEO CẤU TRÚC ĐỀ MINH HỌA BGD NGÀY 22-3-2024 KỲ THI TỐT NGHI...
30 ĐỀ PHÁT TRIỂN THEO CẤU TRÚC ĐỀ MINH HỌA BGD NGÀY 22-3-2024 KỲ THI TỐT NGHI...
 
30 ĐỀ PHÁT TRIỂN THEO CẤU TRÚC ĐỀ MINH HỌA BGD NGÀY 22-3-2024 KỲ THI TỐT NGHI...
30 ĐỀ PHÁT TRIỂN THEO CẤU TRÚC ĐỀ MINH HỌA BGD NGÀY 22-3-2024 KỲ THI TỐT NGHI...30 ĐỀ PHÁT TRIỂN THEO CẤU TRÚC ĐỀ MINH HỌA BGD NGÀY 22-3-2024 KỲ THI TỐT NGHI...
30 ĐỀ PHÁT TRIỂN THEO CẤU TRÚC ĐỀ MINH HỌA BGD NGÀY 22-3-2024 KỲ THI TỐT NGHI...
 
30 ĐỀ PHÁT TRIỂN THEO CẤU TRÚC ĐỀ MINH HỌA BGD NGÀY 22-3-2024 KỲ THI TỐT NGHI...
30 ĐỀ PHÁT TRIỂN THEO CẤU TRÚC ĐỀ MINH HỌA BGD NGÀY 22-3-2024 KỲ THI TỐT NGHI...30 ĐỀ PHÁT TRIỂN THEO CẤU TRÚC ĐỀ MINH HỌA BGD NGÀY 22-3-2024 KỲ THI TỐT NGHI...
30 ĐỀ PHÁT TRIỂN THEO CẤU TRÚC ĐỀ MINH HỌA BGD NGÀY 22-3-2024 KỲ THI TỐT NGHI...
 
TỔNG HỢP ĐỀ THI CHÍNH THỨC KỲ THI TUYỂN SINH VÀO LỚP 10 THPT MÔN NGỮ VĂN NĂM ...
TỔNG HỢP ĐỀ THI CHÍNH THỨC KỲ THI TUYỂN SINH VÀO LỚP 10 THPT MÔN NGỮ VĂN NĂM ...TỔNG HỢP ĐỀ THI CHÍNH THỨC KỲ THI TUYỂN SINH VÀO LỚP 10 THPT MÔN NGỮ VĂN NĂM ...
TỔNG HỢP ĐỀ THI CHÍNH THỨC KỲ THI TUYỂN SINH VÀO LỚP 10 THPT MÔN NGỮ VĂN NĂM ...
 
TUYỂN TẬP 20 ĐỀ THI KHẢO SÁT HỌC SINH GIỎI MÔN TIẾNG ANH LỚP 6 NĂM 2020 (CÓ Đ...
TUYỂN TẬP 20 ĐỀ THI KHẢO SÁT HỌC SINH GIỎI MÔN TIẾNG ANH LỚP 6 NĂM 2020 (CÓ Đ...TUYỂN TẬP 20 ĐỀ THI KHẢO SÁT HỌC SINH GIỎI MÔN TIẾNG ANH LỚP 6 NĂM 2020 (CÓ Đ...
TUYỂN TẬP 20 ĐỀ THI KHẢO SÁT HỌC SINH GIỎI MÔN TIẾNG ANH LỚP 6 NĂM 2020 (CÓ Đ...
 
TUYỂN TẬP 25 ĐỀ THI HỌC SINH GIỎI MÔN TIẾNG ANH LỚP 6 NĂM 2023 CÓ ĐÁP ÁN (SƯU...
TUYỂN TẬP 25 ĐỀ THI HỌC SINH GIỎI MÔN TIẾNG ANH LỚP 6 NĂM 2023 CÓ ĐÁP ÁN (SƯU...TUYỂN TẬP 25 ĐỀ THI HỌC SINH GIỎI MÔN TIẾNG ANH LỚP 6 NĂM 2023 CÓ ĐÁP ÁN (SƯU...
TUYỂN TẬP 25 ĐỀ THI HỌC SINH GIỎI MÔN TIẾNG ANH LỚP 6 NĂM 2023 CÓ ĐÁP ÁN (SƯU...
 
BỘ ĐỀ PHÁT TRIỂN THEO CẤU TRÚC ĐỀ MINH HỌA BGD NGÀY 22-3-2024 KỲ THI TỐT NGHI...
BỘ ĐỀ PHÁT TRIỂN THEO CẤU TRÚC ĐỀ MINH HỌA BGD NGÀY 22-3-2024 KỲ THI TỐT NGHI...BỘ ĐỀ PHÁT TRIỂN THEO CẤU TRÚC ĐỀ MINH HỌA BGD NGÀY 22-3-2024 KỲ THI TỐT NGHI...
BỘ ĐỀ PHÁT TRIỂN THEO CẤU TRÚC ĐỀ MINH HỌA BGD NGÀY 22-3-2024 KỲ THI TỐT NGHI...
 
Sáng kiến Dạy học theo định hướng STEM một số chủ đề phần “vật sống”, Khoa họ...
Sáng kiến Dạy học theo định hướng STEM một số chủ đề phần “vật sống”, Khoa họ...Sáng kiến Dạy học theo định hướng STEM một số chủ đề phần “vật sống”, Khoa họ...
Sáng kiến Dạy học theo định hướng STEM một số chủ đề phần “vật sống”, Khoa họ...
 
Sáng kiến “Sử dụng ứng dụng Quizizz nhằm nâng cao chất lượng ôn thi tốt nghiệ...
Sáng kiến “Sử dụng ứng dụng Quizizz nhằm nâng cao chất lượng ôn thi tốt nghiệ...Sáng kiến “Sử dụng ứng dụng Quizizz nhằm nâng cao chất lượng ôn thi tốt nghiệ...
Sáng kiến “Sử dụng ứng dụng Quizizz nhằm nâng cao chất lượng ôn thi tốt nghiệ...
 
30 ĐỀ PHÁT TRIỂN THEO CẤU TRÚC ĐỀ MINH HỌA BGD NGÀY 22-3-2024 KỲ THI TỐT NGHI...
30 ĐỀ PHÁT TRIỂN THEO CẤU TRÚC ĐỀ MINH HỌA BGD NGÀY 22-3-2024 KỲ THI TỐT NGHI...30 ĐỀ PHÁT TRIỂN THEO CẤU TRÚC ĐỀ MINH HỌA BGD NGÀY 22-3-2024 KỲ THI TỐT NGHI...
30 ĐỀ PHÁT TRIỂN THEO CẤU TRÚC ĐỀ MINH HỌA BGD NGÀY 22-3-2024 KỲ THI TỐT NGHI...
 
10 ĐỀ KIỂM TRA + 6 ĐỀ ÔN TẬP CUỐI KÌ 2 VẬT LÝ 11 - KẾT NỐI TRI THỨC - THEO C...
10 ĐỀ KIỂM TRA + 6 ĐỀ ÔN TẬP CUỐI KÌ 2 VẬT LÝ 11 - KẾT NỐI TRI THỨC - THEO C...10 ĐỀ KIỂM TRA + 6 ĐỀ ÔN TẬP CUỐI KÌ 2 VẬT LÝ 11 - KẾT NỐI TRI THỨC - THEO C...
10 ĐỀ KIỂM TRA + 6 ĐỀ ÔN TẬP CUỐI KÌ 2 VẬT LÝ 11 - KẾT NỐI TRI THỨC - THEO C...
 

Recently uploaded

Các điều kiện bảo hiểm trong bảo hiểm hàng hoá
Các điều kiện bảo hiểm trong bảo hiểm hàng hoáCác điều kiện bảo hiểm trong bảo hiểm hàng hoá
Các điều kiện bảo hiểm trong bảo hiểm hàng hoámyvh40253
 
GNHH và KBHQ - giao nhận hàng hoá và khai báo hải quan
GNHH và KBHQ - giao nhận hàng hoá và khai báo hải quanGNHH và KBHQ - giao nhận hàng hoá và khai báo hải quan
GNHH và KBHQ - giao nhận hàng hoá và khai báo hải quanmyvh40253
 
3-BẢNG MÃ LỖI CỦA CÁC HÃNG ĐIỀU HÒA .pdf - ĐIỆN LẠNH BÁCH KHOA HÀ NỘI
3-BẢNG MÃ LỖI CỦA CÁC HÃNG ĐIỀU HÒA .pdf - ĐIỆN LẠNH BÁCH KHOA HÀ NỘI3-BẢNG MÃ LỖI CỦA CÁC HÃNG ĐIỀU HÒA .pdf - ĐIỆN LẠNH BÁCH KHOA HÀ NỘI
3-BẢNG MÃ LỖI CỦA CÁC HÃNG ĐIỀU HÒA .pdf - ĐIỆN LẠNH BÁCH KHOA HÀ NỘIĐiện Lạnh Bách Khoa Hà Nội
 
sách sinh học đại cương - Textbook.pdf
sách sinh học đại cương   -   Textbook.pdfsách sinh học đại cương   -   Textbook.pdf
sách sinh học đại cương - Textbook.pdfTrnHoa46
 
GIÁO TRÌNH KHỐI NGUỒN CÁC LOẠI - ĐIỆN LẠNH BÁCH KHOA HÀ NỘI
GIÁO TRÌNH  KHỐI NGUỒN CÁC LOẠI - ĐIỆN LẠNH BÁCH KHOA HÀ NỘIGIÁO TRÌNH  KHỐI NGUỒN CÁC LOẠI - ĐIỆN LẠNH BÁCH KHOA HÀ NỘI
GIÁO TRÌNH KHỐI NGUỒN CÁC LOẠI - ĐIỆN LẠNH BÁCH KHOA HÀ NỘIĐiện Lạnh Bách Khoa Hà Nội
 
Campbell _2011_ - Sinh học - Tế bào - Ref.pdf
Campbell _2011_ - Sinh học - Tế bào - Ref.pdfCampbell _2011_ - Sinh học - Tế bào - Ref.pdf
Campbell _2011_ - Sinh học - Tế bào - Ref.pdfTrnHoa46
 
Nhiễm khuẩn tiêu hóa-Tiêu chảy do vi khuẩn.pptx
Nhiễm khuẩn tiêu hóa-Tiêu chảy do vi khuẩn.pptxNhiễm khuẩn tiêu hóa-Tiêu chảy do vi khuẩn.pptx
Nhiễm khuẩn tiêu hóa-Tiêu chảy do vi khuẩn.pptxhoangvubaongoc112011
 
Chuong trinh dao tao Su pham Khoa hoc tu nhien, ma nganh - 7140247.pdf
Chuong trinh dao tao Su pham Khoa hoc tu nhien, ma nganh - 7140247.pdfChuong trinh dao tao Su pham Khoa hoc tu nhien, ma nganh - 7140247.pdf
Chuong trinh dao tao Su pham Khoa hoc tu nhien, ma nganh - 7140247.pdfhoangtuansinh1
 
1.DOANNGOCPHUONGTHAO-APDUNGSTEMTHIETKEBTHHHGIUPHSHOCHIEUQUA (1).docx
1.DOANNGOCPHUONGTHAO-APDUNGSTEMTHIETKEBTHHHGIUPHSHOCHIEUQUA (1).docx1.DOANNGOCPHUONGTHAO-APDUNGSTEMTHIETKEBTHHHGIUPHSHOCHIEUQUA (1).docx
1.DOANNGOCPHUONGTHAO-APDUNGSTEMTHIETKEBTHHHGIUPHSHOCHIEUQUA (1).docxTHAO316680
 
PHƯƠNG THỨC VẬN TẢI ĐƯỜNG SẮT TRONG VẬN TẢI
PHƯƠNG THỨC VẬN TẢI ĐƯỜNG SẮT TRONG VẬN TẢIPHƯƠNG THỨC VẬN TẢI ĐƯỜNG SẮT TRONG VẬN TẢI
PHƯƠNG THỨC VẬN TẢI ĐƯỜNG SẮT TRONG VẬN TẢImyvh40253
 
Đề cương môn giải phẫu......................
Đề cương môn giải phẫu......................Đề cương môn giải phẫu......................
Đề cương môn giải phẫu......................TrnHoa46
 
CD21 Exercise 2.1 KEY.docx tieng anh cho
CD21 Exercise 2.1 KEY.docx tieng anh choCD21 Exercise 2.1 KEY.docx tieng anh cho
CD21 Exercise 2.1 KEY.docx tieng anh chonamc250
 
Kiểm tra cuối học kì 1 sinh học 12 đề tham khảo
Kiểm tra cuối học kì 1 sinh học 12 đề tham khảoKiểm tra cuối học kì 1 sinh học 12 đề tham khảo
Kiểm tra cuối học kì 1 sinh học 12 đề tham khảohoanhv296
 
Giới thiệu Dự án Sản Phụ Khoa - Y Học Cộng Đồng
Giới thiệu Dự án Sản Phụ Khoa - Y Học Cộng ĐồngGiới thiệu Dự án Sản Phụ Khoa - Y Học Cộng Đồng
Giới thiệu Dự án Sản Phụ Khoa - Y Học Cộng ĐồngYhoccongdong.com
 

Recently uploaded (15)

Các điều kiện bảo hiểm trong bảo hiểm hàng hoá
Các điều kiện bảo hiểm trong bảo hiểm hàng hoáCác điều kiện bảo hiểm trong bảo hiểm hàng hoá
Các điều kiện bảo hiểm trong bảo hiểm hàng hoá
 
GNHH và KBHQ - giao nhận hàng hoá và khai báo hải quan
GNHH và KBHQ - giao nhận hàng hoá và khai báo hải quanGNHH và KBHQ - giao nhận hàng hoá và khai báo hải quan
GNHH và KBHQ - giao nhận hàng hoá và khai báo hải quan
 
3-BẢNG MÃ LỖI CỦA CÁC HÃNG ĐIỀU HÒA .pdf - ĐIỆN LẠNH BÁCH KHOA HÀ NỘI
3-BẢNG MÃ LỖI CỦA CÁC HÃNG ĐIỀU HÒA .pdf - ĐIỆN LẠNH BÁCH KHOA HÀ NỘI3-BẢNG MÃ LỖI CỦA CÁC HÃNG ĐIỀU HÒA .pdf - ĐIỆN LẠNH BÁCH KHOA HÀ NỘI
3-BẢNG MÃ LỖI CỦA CÁC HÃNG ĐIỀU HÒA .pdf - ĐIỆN LẠNH BÁCH KHOA HÀ NỘI
 
1 - MÃ LỖI SỬA CHỮA BOARD MẠCH BẾP TỪ.pdf
1 - MÃ LỖI SỬA CHỮA BOARD MẠCH BẾP TỪ.pdf1 - MÃ LỖI SỬA CHỮA BOARD MẠCH BẾP TỪ.pdf
1 - MÃ LỖI SỬA CHỮA BOARD MẠCH BẾP TỪ.pdf
 
sách sinh học đại cương - Textbook.pdf
sách sinh học đại cương   -   Textbook.pdfsách sinh học đại cương   -   Textbook.pdf
sách sinh học đại cương - Textbook.pdf
 
GIÁO TRÌNH KHỐI NGUỒN CÁC LOẠI - ĐIỆN LẠNH BÁCH KHOA HÀ NỘI
GIÁO TRÌNH  KHỐI NGUỒN CÁC LOẠI - ĐIỆN LẠNH BÁCH KHOA HÀ NỘIGIÁO TRÌNH  KHỐI NGUỒN CÁC LOẠI - ĐIỆN LẠNH BÁCH KHOA HÀ NỘI
GIÁO TRÌNH KHỐI NGUỒN CÁC LOẠI - ĐIỆN LẠNH BÁCH KHOA HÀ NỘI
 
Campbell _2011_ - Sinh học - Tế bào - Ref.pdf
Campbell _2011_ - Sinh học - Tế bào - Ref.pdfCampbell _2011_ - Sinh học - Tế bào - Ref.pdf
Campbell _2011_ - Sinh học - Tế bào - Ref.pdf
 
Nhiễm khuẩn tiêu hóa-Tiêu chảy do vi khuẩn.pptx
Nhiễm khuẩn tiêu hóa-Tiêu chảy do vi khuẩn.pptxNhiễm khuẩn tiêu hóa-Tiêu chảy do vi khuẩn.pptx
Nhiễm khuẩn tiêu hóa-Tiêu chảy do vi khuẩn.pptx
 
Chuong trinh dao tao Su pham Khoa hoc tu nhien, ma nganh - 7140247.pdf
Chuong trinh dao tao Su pham Khoa hoc tu nhien, ma nganh - 7140247.pdfChuong trinh dao tao Su pham Khoa hoc tu nhien, ma nganh - 7140247.pdf
Chuong trinh dao tao Su pham Khoa hoc tu nhien, ma nganh - 7140247.pdf
 
1.DOANNGOCPHUONGTHAO-APDUNGSTEMTHIETKEBTHHHGIUPHSHOCHIEUQUA (1).docx
1.DOANNGOCPHUONGTHAO-APDUNGSTEMTHIETKEBTHHHGIUPHSHOCHIEUQUA (1).docx1.DOANNGOCPHUONGTHAO-APDUNGSTEMTHIETKEBTHHHGIUPHSHOCHIEUQUA (1).docx
1.DOANNGOCPHUONGTHAO-APDUNGSTEMTHIETKEBTHHHGIUPHSHOCHIEUQUA (1).docx
 
PHƯƠNG THỨC VẬN TẢI ĐƯỜNG SẮT TRONG VẬN TẢI
PHƯƠNG THỨC VẬN TẢI ĐƯỜNG SẮT TRONG VẬN TẢIPHƯƠNG THỨC VẬN TẢI ĐƯỜNG SẮT TRONG VẬN TẢI
PHƯƠNG THỨC VẬN TẢI ĐƯỜNG SẮT TRONG VẬN TẢI
 
Đề cương môn giải phẫu......................
Đề cương môn giải phẫu......................Đề cương môn giải phẫu......................
Đề cương môn giải phẫu......................
 
CD21 Exercise 2.1 KEY.docx tieng anh cho
CD21 Exercise 2.1 KEY.docx tieng anh choCD21 Exercise 2.1 KEY.docx tieng anh cho
CD21 Exercise 2.1 KEY.docx tieng anh cho
 
Kiểm tra cuối học kì 1 sinh học 12 đề tham khảo
Kiểm tra cuối học kì 1 sinh học 12 đề tham khảoKiểm tra cuối học kì 1 sinh học 12 đề tham khảo
Kiểm tra cuối học kì 1 sinh học 12 đề tham khảo
 
Giới thiệu Dự án Sản Phụ Khoa - Y Học Cộng Đồng
Giới thiệu Dự án Sản Phụ Khoa - Y Học Cộng ĐồngGiới thiệu Dự án Sản Phụ Khoa - Y Học Cộng Đồng
Giới thiệu Dự án Sản Phụ Khoa - Y Học Cộng Đồng
 

DE PHAT TRIEN THEO CAU TRUC DE MINH HOA 2022 MON TIENG ANH DE 21 25.pdf

  • 1. Question 1: Robert Mugabe has ruled Zimbabwe since the _______ achieved independence in 1980. A. national B. nation C. nationality D. nationally Kiến thức: Từ loại Giải thích: Xét cá đáp án: A. national (a): quốc gia B. Nation (n): đất nước C. nationality (n): quốc tịnh D. Nationally (adv): quốc gia - Sau mạo từ the + danh từ, có 2 đáp án B, C là danh từ Đáp án đúng là B Tạm dịch: Robert Mugabe đã thống trị Zimbabwe kể từ khi đất nước giành được độc lập vào năm1980.
  • 2. Question 2: It is time to begin, _______? A. isn’t it B. are they C. do I D. is it Kiến thức: Câu hỏi đuôi Giải thích: - câu hỏi đuôi, It is là khẳng định => đuôi là phủ định Đáp án đúng là A Tạm dịch: Đã đến lúc bắt đầu rồi, đúng không?
  • 3. Question 3: At first, I give a(n) _______ impression that you are too anxious for a settlement. A. wrong B. false C. improper D. incorrect Kiến thức: Thành ngữ Giải thích: Ta có thành ngữ: give a false impression (collocation): nhầm tưởng Đáp án đúng là B Tạm dịch: Lúc đầu, tôi nhầm tưởng rằng bạn lo lắng cho bản thỏa thuận.
  • 4. Question 4: Grinder, a 35-year military veteran, was named to the post _______ Thursday, Jan. 13 A. on B. in C. of D. with Kiến thức: Giới từ Giải thích: Ta có, trước ngày tháng năm + on Đáp án đúng là A Tạm dịch: Grinder, một quân nhân 35 năm kinh nghiệm, được cử đến doanh trại quân đội vào thứ năm ngày 13/1.
  • 5. Question 5: Poteet will receive a $100 cash _______ and $200 for his school to purchase books. A. bonus B. award C. gift D. prize Kiến thức: Từ vựng Giải thích: Xét các đáp án: - prize : giải thưởng, phần thưởng (ít trang trọng, dùng cho người có hành động tốt) - award: giải thưởng, phần thưởng (dùng cho người nào đó làm hành động lớn lao ) - gift: món quà - bonus: phần thưởng thêm Đáp án đúng là D Tạm dịch: Poteet sẽ nhận được một phần thưởng tiền mặt 200 đô và 200 đô cho trường anh ấy để mua sách.
  • 6. Question 6: Kids without guidance got into trouble _______ there was nothing else to do. A. because B. though C. because of D. despite Kiến thức: Liên từ Giải thích: Xét các đáp án: - because + clause: bởi vì - because of + N: bởi vì - though + clause: mặc dù - despite + N: mặc dù Đáp án đúng là A Tạm dịch: Những đứa trẻ không có sự hướng dẫn sẽ gặp phiền toái bởi vì không có cái gì khác để làm.
  • 7. Question 7: Tonight, Hana wore a _______ dress at the party. A. gorgeous red Chinese B. gorgeous Chinese red C. red Chinese gorgeous D. Chinese red gorgeous Kiến thức: Trật tự tính từ Giải thích: - Theo quy tắc trật tự tính từ trong câu: OSASCOMP - gorgeous : tính từ chỉ quan điểm - red: màu sắc - Chinese: nguồn gốc Đáp án đúng là A Tạm dịch: Tối nay, Hana mặc 1 chiếc váy lộng lẫy, màu đỏ , bắt nguồn từ Trung Hoa đến buổi tiệc.
  • 8. Question 8: Don't _______ when he is telling the story. A. stop in B. stop off C. break in D. break into Kiến thức: Cụm động từ Giải thích: Xét các đáp án: - stop in: ở nhà - stop off: ngừng chuyến đi/hành trình - break in: chen lời, ngắt lời - break into: đột nhập Tạm dịch: Đừng có ngắt lời khi anh ấy đang kể 1 câu chuyện.
  • 9. Question 9: My father _______ TV when my friend suddenly came to my house. A. has watched B. watches C. watched D. was watching Kiến thức: Thì quá khứ tiếp diễn Giải thích: Ta thấy mệnh đề trạng ngữ chỉ thời gian bắt đầu bằng WHEN và động từ CAME ở quá khứ đơn nên động từ ở mệnh đề chính ở thì quá khứ tiếp diễn (một hành động đang diễn ra thì bị một hành động khác xen vào). Công thức: S1 + was/ were + V1-ing + when + S2 + V2-simple past Đáp án đúng là D Tạm dịch: Khi ba tôi đang xem TV, bạn tôi bất thình lình ghé nhà tôi.
  • 10. Question 10: _______, I will give him your regards. A. When I see him B. When I saw him C. When I was seeing him D. When I had seen him Kiến thức: Sự phối hợp các thì (Tương lai & hiện tại) Giải thích: - Sau mệnh đề chỉ thời gian chia hiện tại, vế còn lại chia tương lai. (mang nghĩa tương lai) Đáp án đúng là A Tạm dịch: Khi tôi gặp anh ấy, tôi sẽ chuyển lời hỏi thăm của bạn.
  • 11. Question 11: Could you _______ me a favour and pick up Sam from school today? A. make B. do C. put D. go Kiến thức: Cụm từ cố định Giải thích: - Ta có cụm từ cố định: do sb a favour: giúp ai đó Đáp án đúng là B Tạm dịch: Bạn có thể giúp tôi đón Sam đi học về hôm nay không?
  • 12. Question 12: The doctor said that she might never be able to _______ children. A. care B. tolerate C. bear D. feed Kiến thức: Từ vựng Giải thích: Xét các đáp án: - bear (v): chịu đựng/sinh con- tolerate (v): chịu đựng - care (v): quan tâm - feed (v): cho ai ăn Đáp án đúng là C Tạm dịch: Bác sĩ nói rằng cô ấy có khả năng không thể sinh con.
  • 13. Question 13: This house _______ in the 18th century. A. is building B. were build C. was built D. is built Kiến thức: Câu bị động Giải thích: Ta thấy, trạng ngữ chỉ thời gian “in the 18th century” – nên động từ chia ở quá khứ và chủ ngữ là “this house” nên động từ ở thể bị động. - Bị động quá khứ đơn: was/ were+ V3/ V-ed Đáp án đúng là C Tạm dịch: Ngôi nhà này được xây dựng thế kỉ 18.
  • 14. Question 14: _______ her homework, she watched her favourite movie. A. Having finished B. finish C. being finished D. she finishes Kiến thức : Rút gọn mệnh đề trạng ngữ Giải thích: - Khi hai mệnh đề có cùng chủ ngữ có thể rút gọn về: having +V3/ V-ed (quá khứ phân từ) Đáp án đúng là A Tạm dịch: Hoàn thành xong bài tập về nhà, cô ấy xem bộ phim yêu thích nhất
  • 15. Question 15: The sooner JK take your medicine, _______ he will feel. A. better B. the better C. the good D. well Kiến thức: So sánh kép Giải thích: - So sánh đồng tiến: The + so sánh hơn S+V, The + so sánh hơn S+V: càng… càng Đáp án đúng là B Tạm dịch: JK càng uống thuốc sớm, anh ta càng cảm thấy khỏe
  • 16. Question 16: Ann and Peter are doing homework together. - Ann: “Could you lend me that book” - Peter: “_______” A. No, I’m busy. B. That’s what I think. C. Why did you say that? D. Of course, here it is. Kiến thức: Tình huống giao tiếp (Đáp lại lời đề nghị) Giải thích: Tình huống giao tiếp: - Ann: Bạn có thể cho tôi mượn quyển sách đó? - “_______” Xét các đáp án: - A. Không, tôi bận - B. Đó là những gì tôi nghĩ - C. Tại sao bạn nói như vậy? - Dĩ nhiên rồi. Đây nè.
  • 17. Question 17: Linda is thanking Daniel for his birthday present. - Linda: “Thanks for the book. I’ve been looking for it for months.” - Daniel: “_______” A. You can say that again. B. Thank you for looking at it. C. I like reading books. D. I’m glad you like it. Kiến thức: Tình huống giao tiếp Giải thích: Tình huống giao tiếp: Linda đang cảm ơn Daniel về món quà sinh nhật. - Linda: “Thanks for the book. I’ve been looking for it for months.” - Daniel: “_______” Xét các đáp án: A. Tôi hoàn toàn đồng ý. B. Cảm ơn vì đã nhìn nó. C. Tôi thích đọc sách. D. Tôi nghĩ bạn thích nó.
  • 18. Question 18: A. permission B. suggestion C. refusal D. possible Kiến thức: Trọng âm của từ 3 âm tiết Giải thích: A. permission /pəˈmɪʃn/: Trọng âm rơi vào âm tiết thứ 2 vì theo quy tắc đuôi _sion trọng âm rơi vào âm tiết trước nó. B. suggestion /səˈdʒestʃən/: Trọng âm rơi vào âm tiết thứ 2 vì theo quy tắc đuôi _tion trọng âm rơi vào âm tiết trước nó. C. refusal /rɪˈ uːzl/: Trọng âm rơi vào âm tiết thứ 2 vì theo quy tắc hậu tố không thay đổi trọng âm chính D. possible /ˈpɒsəbl/: Trọng âm rơi vào âm tiết thứ nhất vì theo quy tắc từ 3 âm tiết mà cả 3 âm tiết đều chứa nguyên âm ngắn thì trọng âm rơi vào âm tiết thứ nhất.
  • 19. Question 19: A. forest B. succeed C. homeless D. nation Kiến thức: Trọng âm của từ 2 âm tiết Giải thích: A. forest /ˈfɒrɪst/ (n): Trọng âm rơi vào âm tiết thứ nhất vì theo quy tắc danh từ, tính từ 2 âm tiết trọng âm thường rơi vào âm tiết thứ nhất. B. succeed /səkˈsiːd/ (v): Trọng âm rơi vào âm tiết thứ hai vì theo quy tắc động từ 2 âm tiết trọng âm thường rơi vào âm tiết thứ hai. C. homeless /ˈhəʊmləs/ (adj): Trọng âm rơi vào âm tiết thứ nhất vì theo quy tắc danh từ, tính từ 2 âm tiết trọng âm thường rơi vào âm tiết thứ nhất. D. nation /ˈneɪʃn/ (n): Trọng âm rơi vào âm tiết thứ nhất vì theo quy tắc danh từ, tính từ 2 âm tiết trọng âm thường rơi vào âm tiết thứ nhất.
  • 20. Question 20: A. stopped B. wanted C. decided D. visited Kiến thức : Cách phát âm đuôi -ED Giải thích: – Phát âm là /t/ khi từ có tận cùng bằng các phụ âm vô thanh: /θ/, /p/, /k/, /f/, /s/, /ʃ/, /tʃ/ – Phát âm là /id/ khi từ có tận cùng là các âm: /t/, /d/ – Phát âm là /d/ khi các từ có tận cùng là nguyên âm và các phụ âm hữu thanh còn lại Xét các đáp án: A. stopped /t/ B. wanted /id/ C. decided /id/ D. visited /id/
  • 21. Question 21: A. head B. please C. heavy D. measure Kiến thức: Cách phát âm của nguyên âm Giải thích: Xét các đáp án: A. head /hed/ B. please /pliːz/ C. heavy /ˈhevi/ D. measure /ˈmeʒə(r)/ Ta thấy các phương án A, C, D – phần gạch chân được phát âm là /e/ còn phương án B được phát âm là /i:/
  • 22. Question 22: They tell us she isn't allowed to leave unless she gets the doctor's permission. A. disapproval B. objection C. agreement D. refusal Kiến thức: Đồng nghĩa (từ đơn) Giải thích: Ta có: permission (n) sự cho phép Xét các phương án: A. disapproval (n) sự không tán thành B. objection (n) sự phản đối C. agreement (n) sự đồng ý D. refusal (n) từ chối - permission (n) sự cho phép = agreement (n) sự đồng ý Vậy đáp án đúng là C Tạm dịch: Họ bảo với chúng tôi rằng cô ấy không được phép rời khỏi nếu không có sự đồng ý của bác sĩ.
  • 23. Question 23: Before the 1950s, urbanization mainly occurred in more economically developed countries (MEDCs). A. happened B. take place C. concentrated D. expanded Kiến thức: Đồng nghĩa (từ đơn) Giải thích: Ta có: occur (v): xảy ra, diễn ra Xét các đáp án: A. happen (v): diễn ra, xảy ra B. take place (phr.v): diễn ra, xảy ra => Ngữ cảnh đang chia ở quá khứ nên đáp án này không phù hợp về thì. C. concentrate /ˈkɑːnsntreɪt/ (v): tập trung D. expand /ɪkˈspænd/ (v): mở rộng =>Do đó: occurred ~ happened
  • 24. Question 24: I would love to go back to college, but unfortunately, that ship has sailed. A. it was late B. it was timely C. it was inconvenient D. it was traditional Kiến thức: Trái nghĩa (cụm từ hoặc thành ngữ) Giải thích: Ta có: That ship has sailed (idiom): đã quá muộn Xét các đáp án: A. It was late: đã muộn, trễ B. It was timely: kịp lúc C. It was inconvenient: bất tiện D. It was traditional: theo truyền thống That ship has sailed (idiom): đã quá muộn >< It was timely: kịp lúc Vậy đáp án đúng là B Tạm dịch: Tôi muốn quay lại trường nhưng thật không may, đã quá muộn.
  • 25. Question 25: A comfortable working environment will increase productivity. A. promote B. raise C. decrease D. go up Kiến thức: Trái nghĩa (từ đơn) Giải thích: Ta có: increase (v): tăng - promote (v) thúc đẩy, thăng chức - raise (v) = go up = increase: tăng>< decrease (v) giảm Vậy đáp án đúng là C Tạm dịch: Một môi trường làm việc thoải mái sẽ tang năng suất.
  • 26. Question 26: It was not until they had reached Dak Lak that they realized how far they had gone. A. Not until they had reached Dak Lak did they realize how far they had gone. B. Not until had they reached Dak Lak, they realized how far they had gone. C. Not until they reached Dak Lak had they realized how far they had gone. D. Not until they realized how far they had gone had they reached Dak Lak. Kiến thức: Kết hợp câu – đảo ngữ Giải thích: Cấu trúc đảo ngữ: Not until + clause, aux +S+V: chỉ tới khi mà…….thì….. It was not until they had reached Dak Lak that they realized how far they had gone. = A. Not until they had reached Dak Lak did they realize how far they had gone. Tạm dịch: Chỉ tới khi họ đến Đăk Lăk, họ mới nhận ra họ đã đi được bao xa.
  • 27. Question 27: They have a lot of homework today. A. They wish they had not a lot of homework today. B. They wish they didn’t have a lot of homework today. C. They wish they had had little homework today. D. They wish they wouldn’t have a lot of homework today. Kiến thức: Kết hợp câu – câu ước Giải thích: - câu ước muốn loại 2: S+wish+ S + V2/V-ed Câu đề bài: They have a lot of homework today. = B. They wish they didn’t have a lot of homework today. Tạm dịch: Họ có rất nhiều bài tập hôm nay =>Họ ước họ không có nhiều bài tập hôm nay
  • 28. Question 28: Let's start by reviewing what we do with those friends last week. A. start B. reviewing C. do D. friends Kiến thức: Lỗi sai – Thì của động từ Giải thích: - Sai thì của động từ vì trong câu có last week nên chia động từ ở thì quá khứ. Vậy đáp án đúng là C Sửa lỗi: do → did Tạm dịch: Hãy bắt đầu xem xét lại những gì chúng ta làm với những người bạn vào tuần trước.
  • 29. Question 29: Lily and her mother went to hospital last week to check our health. A. her B. went C. last D. our health Kiến thức: Lỗi sai – Đại từ nhân xưng/ tính từ sở hữu Giải thích: - Sai tính từ sở hữu vì chủ ngữ Lily and her mother nên tính từ sở hữu thay thế là THEIR Vậy đáp án đúng là D Sửa lỗi: our → their Tạm dịch: Lily và mẹ cô ấy đi bệnh viện tuần trước để kiểm tra sức khỏe.
  • 30. Question 30: I must complement you on your handling of a very difficult situation. A. must B. complement C. handling D. very Kiến thức: Lỗi sai – Từ vựng Giải thích: - complement: thêm vào - compliment sb on: khen ai đó Dựa vào nghĩa, đáp án đúng là B Sửa lỗi: complement → compliment Tạm dịch: Tôi phải khen bạn vì đã giải quyết 1 tình huống cực kì khó khăn
  • 31. Question 31: She advised him to go to the dentist immediately. A. He would go to the dentist immediately. B. She should go to the dentist immediately. C. He should go to the dentist immediately. D. He must go to the dentist immediately. Kiến thức: Câu đồng nghĩa – Động từ khuyết thiếu Giải thích: Câu đề bài: She advised him to go to the dentist immediately. (Cô ấy khen anh ta đi nha sĩ ngay lập tức.) (khuyên bảo) = C. He should go to the dentist immediately. (Anh ấy nên đi nha sĩ ngay lập tức.)
  • 32. Question 32: She said, “Don’t laugh, Jessica. Be serious.” A. She said not to laugh and be serious. B. She said Jessica not to laugh and be serious. C. She told Jessica not to laugh and asked him to be serious. D. She told Jessica do not laugh and be serious. Kiến thức: Câu đồng nghĩa – Câu tường thuật Giải thích: Câu đề bài: She said, “Don’t laugh, Jessica. Be serious.” (Cô ấy nói : “ Đừng cười, Jessica. Hãy nghiêm túc.”) - A,B, D. Sai cấu trúc ngữ pháp - tell sb (not) to do sth: bảo ai làm gì và ask sb to do sth: yêu cầu ai làm gì. Vậy đáp án đúng là C C. She told Jessica not to laugh and asked him to be serious. (Cô ấy bảo Jessica không cười và yêu cầu anh ấy nghiêm túc.)
  • 33. Question 33: The last time I played the piano was 10 years ago. A. I have not played the piano for 10 years ago. B. I have not played the piano for 10 years. C. I have not played the piano since 10 years. D. I did not play the piano since 10 years. Kiến thức: Câu đồng nghĩa – Thì của động từ Giải thích: Ta có công thức: S + haven’t/ hasn’t + V- pp + O + for + time = S + (last) + V-simple past + O + time + ago = It’s + time + since + S + (last) + V-simple past + O = The last time + S + V-simple past + O + was + time + ago Vậy đáp án đúng là B The last time I played the piano was 10 years ago. = B. I have not played the piano for 10 years. Tạm dịch: Lần cuối cùng tôi chơi piano là cách đây 10 năm.
  • 34. Question 34: A. many B. each C. much D. little Plants and animals will find it difficult to escape from or adjust to the effect of global warming. Scientists have already observed shifts in the life cycles of (34) _______ plants and animals, Kiến thức: Đọc điền từ Giải thích: Xét các đáp án: A. Many + danh từ số nhiều đếm được: nhiều B. Each + danh từ số ít: mỗi, một C. Much + danh từ không đếm được D. Little + danh từ không đếm được
  • 35. Question 35: A. increasing B. blooming C. growing D. swelling such as flowers (35) _______ earlier and birds hatching earlier in the spring. Kiến thức: Đọc điền từ Giải thích: Xét các đáp án: A. increasing : tăng B. blooming: nở hoa C. growing: phát triển D. swelling: sưng lên Dựa vào nghĩa, đáp án đúng là B Tạm dịch: Các nhà khoa học đã quan sát thấy sự thay đổi trong chu kỳ sống của nhiều loài thực vật và động vật, chẳng hạn như hoa nở sớm hơn và chim nở sớm hơn vào mùa xuân.
  • 36. Question 36: A. that B. what C. which D. where Many species have begun shifting (36) _______ they live or their annual migration patterns due to warmer temperatures. Kiến thức: Đọc điền từ Giải thích: Xét các đáp án: A. that: thay thế cho danh từ chỉ người và vật có chức năng làm chủ ngữ hoặc tân ngữ trong mệnh đề quan hệ xác định. B. what: cái gì (không phải đại từ quan hệ) C. which: thay thế cho danh từ chỉ vật có chức năng làm chủ ngữ hoặc tân ngữ trong mệnh đề quan hệ. D. where: thay thế cho danh từ chỉ nơi chốn và có chức năng làm trạng từ chỉ nơi chốn trong mệnh đề quan hệ. Ta thấy danh từ mà đại từ quan hệ thay thế “shifting” và sau đại từ quan hệ là “they live” nên đáp án đúng là D (where)
  • 37. Question 37: A. because B. but C. and D. although Species living in unique ecosystems, such as those found in polar and mountaintop regions, are especially at risk (37) _______ migration to new habitats is not possible. Kiến thức: Đọc điền từ Giải thích: Xét các đáp án: A. because (+ clause): bởi vì B. but : nhưng C. and : và D. although: mặc dù Tạm dịch: Các loài sống trong các hệ sinh thái độc đáo, chẳng hạn như các loài được tìm thấy ở các vùng cực và đỉnh núi, đặc biệt gặp rủi ro vì không thể di cư đến các môi trường sống mới.
  • 38. Question 38: A. decreasing B. falling C. deepening D. dwindling For example, polar bears and marine mammals in the Arctic are already threatened by (38) _______ sea ice but have nowhere farther to go. Kiến thức: Đọc điền từ Giải thích: Xét các đáp án: A. decreasing: giảm B. falling: rơi C. deepening: trở nên sâu hơn D. dwindling: tan, thu nhỏ dần Dựa vào nghĩa, đáp án đúng là D Tạm dịch: Ví dụ, gấu Bắc Cực và động vật biển có vú ở Bắc Cực đã bị đe dọa bởi băng biển đang tan dần nhưng không còn nơi nào xa hơn để đi.
  • 39. Question 39: What is the passage mainly about? A. A New Year’s Eve to remember. B. Preparations before the event. C. Activities during the ceremony. D. A great party. Kiến thức: Đọc hiểu Giải thích: Đoạn văn này chủ yếu nói về cái gì? A. Một đêm giao thừa đáng nhớ (tác giả kể lại trải nghiệm đón năm mới) B. Sự chuẩn bị trước sự kiện (đây chỉ là 1 ý trong đoạn vì có đề cập đến trong 5 ngày ở đầu đoạn 1) C. Sự chuẩn bị trong quá trình lễ (có trước và trong lễ): thông tin chưa đủ D. Một bữa tiệc tuyệt vời (trong bài đề cập đến năm mới không phải là tiệc)
  • 40. Question 40: Which of the following is NOT mentioned in paragraph 4 as preparations before the event? A. People decorated their houses. B. People bought new clothes and jewelry. C. Women made tasty candies. D. Women went to pagodas to pray for health. Kiến thức: Đọc hiểu Giải thích: Điều nào sau đây KHÔNG được đề cập trong đoạn 4 như là sự chuẩn bị trước sự kiện? Xét các đáp án: A. Mọi người trang trí nhà cửa. B. Mọi người mua quần áo và đồ trang sức mới. C. Phụ nữ làm ra những viên kẹo ngon. D. Phụ nữ đi lễ chùa để cầu sức khỏe. Thông tin: People cleaned their homes and painted wonderful designs called rangolis on the walls and floors. They bought beautiful new clothes and jewelry to wear during the festival, and women made delicious sweets …: Tạm dịch: Mọi người lau dọn nhà cửa và sơn những mẫu thiết kế tuyệt vời được gọi là rangolis trên tường và cửa chính. Họ mua quần áo mới và trang sức để mặc trong lễ hội và phụ nữ làm những món kẹo rất ngon… => đáp án không có thông tin trong bài là D
  • 41. Question 41: According to paragraph 2, which of the following can be true about the "Festival of Lights”? A. It takes place annually. B. Its aim is to celebrate the victory of evil over good. C. It lasts for 2 weeks. D. It takes place in the second week of November. Question 41: Đáp án A Kiến thức: Đọc hiểu Giải thích: Theo đoạn 2, câu nào sau đây là đúng về “Lễ hội ánh sáng” A. Nó diễn ra hàng năm B. Mục đích của nó là tổ chức sự chiến thắng cái ác đối với cái thiện (sai) C. Nó kéo dài 2 tuần D. Nó diễn ra vào tuần thứ 2 của tháng 11 Thông tin 1: the "Festival of Lights" which takes place every year to celebrate the victory of good over evil. Tạm dịch: Lễ hội ánh sáng diễn ra hàng năm để tổ chức chiến thắng cái thiện đối với cái ác. Thông tin 2: the "Festival of Lights" which takes place every year to celebrate the victory of good over evil. Tạm dịch: Lễ hội ánh sáng diễn ra hàng năm để tổ chức chiến thắng cái thiện đối với cái ác. Thông tin 3: This festival lasts for five days Tạm dịch: Lễ hội kéo dài 5 ngày Thông tin 4: at the end of October or in the first week of November: Tạm dịch: cuối tháng 10 hoặc tuần đầu tháng 11
  • 42. Question 42: The word glittered in paragraph 1 is closest in meaning to _________. A. sparkled B. lighted C. pressed D. rubbed Kiến thức: Đọc hiểu Giải thích: Từ glittered trong đoạn 1 gần nghĩa nhất với _________. Xét các đáp án: A. sparkled: lấp lánh B. lighted: sáng C. pressed : ép D. rubbed: cọ xát - glitter : long lanh = sparkle: lấp lánh Vậy đáp án đúng là A Thông tin: After dark, the whole neighbourhood glittered with thousands of tiny lights, as though in a fairy tale. Tạm dịch: Sau khi trời tối, cả khu phố lấp lánh với hàng nghìn ngọn đèn nhỏ như trong truyện cổ tích.
  • 43. Question 43: The word others in paragraph 4 refers to _________. A. people B. women C. sweets D. clothes Kiến thức: Đọc hiểu Giải thích: Từ others trong đoạn 4 đề cập đến _________. A. người B. phụ nữ C. đồ ngọt D. quần áo Thông tin: They bought beautiful new clothes and jewelry to wear during the festival, and women made delicious sweets which were better than any others I have ever tasted. Tạm dịch: Họ mua những bộ quần áo mới và đồ trang sức đẹp để mặc trong lễ hội, và những người phụ nữ làm những món đồ ngọt ngon tuyệt hơn bất kỳ loại đồ ngọt nào mà tôi từng nếm.
  • 44. Question 44: Which of the following is the best title for the passage? A. The disappearance of food species. B. Food for the Future. C. Ways to increase the number of food species. D. How to increase the food production. Kiến thức: Đọc hiểu Giải thích: Cái nào sau đây là tiêu đề tốt nhất của đoạn văn? A. Sự biến mất của các loài thực phẩm B. Thực phẩm cho tương lai C. Các cách để tăng số lượng loài thực phẩmD. Cách tang sản lượng thực phẩm Ta thấy, đoạn 1, 2 tác giả nói về thực trạng của việc trồng trọt, canh tác các loài của người nông dân. Do nhu cầu của con người dẫn đến nhiều loài có nguy cơ tuyệt chủng. Ở đoạn 3, 4, tác giả nói về giải pháp cho thực phẩm cho tương lai.
  • 45. Question 45: According to paragraph 1, why does the writer mention Ireland? A. To give an example of why it is dangerous to depend on a single crop species. B. To explain how worldwide interest in crop varieties first developed. C. To describe how Irish researchers are searching for a new crop variety. D. To illustrate how important it is to preserve potato plants. Kiến thức: Đọc hiểu Giải thích: Theo như đoạn 1, tại sao tác giả lại đề cập Ireland? A. Để đưa ra một ví dụ về lý do tại sao phụ thuộc vào một loài cây trồng duy nhất là nguy hiểm. B. Để giải thích mối quan tâm trên toàn thế giới về các giống cây trồng lần đầu tiên phát triển như thế nào. C. Để mô tả cách các nhà nghiên cứu Ireland đang tìm kiếm một giống cây trồng mới. D. Để minh họa tầm quan trọng của việc bảo quản cây khoai tây. Thông tin: But in Ireland, in 1845, people depended almost solely on the potato for food. The death of one species caused a terrible famine. Tạm dịch: Nhưng ở Ireland, vào năm 1845, mọi người hầu như chỉ phụ thuộc vào khoai tây để làm thực phẩm. Cái chết của một loài đã gây ra nạn đói khủng khiếp.
  • 46. Question 46: The word struck in paragraph 1 is closest in meaning to _________. A. approachedB. touched C. damaged D. reached Kiến thức: Đọc hiểu Giải thích: Từ struck trong đoạn 1 gần nghĩa nhất với _________. A. dính vào B. chạm vào C. gây tổn hại D. đạt tới - strike = damage: gây tổn hại, tổn thất Vậy đáp án đúng là C Thông tin: In 1845, a deadly disease struck the farms of Ireland, killing all the Lumper potato plants. Tạm dịch: Năm 1845, một căn bệnh chết người đã gây tổn hại các trang trại của Ireland, giết chết tất cả các cây khoai tây Lumper.
  • 47. Question 47: The word them in the final paragraph refers to _________. A. people B. seeds C. foods D. years Kiến thức: Đọc hiểu Giải thích: Từ them trong đoạn văn cuối cùng đề cập đến _________. A. con người B. hạt giống C. thực phẩm D. năm Thông tin: But the people at Heritage Farm don’t just store the seeds; they plant them. Tạm dịch: Nhưng những người ở Trang trại Di sản không chỉ lưu trữ hạt giống; họ trồng chúng.
  • 48. Question 48: The word flavor in the final paragraph is closest in meaning to _________. A. smell B. taste C. sense D. feeling Kiến thức: Đọc hiểu Giải thích: Từ flavor trong đoạn văn cuối cùng gần nghĩa nhất với _________. A. smell: ngửi B. Taste: nếm C. sense: giác quan D. feeling: cảm giác - flavor = taste: hương vị Vậy đáp án đúng là B Thông tin: These food species are not just special in terms of appearance or flavor. Tạm dịch: Những loài thực phẩm này không chỉ đặc biệt về hình thức hoặc hương vị.
  • 49. Question 49. Which of the following is NOT true according to the passage? A. Nikolay Vavilov make suggestions of collecting plant seeds. B. In Ireland, in 1845, there was a serious shortage of food. C. Nikolay Vavilov has worked in many seed banks around the world. D. Nikolay Vavilov was one of the pioneers to research a new crop. Kiến thức: Đọc hiểu Giải thích: Câu nào sau đây là không đúng theo đoạn văn? A. Nikolay Vavilov đưa ra đề nghị tập hợp những hạt giống B. Ở Ireland, năm 1845, thực phẩm thiếu trầm trọng C. Nikolay Vavilov đã làm việc tại nhiều ngân hàng hạt giống trên khắp thế giới. (Không có thông tin) D. Nikolay Vavilov là một trong những nhà tiên phong trong nghiên cứu vụ mùa mới. Thông tin 1: One solution to this problem is to collect and preserve the seeds of as many different plant varieties as we can before they disappear. The idea was first suggested by Russian scientist Nikolay Vavilov: Tạm dịch: Một giải pháp cho vấn đề này là thu thập và bảo quản hạt giống của càng nhiều giống cây trồng khác nhau càng tốt trước khi chúng biến mất. Ý tưởng lần đầu tiên được đề xuất bởi nhà khoa học người Nga Nikolay Vavilov. Thông tin 2. The death of one species caused a terrible famine. Tạm dịch: Cái chết của một loài đã gây ra nạn đói khủng khiếp.
  • 50. Question 50. Which of the following can be inferred from Daine Ott Whealy’s opinion? A. Historic seeds are usually difficult to reintroduce to the marketplace. B. It’s important to not only store seeds but also plant them. C. Food grown from historic and new seeds generally have a similar taste. D. Storing the seeds is likely to be time-consuming. Kiến thức: Đọc hiểu Giải thích: Điều nào sau đây có thể được suy ra từ ý kiến của Daine Ott Whealy? A. Hạt giống lịch sử thường khó được giới thiệu lại trên thị trường. B. Điều quan trọng là không chỉ lưu trữ hạt giống mà còn phải trồng chúng. C. Thức ăn được trồng từ hạt cũ và hạt mới thường có mùi vị tương tự nhau. D. Bảo quản hạt giống sẽ tốn nhiều thời gian. Thông tin: By doing this, they are reintroducing foods into the marketplace that haven’t been grown for years. Tạm dịch: Bằng cách làm này, họ đang đưa các loại thực phẩm vào thị trường đã không được trồng trong nhiều năm.
  • 51. Question 1. In some ways, financial planning for singles is more ________ than for couples. A. Importable B. importance C. important D. importantly Kiến thức: Từ loại Giải thích: Xét các đáp án: A. importable : (không có từ này) B. importance (n): tầm quan trọng C. important (a): quan trọng D. importantly (adv): một cách quan trọng Vị trí chỗ trống này cần một tính từ, vì phía trước có động từ “to be”. Vì vậy chọn đáp án A. Tạm dịch: Trong một số trường hợp, kế hoạch tài chính cho những người độc thân thì quan trọng hơn là đối với các cặp vợ chồng.
  • 52. Question 2. It is your homework, _______? A. isn’t it B. didn’t you C. don’t you D. do you Kiến thức: Câu hỏi đuôi Giải thích: - Chủ ngữ là “it” , phần câu trần thuật động từ chia ở phủ định của thì quá khứ đơn nên câu hỏi đuôi chia ở khẳng định. Dịch: Nó là bài tập về nhà của bạn phải không?
  • 53. Question 3. Cuba can't _________ business in America because of a nearly 50-year-old trade embargo. A. set B. do C. stay D. have Kiến thức: Từ vựng Giải thích: Xét các đáp án: A. Set up in business: mở kinh doanh B. Do business: làm ăn, kinh doanh C. Stay business: duy trì kinh doanh D. Have business: có quyền Vậy đáp án đúng là B Tạm dịch: Cu Ba không thể kinh doanh tại Mỹ do việc cấm vận thương mại trong gần 50 năm qua.
  • 54. Question 4. Thai Ngo Hieu’ s brave action has contributed_________beautifying the image of police officers in the people’s hearts. A. with B. to C. at D. for Kiến thức: Giới từ Giải thích: - Động từ “ contribute to”: nghĩa là : góp phần vào. Vì vậy nên đáp án B đúng. Tạm dịch: Hành động dũng cảm của Thai Ngo Hieu đã góp phần vào việc làm đẹp hình ảnh người công an trong lòng nhân dân..
  • 55. Question 5. I cannot think who had_______ the gaff, but it seems everyone knows that Nicole and I are planning to get married. A. burst B. blown C. split D. banged Kiến thức: Thành ngữ Giải thích: Xét các đáp án : A. burts (PII) : đốt cháy, thiêu cháy B. blown (PII) : thổi C. split (PII) : chia ra từng phần/ chia rẽ/ . D. banged (PII) : đánh mạnh/ đập mạnh Ta có thành ngữ: - Blow the gaff: tiết lộ bí mật Vậy đáp án đúng là B Tạm dịch: Tôi không thể nghĩ là ai đã tiết lộ bí mật , nhưng có vẻ như mọi người đều biết rằng Nicole và tôi đang chuẩn bị kết hôn.
  • 56. Question 6. At 84 he's still quite active ______ he walks with the aid of a stick. A. because B. because of C. despite D. although Kiến thức: Liên từ Giải thích: Xét các đáp án: A. Because (+ clause): bởi vì B. Because of (+ V- ing): bởi vì C. Despite (+ V-ing/ N.P): mặc dù D. Although (+ clause): mặc dù Dựa vào nghĩa, đáp án D là đúng Tạm dịch: Ở tuổi 84, ông ta khá là năng động mặc dù ông ấy vẫn phải chống gậy khi đi lại.
  • 57. Question 7. When I was young, I used to live in a____________house in the central of Vietnam. A. small wooden simple B. wooden simple small C. simple wooden small D. simple small wooden Kiến thức: Trật tự tính từ Giải thích: Theo quy tắc trật tự tính từ trong câu: OSASCOMP simple – opinion; small – size; wooden – Material Tạm dịch: Khi tôi còn trẻ, tôi đã từng sống trong một ngôi nhà bằng gỗ nhỏ, giản dijowr miền Trung Việt Nam.
  • 58. Question 8. Food is likely to __________ if its wrapping is damaged, since even the tiniest of holes can allow bacteria in. A. keep onB. put off C. leave out D. go off Kiến thức: Cụm động từ Giải thích: Xét các đáp án: A. keep on; tiếp tục B. put off: hoãn C. leave out: không bao gồm D. go off: nổ, reo, ôi thiu, ngừng thích Dựa vào nghĩa và ngữ cảnh, đáp án D là đúng Tạm dịch: Thức ăn có thể bị ôi thiu nếu phần đóng gói của nó bị hư hỏng , bởi vì thậm chí những cái lỗ nhỏ nhất cũng có thể cho phép vi khuẩn xâm nhập.
  • 59. Question 9. I ________a presentation in front of 500 people when the microphone stopped working. A. made B. was making C. had made D. make Kiến thức: Thì quá khứ tiếp diễn Giải thích: Căn cứ vào mệnh đề trạng ngữ chỉ thời gian: S + was/ were + V-ing when +S + Ved. Diễn tả hành động đang xảy ra thì hành động khác xen vào. Vậy vế đầu chia ở thì quá khứ tiếp diễn. Tạm dịch: Tôi đang phát biểu trước 500 người thì chiếc mic bị hỏng.
  • 60. Question 10. We will go on holiday in Nha Trang ___________. A. as soon as our son finishes the final examination B. before our son finished the final examination C. when our son will finish the final examination D. after our son had finished the final examination Kiến thức: Sự phối hợp các thì (Tương lai & hiện tại) Giải thích: Căn cứ vào công thức của mệnh đề trạng ngữ chỉ thời gian: Động từ ở mệnh đề chính chia ở thì tương lai + as soon as + S + V(s/es). Tạm dịch: Chúng tôi sẽ đi du lịch Nha Trang ngay khi con trai chúng tôi thi tốt nghiệp xong.
  • 61. Question 11. The closure of the school is a slap in the________to the local community. A. head B. feet C. mouth D. face Kiến thức: Cụm từ cố định Giải thích: a slap in the face: sự sỉ nhục/ sự lăng mạ Tạm dịch: Sự đóng cửa trường học là một sự sỉ nhục đối với cộng đồng địa phương
  • 62. Question 12. Researchers fear the virus could ________ and become transmissible between humans. A. vanish B. fade C. mutate D. rotate Kiến thức: Từ vựng Giải thích: Xét các đáp án : A. vanish (verb): biến mất B. fade (verb): mờ dần; bạc màu C. mutate (verb): đột biến, biến đổi D. rotate (verb): làm xoay quanh; luân phiên nhau Dựa nghĩa và ngữ cảnh, đáp án đúng là C Tạm dịch : Các nhà nghiên cứu đang lo sợ vi rút có thể biến đổi và trở nên dễ dàng lây nhiễm trên con người
  • 63. Question 13. Miss Vietnam Do Thi Ha __________ in the list of 13 finalists in the Top Model competition at the Miss World 2021 beauty pageant on December 7. A. was namedB. named C. was naming D. names Kiến thức: Câu bị động Giải thích: Căn cứ vào nghĩa, ta cân chia động từ ở thể bị động vì “Đỗ Thị Hà” “đã được gọi tên” chứ không thể tự mình gọi tên được. Do đó, động từ chia ở bị động và ở “ dạng “was + PII” Tạm dịch: Hoa Hậu Đỗ Thị Hà đã được gọi tên vào danh sách 13 thí sinh xuất săc trong phần thi Siêu Mẫu tại cuộc thi Hoa Hậu Thế Giới vào ngày 7thangs 12.
  • 64. Question 14________ every major tennis title, Alan retired from international competition. A. Having won B. Winning C. When he won D. On winning Kiến thức : Rút gọn mệnh đề trạng ngữ Giải thích: Khi hai mệnh đề cùng chủ ngữ, và câu muốn nhấn mạnh hành động phía trước được hoàn thành xong trước rồi hành động phía sau mới xảy ra thì chúng ta dùng công thức: Having + P2, S+Ved. Đáp án đúng là A. Tạm dịch: Sau khi giành được danh hiệu quan trọng trong môn quần vợt , Alan đã rời khỏi các cuộc thi đấu quốc tê .
  • 65. Question 15. The harder he studies, ______ results he gets. A. the best results he gets B. his results get better C. the better do his results get D. the better Kiến thức: So sánh kép Giải thích: Ta có công thức so sánh kép: The comparative + S1 + V1, the comparative + S2 + V2: càng …., càng …. Xét các đáp án: A. the best results he gets: sai B. his results get better : sai C. the better do his results get: sai D. the better : đúng Tạm dịch: Anh ấy càng học hành chăm chỉ, anh ấy càng đạt được kết quả tốt.
  • 66. Question 16. The hotel receptionist is asking the customer his name. Receptionist: “Could you please say your name again please?” Customer: “__________” A. How can you say so? B. Why do I have to? C. No, I don’t remember. D. Sure, that’s Robert. Kiến thức: Tình huống giao tiếp (Đáp lại lời đề nghị) Giải thích: Nhân viên lễ tân khách sạn đang hỏi khách hàng về tên của anh ta. - Nhân viên lễ tân: " Ông/anh làm ơn có thể nói lại tên của mình được không?" - Khách hàng: "_______." Xét các đáp án: A. Sao ông có thể nói thế B. Sao tôi phải làm vậy C. Không, tôi không nhớ D. Chắc chắn rồi, đó là Robert
  • 67. Question 17. Jolie is reading a magazine on famous people, asking her friend. Jolie: “Do you think celebrities today tend to focus more on wealth rather than achievements?” Jolie's friend: “__________________. And this sets bad examples for young people.” A. Not at all B. You can say that again C. I think they focus more on achievements D. It's out of the question Kiến thức: Tình huống giao tiếp (Hỏi về quan điểm của người khác) Giải thích: Jolie đang đọc một cuốn tạp chí về người nổi tiếng, hỏi bạn cô ấy. - Jolie: "Bạn có nghĩ những người nổi tiếng ngày nay có xu hướng tập trung vào tiền bạc hơn về những thành tích không?." - Bạn của Jolie: "_______, và điều này tạo ra những tấm gương xấu cho các bạn trẻ" Xét các đáp án: A. Hoàn toàn không. B. Tôi đồng ý. C. Tôi nghĩ họ tập trung nhiều hơn về những thành tích D. Nó là vấn đề không thể bàn đến
  • 68. Question 18. A. publicity B. chemistry C. attention D. romantic Kiến thức: Trọng âm của từ 3 âm tiết trở lên Giải thích: A. publicity /pʌbˈlɪsəti/ (n): Trọng âm rơi vào âm tiết thứ hai vì đuôi –ITY – trọng âm rơi vào âm tiết liền trước. B. chemistry /ˈkemɪstri/ (n): Trọng âm rơi vào âm tiết thứ nhất C. attention /əˈtenʃn/ (n): Trọng âm rơi vào âm tiết thứ hai vì đuôi – TION- trong âm rơi vào âm liền trước D. romantic /rəʊˈmæntɪk/ (a): Trọng âm rơi vào âm tiết thứ hai vì đuôi – IC - trong âm rơi vào âm liền trước
  • 69. Question 19. A. attract B. discuss C. follow D. confide Kiến thức: Trọng âm của từ 2 âm tiết Giải thích: - Đáp án C có trọng âm rơi vào âm tiết thứ 1. Các đáp án còn lại có trọng âm rơi vào âm tiết thứ 2. A. attract /əˈtrækt/ (v): B. discus /dɪˈskʌs/ (v): C. follow /dɪˈskʌs/ (v): D. confide /kənˈfaɪd/ (v): * Note - Động từ 2 âm tiết – trọng âm thường rơi vào âm tiết thứ hai. Trừ những động từ có tận cùng là : er/ ow / en / ish trong âm rơi vào âm tiết số 1
  • 70. Question 20. A. accessed B. searched C. recorded D. developed Question 20. Đáp án C Kiến thức : Cách phát âm đuôi -ED Giải thích: – Phát âm là /t/ khi từ có tận cùng bằng các phụ âm vô thanh: /θ/, /p/, /k/, /f/, /s/, /ʃ/, /tʃ/ – Phát âm là /id/ khi từ có tận cùng là các âm: /t/, /d/ – Phát âm là /d/ khi các từ có tận cùng là nguyên âm và các phụ âm hữu thanh còn lại Xét các đáp án: A. accessed B. searched C. recorded D. developed
  • 71. Question 21. A. mouth B. founder C. around D. country Kiến thức: Cách phát âm của nguyên âm Giải thích: A. mouth /maʊθ/ B. founder /ˈfaʊndə(r)/ C. around /əˈraʊnd/ D. country /ˈkʌntri/ Ta thấy: Các phương án A, B, C – phần gạch chân được phát âm là /aʊ/ Phương án D – phần gạch chân được phát âm là /ʌ/
  • 72. Question 22. Some students only cram for tests when there is little time left, so their results are not satisfactory. A. prepare in a short period B. prepare in a long time C. prepare well D. prepare badly Kiến thức: Đồng nghĩa (từ đơn) Giải thích: cram (v): học gạo ( học trong khoảng thời gian ngắn để thi) Xét các đáp án: A. prepare in a short period: chuẩn bị trong thời gian ngắn B. prepare in a long time: chuẩn bị trong thời gian dài C. prepare well: chuẩn bị tốt D. prepare badly: chuẩn bị không tốt
  • 73. Question 23. Please, you are so nervous, do try to contain your anger. A. hold back B. consult C. consume D. contact Kiến thức: Đồng nghĩa (từ đơn) Giải thích: Contain (v) : chứa đựng/ kiềm chế/ ngăn lại Xét các đáp án: A. hold back: ngăn lại/ giữ lại/kìm lại B. consult(v) hỏi ý kiến/ tham khảo C. consume(v) tiêu thụ/ tiêu dùng D. contact (v) liên lạc Vậy đáp án đúng là A Tạm dịch: Bạn đang rất căng thẳng , làm ơn cố gắng kiềm chế cơn tức giận.
  • 74. Question 24 The principle of use and disuse states that those parts of organisms' bodies that are used grown larger while those parts that are not tend to wither away. A. split B. swell C. perish D. shrink Kiến thức: Trái nghĩa (cụm từ hoặc thành ngữ) Giải thích: Ta có: - wither away: trở nên yếu đi/ nhỏ lại Xét các đáp án: A. split (v) chia/ tách ra B. swell (v) phồng lên/ to lên C. perish (v) diệt vong/ bỏ mạng D. shrink (v) nhỏ lại Vậy đáp án đúng là B Wither away >< swell
  • 75. Question 25. He is from such an unemotional family, he will never learn to unleash his feelings. A. describe B. conceal C. release D. extend Kiến thức: Trái nghĩa (từ đơn) Giải thích: Ta có: unleash (v): buông lỏng/ giải phóng/ thể hiện ( cảm xúc). Xét các đáp án: A. describe (v) miêu tả B. conceal (v) che dấu C. release (v) thả/ phóng thích/làm nhẹ bớt D. extend (v) mở rộng
  • 76. Question 26. My eldest sister graduated from university. She soon started working as a freelance journalist. A. Hardly had my eldest started working as a freelance journalist when she graduated from university. B. No sooner had my eldest sister graduated from university than she started working as a freelance journalist. C. No sooner had my eldest sister started working as a freelance journalist than she graduated from university. D. After my eldest sister graduated from university, she had started working as a freelance journalist. Kiến thức: Kết hợp câu – đảo ngữ Giải thích: Ta có công thức đảo ngữ: No sooner + had + S + PII than + S + Ved Câu đề bài: Chị gái tôi tốt nghiệp đại học. Chị ấy sớm bắt đầu làm một nhà báo tự do.
  • 77. Question 27. You didn’t give me a chance to tell you the truth. I regret that. A. If you gave me a chance, I could tell you the truth. B. I wish you had given me a chance to tell you the truth. C. I wish you didn’t give me a chance to tell you the truth. D. If only you hadn’t given me a chance to tell you the truth. Kiến thức: Kết hợp câu – câu điều ước Giải thích: Giải thích: Câu đề bài: Bạn không cho tôi cơ hội nói cho bạn biết sự thật. Tôi thấy tiếc vì điều đó Ta thấy câu này diễn đạt hành động trong quá khứ, nên cần có đáp án diễn tả điều ước ở quá khứ A. Nếu bạn cho tôi cơ hội, tôi đã có thể nói cho bạn sự thật.=> sai cấu trúc ( câu điều kiện loại 2- hiện tại) B. Tôi ước bạn đã cho tôi cơ hội để nói sự thật => đúng ( câu ước ở quá khứ) C. Tôi ước bạn đã không cho tôi cơ hội để nói sự thật.=> sai nghĩa + sai cấu trúc ( câu ước ở hiện tại) D. Tôi ước bạn đã không cho tôi cơ hội để nói sự thật. => sai nghĩa
  • 78. Question 28. We have a very happy party at Tom’s house last weekend. A. have B. very C. party D. Tom’s Kiến thức: Lỗi sai – Thì của động từ Giải thích: Ta thấy thời gian: last weekend nên động từ phải chia ở thì quá khứ. Vậy đáp án đúng là A Sửa lỗi: have → had Tạm dịch: Chúng tôi đã có một bữa tiệc rất vui tại nhà Tom cuối tuần trước.
  • 79. Question 29. Everyone has its own ideas about the best way to bring up children. A. its B. about C. way D. children Kiến thức: Lỗi sai – Đại từ nhân xưng/ tính từ sở hữu Giải thích: Ta thấy Everyone là đại từ bất định chỉ người nên tính từ sở hữu tương ứng là their hoặc his/her. Vậy đáp án đúng là A Sửa lỗi: its → their, his/her Tạm dịch: Mỗi người có ý tưởng riêng của mình về cách tốt nhất để nuôi nấng con cái.
  • 80. Question 30. Young people of an impressive age can easily be manipulated. A. Yong B. impressive C. easily D. manipulated Kiến thức: Lỗi sai – Từ vựng Giải thích: impressive (a) = gây ấn tượng sâu sắc/gây xúc động ; impressionable (a) = nhạy cảm/ dễ bị ảnh hưởng Sửa lỗi: impressive → impressionable Tạm dịch: Các bạn trẻ đang ở độ tuổi nhạy cảm có thể dễ dàng bị dụ dỗ/ lôi kéo.
  • 81. Question 31. I am sure he said something terrible to her because of her red eyes. A. He must have said something terrible to her because I could see it from her red eyes B. He could have said something terrible to her because I could see it from her red eyes C. He should have said something terrible to her because I could see it from her red eyes D. He may have said something terrible to her because I could see it from her red eyes Kiến thức: Câu đồng nghĩa – Động từ khuyết thiếu Giải thích: Sử dụng cấu trúc “động từ khuyết thiếu + (not) + have + Vp2” để đưa ra dự đoán, lời khuyên,… cho các hành động trong quá khứ. Câu đề bài: Tôi chắc chắn là anh ta đã nói điều gì đó rất tệ với cô ta vì mắt cô ấy rất đỏ. Các phương án: A. Anh ta chắc chắn đã nói điều gì đó rất tệ với cô ta bởi vì tôi có thể thấy điều đó từ đôi mắt đỏ của cô ta. B. Anh ta lẽ ra đã có thể nói điều gì đó rất tệ với cô ta bởi vì tôi có thể thấy điều đó từ đôi mắt đỏ của cô ta. C. Anh ta lẽ ra đã nên nói điều gì đó rất tệ với cô ta bởi vì tôi có thể thấy điều đó từ đôi mắt đỏ của cô ta. D. Anh ta có lẽ là đã nói điều gì đó rất tệ với cô ta bởi vì tôi có thể thấy điều đó từ đôi mắt đỏ của cô ta. Phù hợp với nghĩa câu gốc nhất là đáp án A.
  • 82. Question 32. “What have you done to your hair?” she said to her son. A. She asked her son what to do to his hair. B. She wanted her son to know what he had done to his hair. C. She wanted to know what did her son do to his hair. D. She asked her son what he had done to his hair. Kiến thức: Câu đồng nghĩa – Câu tường thuật Giải thích: Đề bài: "Con đã làm gì với tóc của con vậy", Cô ta nói với con trai mình. Ta nhận thấy đây là một câu hỏi ở thì hiện tại hoàn thành - lùi thì về quá khứ hoàn thành A. Cô ta hỏi con trai phải làm gì với mái tóc của nó. => sai nghĩa B. Cô ta muốn con trai biết nó đã làm gì với mái tóc của nó => Sai nghĩa C. Cô ta muốn biết con trai cô ta đã làm gì với mái tóc của nó.=> Đáp án này sai vì chưa lùi thì và đảo ngữ D. Cô ta muốn biết con trai cô ta đã làm gì với mái tóc của nó. => Đáp án này đúng.
  • 83. Question 33. It’s almost nine months since I last subscribed to that magazine. I haven’t subscribed to that magazine for almost nine months. I have subscribed to that magazine almost nine months ago. I subscribed to that magazine almost for nine months . I have subscribed to that magazine for almost nine months, but now I stopped. Kiến thức: Câu đồng nghĩa – Thì của động từ Giải thích: Câu đề bài: Gần chín tháng kể từ lần cuối cùng tôi đăng ký tạp chí đó. = A. Tôi không đăng ký tạp chí đó được gần chín tháng rồi. Cấu trúc: S + haven’t/ hasn’t + Vp2 + for + time = It ‘s + time + since + S + (last) + Vp1 + .
  • 84. Question 34: A. making B. providing C. allowing D. holding Countries all over the world are now beginning to enter the sector of eLearning, (34) ______ people to have a broader access to learning opportunities that weren't otherwise possible in the past. Kiến thức: Đọc điền từ Giải thích: A. make sb + Vnt: bảo ai làm gì B. provide sb with st: cung cấp cho ai cái gì C. allow sb to + Vnt: cho phép ai làm gì D. hold sb/ st: cầm / nắm … Ta thấy chỗ trống cần một động từ + tân ngữ + to Vnt
  • 85. Question 35: A. who B. whom C. when D. why The reason why this industry has become so popular over the last few years is the convenience it offers to those (35) ______ are interested in it. Kiến thức: Đọc điền từ Giải thích: A. who: Thay thế cho danh từ người có chức năng làm chủ ngữ và tân ngữ trong mệnh đề quan hệ. B. whom: Thay thế cho danh từ người có chức năng làm tân ngữ trong mệnh đề quan hệ. C. when: Thay thế cho trạng từ chỉ thời gian có chức năng làm bổ ngữ trong mệnh đề quan hệ C. why: Thay thế cho trạng từ chỉ lý do có chức năng làm bổ ngữ trong mệnh đề quan hệ Ta có cấu trúc : those = those people => cần đại từ quan hệ thay thế cho người làm chủ ngữ
  • 86. Question 36: A. constitution B. institution C. restitution D. destitution Traditionally, if a person wanted to learn something or that can teach you to get a degree, they would have to go to a university, a college, or any (36) ______ giving you a certification at the end. Kiến thức: Từ vựng Giải thích: A. constitution (n) hiến pháp B. institution (n) cơ sở giáo dục C. restitution (n) sự hoàn lại D. destitution (n) sự cơ cự , thiếu thốn Tạm dịch: Theo truyền thống, nếu một người nào đó muốn học một thứ gì đó hoặc có thể dạy bạn để lấy bằng, họ phải học đại học , cao đẳng hoặc bất kỳ một cơ sở giáo dục nào mà cấp chứng chỉ cho bạn.
  • 87. Question 37: A. Because B. However C. Therefore D. Even though (37)_________there are several benefits to developing eLearning sectors in countries, there are still a few nations that are moving relatively backward in this industry. Kiến thức: Đọc điền từ Giải thích: Xét các đáp án: A. Because: bởi vì B. However: tuy nhiên C. Therefore: cho nên D. Eventhough: mặc dù Tạm dịch: Mặc dù có một số lợi ích đối với việc phát triển các khu vực eLearning ở các quốc gia, nhưng vẫn còn một vài quốc gia đang khá là lạc hậu đối với nghành công nghiệp này
  • 88. Question 38: A. each B. others C. every D. another There are, of course, countries that have already gone far ahead of (38)_________, setting up more and more eLearning platforms. Kiến thức: Đọc điền từ Giải thích: Xét các đáp án: A. each (+ N –số ít ) : mỗi một B. others : những người khác C. every ( + N-số ít) : mọi/ mỗi…. D. another ( + N-số ít) : một cái/người khác nữa Tạm dịch: Tất nhiên có những quốc gia vượt xa các quốc gia khác, thiết lập càng nhiều nền tảng eLearning
  • 89. Question 39. What could be the best title for the passage? A. legal limit for driving B. driving ability C. drinking and driving D. standard of drinks Kiến thức: Đọc hiểu Giải thích: Câu nào có thể là tiêu đề hay nhất cho đoạn văn. A. Giới hạn hợp pháp cho việc lái xe B. Khả năng lái xe C. Uống rượu và lái xeD. Tiêu chuẩn của việc uống rượu Bài viết nói về mối liên quan giữa việc uống rượu và lái xe
  • 90. Question 40. The word “breathalyzer” in paragraph 5 means ______ . A. a device used by the police to measure the amount of alcohol in a driver’s breath. B. a device used to analyse driver’s blood type. C. a person who measures the amount of alcohol in a driver’s breath. D. a person who has to take a breath test Kiến thức: Đọc hiểu Giải thích: Từ “ breathalyser” trong đoạn 5 có nghĩa là: A. một thiết bị được sử dụng bởi cảnh sát để đo nồng độ cồn trong hơi thở của người lái xe B. một thiết bị được sử dụng để phân tích nhóm máu của người lái xe C. một người mà đo nồng độ cồn trong hơi thở của người lái xe D. một người mà phải kiểm tra hơi thở -“ breathalyser”: dụng cụ kiểm tra lượng rượu trong hơi thở.
  • 91. Question 41. The word “they” in paragraph 4 refers to ______ . A. professional drivers B. moveable posts C. alcoholic drinks D. tests Kiến thức: Đọc hiểu Giải thích: Từ “they” trong đoạn 4 là chỉ: A. Những người lái xe chuyên nghiệp B. Những cột mốc có thể di chuyển được C. Những đồ uống có cồn D. những bài kiểm tra Thông tin: “In a test with professional drivers, the more alcoholic drinks they had had, the more certain they were that they could drive a test course through a set of moveable posts … and the less able they were to do it!
  • 92. Question 42. According to the passage, which of the following is TRUE about the amount of alcohol a person can drink before reaching the legal limit ? A. 800 milligrams of pure alcohol B. approximately three standard drinks C. exactly proportional to body weight D. different for different people Kiến thức: Đọc hiểu Giải thích: Lượng rượu mà một người có thể uống trước khi đạt đến giới hạn pháp lý là . A. 800 mg rượu nguyên chất B. tỷ lệ chính xác với trọng lượng cơ thể C. khoảng ba ly thông thường D. khác nhau với những người khác nhau Thông tin: But there is no sure way of telling how much you can drink before you reach this limit. It varies with each person depending on your weight, your sex, if you‟ve just eaten and what sort of drinks you‟ve had.
  • 93. Question 43. When might you be taken to court by the police for drinking and driving? A. When the police think that you have been drinking from the way you are driving. B. When you have driven a vehicle after drinking any alcohol at all. C. Only when tests show that you have 80mg of alcohol in 100 ml of blood. D. When you have drunk at least three drinks before driving. Kiến thức: Đọc hiểu Giải thích: Khi nào bạn có thể bị cảnh sát đưa ra tòa vì uống rượu và lái xe? A. Khi cảnh sát nhận ra rằng bạn đã uống rượu từ cách bạn lái xe. B. Chỉ khi xét nghiệm cho thấy bạn có 80mg cồn trong 100 ml máu. C. Khi bạn lái xe sau khi uống bất kỳ chút đồ uống có cồn nào. D. Khi bạn đã uống ít nhất ba ly trước khi lái xe. Thông tin: Even if you‟re below the legal limit, you could still be taken to court if a police officer thinks your driving has been affected by alcohol.
  • 94. Question 44. Alternative forms of health care are becoming more popular because _______. A. they have been used for a very long time B. they are able to cure patients much more quickly C. other ways aren't able to get rid of the symptoms as well as them D. medical costs are on the upswing in many developed countries Kiến thức: Đọc hiểu Giải thích: Các hình thức chăm sóc sức khỏe thay thế đang trở nên phổ biến hơn vì _______. A. chúng đã được sử dụng trong một thời gian rất dài B. chúng có thể cứu chữa bệnh nhân nhanh hơn nhiều C. những cách khác không thể loại bỏ các triệu chứng cũng như chúng D. chi phí y tế đang tăng lên ở nhiều nước phát triển Thông tin: As health care costs continually rise in most developed countries, many people are looking for alternative forms of health therapy. Two of the most popular forms of alternative therapies in the West today are acupuncture and herbal medicine,…
  • 95. Question 45. The word "crux" in paragraph 1 mostly means _______. A. basis B. relief C. symptom D. application Kiến thức: Đọc hiểu – từ vựng Giải thích: Từ “crux” trong đoạn 1 thì hầu hết có nghĩa là ________. A. basis: cơ sở, căn cứ B. relief: cứu trợ C. symptom: triệu chứng D. application: ứng dụng => crux (n): mấu chốt, điểm then chốt = basis
  • 96. Question 46. The word "solely" in paragraph 3 is CLOSEST in meaning to _______. A. safely B. barely C. intentionally D. exclusively Kiến thức: Đọc hiểu – từ vựng Giải thích: Từ “solely” trong đoạn 3 thì gần nghĩa nhất với ______. A. safely (adv): an toàn B. barely (adv): vừa đủ C. intentionally (adv): cố ý D. exclusively (adv): duy nhất (for only one particular person, group or use) => solely (adv): only; not involving somebody/something else (chỉ ….) = exclusively Thông tin: This technique is involved solely with the ear and its numerous activation points. Tạm dịch: Kỹ thuật này chỉ liên quan đến tai và nhiều điểm kích hoạt của nó.
  • 97. Question 47. It can be inferred from paragraph 3 that the ear _______. A. has a healthy blood supply, which relieves the pain from needles B. contains a small number of the body's special activation points C. is a beneficial acupuncture area because it contains a myriad of nerves D. is the source of dependency issues such as alcoholism and drug use Kiến thức: Đọc hiểu – suy luận Giải thích: Có thể suy ra từ đoạn 3 rằng tai _______. A. có nguồn cung cấp máu lành mạnh, giúp giảm đau do kim tiêm B. chứa một số lượng nhỏ các điểm kích hoạt đặc biệt của cơ thể C. là vùng huyệt đạo có lợi vì chứa vô số dây thần kinh D. là nguồn gốc của các vấn đề phụ thuộc như nghiện rượu và sử dụng ma túy Thông tin: This technique is involved solely with the ear and its numerous activation points. The ear is considered a highly sensitive area of the human body because it contains a strong, healthy flow of blood as well as numerous nerve points that connect it with the rest of the human body. Acupuncturists place tiny needles in the surface of the skin at certain points, depending on the desired result. In general, ear acupuncture is considered an excellent therapy, especially for treating individuals with psychological or physical dependency issues such as eating disorders and drug or alcohol abuse.
  • 98. Question 48. The word "it" in paragraph 3 refers to _______. A. the ear B. the activation point C. the nerve point D. the flow of blood Kiến thức: Đọc hiểu – từ thay thế Giải thích: Từ “it” trong đoạn 3 đề cập đến _______. A. tai B. 1 điểm kích hoạt C. 1 điểm thần kinh D. dòng chảy của máu Thông tin: The ear is considered a highly sensitive area of the human body because it contains a strong, healthy flow of blood as well as numerous nerve points that connect it with the rest of the human body. Tạm dịch: Tai được coi là khu vực nhạy cảm cao của cơ thể con người vì nó (tai) chứa một dòng chảy của máu lưu thông mạnh mẽ, khỏe mạnh cũng như nhiều điểm thần kinh mà kết nối nó (tai) với phần còn lại của cơ thể con người.
  • 99. Question 49. The author's description of herbal therapy mentions all of the following EXCEPT _______. A. the reaction time is faster for acupuncture than for herbal therapy B. herbs are an excellent way of increasing a person's endurance C. it is used as a preventative form of therapy for healthy people D. it is beneficial to the circulatory system and blood pressure problems Kiến thức: Đọc hiểu – chi tiết Giải thích: Mô tả của tác giả về liệu pháp thảo dược đề cập đến tất cả những điều sau NGOẠI TRỪ _______. A. thời gian phản ứng đối với châm cứu nhanh hơn so với liệu pháp thảo dược B. các loại thảo mộc là một cách tuyệt vời để tăng sức bền của một người C. nó được sử dụng như một hình thức trị liệu phòng ngừa cho những người khỏe mạnh D. nó có lợi cho hệ tuần hoàn và các vấn đề về huyết áp Thông tin: - Traditionally, herbal teas are imbibed to boost the immune system and prevent illnesses from being able to enter the body. - Herbs also have a more direct influence than acupuncture on the body's physical system, as they can aid in normalizing blood pressure. - Other benefits attributed to the use of traditional Chinese herbs are that they build stamina and are good for digestive purposes.
  • 100. Question 50. What is the passage mainly about? A. Acupuncture – the best therapy. B. Using plants for healing the injuries. C. Two forms of alternative therapies in the West. D. Western medical techniques. Kiến thức: Đọc hiểu – chi tiết Giải thích: Đoạn văn này chủ yếu nói về điều gì? A. Châm cứu – liệu pháp chữa bệnh tốt nhất B. Việc sử dụng thảo dược để chữa trị các vết thương. C. Hai liệu pháp chữa bệnh thay thế ở phương Tây D. Kỹ thuật y học ở phương tây Thông tin: “ As health care costs continually rise in most developed countries, many people are looking for alternative forms of health therapy. Two of the most popular forms of alternative therapies in the West today are acupuncture and herbal medicine, both of which have been used for centuries in Asia and are rooted in ancient Chinese medicine.”
  • 101.
  • 102.
  • 103. Question 1. If you don't have anything _______ to say, it's better to say nothing. A. construct B. construction C. constructive D. constructor Kiến thức: Từ loại Giải thích: Chỗ trống cần 1 tính từ vì tính từ đứng sau các đại từ bất định như : something, anything, somebody, anybody … Xét các đáp án : A. construct /kənˈstrʌkt/ (v): xây dựng B. construction /kənˈstrʌkʃn/ (n): sự xây dựng C. constructive /kənˈstrʌktɪv/ (a): có tính xây dựng D. constructor /kənˈstrʌktə(r)/ (n): kỹ sư xây dựng
  • 104. Question 2. Neither of the boys came to school yesterday, _______? A. didn’t they B. does they C. did they D. doesn’t they Kiến thức: Câu hỏi đuôi Giải thích: Ta có công thức của câu hỏi đuôi: S + V + O..., TRỢ ĐỘNG TỪ + ĐẠI TỪ? Chủ ngữ của mệnh đề chính là “the boys” nên đại từ ở câu hỏi đuôi phải chuyển thành “they” Động từ “came” được chia ở thì quá khứ đơn thì trợ động từ ở câu hỏi đuôi là “did” → loại B và D Tuy nhiên trong mệnh đề chính không xuất hiện từ not mang nghĩa phủ định nhưng có một từ mang nghĩa phủ định đó là Neither (không cái nào/người nào trong hai) → trợ động từ câu hỏi đuôi phải ở dạng khẳng định → loại A và chọn được C. * Note: Với những câu hỏi về câu hỏi đuôi nếu mệnh đề chính không có NOT nhưng xuất hiện các từ mang nghĩa phủ định như never, little, seldom, neither, hardly, scarely, ... thì trợ động từ câu hỏi đuôi ở dạng khẳng định.
  • 105. Question 3. It's a little _______ that no one knows where he was at the time of the murder. A. doubtful B. suspicious C. skeptical D. dubious Kiến thức: Từ vựng Giải thích: It's a little ______ that no one knows where he was at the time of the murder. A. doubtful B. suspicious C. skeptical D. Dubious
  • 106. Question 4. Television can make things memorable for the reason that it presents information _______ an effective way. A. over B. with C. in D. on Kiến thức: Giới từ Giải thích: Khi muốn nói theo một cách nào đó ta dùng cụm giới từ in ....way. Cụ thể trong câu này là theo một cách hiệu quả. Các phương án còn lại không dùng với nghĩa này nên đáp án là C. In an effective way: một cachs hiệu quả Tạm dịch: Truyền hình có thể khiến mọi thứ trở nên đáng nhớ vì nó trình bày thông tin theo một cách hiệu quả.
  • 107. Question 5. Someone stopped me on the street and offered to sell me a gold watch for five dollars. I could _______ a rat. A. taste B. see C. think D. smell Kiến thức: Thành ngữ Giải thích: Ta có thành ngữ: smell a rat: tỏ ý nghi ngờ có một điều gì đó Vậy đáp án đúng là D Tạm dịch: Ai đó đã chặn tôi lại trên phố và đề nghị bán cho tôi một chiếc đồng hồ vàng với giá năm đô la. Tôi có thể nghi ngờ có gì đó sai sai.
  • 108. Question 6. All activities have stopped _______ he weather has been terrible since yesterday. A. although B. because of C. because D. despite Kiến thức: Liên từ Giải thích: Xét các đáp án: A. Although (+ clause): mặc dù B. Because of (+ V-ing): bởi vì C. Because (+ clause): bởi vì D. Despite (+ V-ing/ N.P): mặc dù Dựa vào nghĩa, đáp án đúng là C Tạm dịch: Mọi hoạt động đã dừng lại vì thời tiết từ hôm qua trở nên kinh khủng.
  • 109. Question 7. Her husband bought her a _______ scarf when he went on holiday in Singapore last week. A. beautiful silk yellow B. yellow beautiful silk C. beautiful yellow silk D. silk beautiful yellow Kiến thức: Trật tự tính từ Giải thích: Vị trí của tính từ trước danh từ: Opinion (quan điểm) – beautiful + Color (màu sắc) – yellow + Material (chất liệu) – silk + Noun (danh từ) Vậy đáp án đúng là C Tạm dịch: Chồng cô đã mua cho cô chiếc khăn lụa màu vàng tuyệt đẹp khi anh đi nghỉ ở Singapore tuần trước.
  • 110. Question 8. There are other problems of city life which I don't propose to _______ at the moment. A. go into B. go around C. go for D. go up Kiến thức: Cụm động từ Giải thích: A. go into (phr.v): bắt đầu làm gì; thảo luận, kiểm chứng, mô tả, giải thích một cách chi tiết và thận trọng B. go around (phr.v): đủ cho mọi người trong nhóm; đến thăm ai; cư xử tệ C. go for (phr.v): chọn, say mê, cố gắng D. go up (phr.v): tăng lên Tạm dịch: Còn nhiều vấn đề khác về cuộc sống thành thị cái mà tôi không muốn đi vào sâu để thảo luận kỹ ngay lúc này.
  • 111. Question 9. When Carol called last night, I _______ my favorite show on television. A. was watching B. watched C. have watched D. watch Kiến thức: Thì quá khứ tiếp diễn Giải thích: Trong quá khứ, một sự việc đang diễn ra thì sự việc khác xen vào: + Sự việc đang diễn ra chia ở thì quá khứ tiếp diễn: S + was/were + V.ing => was watching + Sự việc xen vào chia ở thì quá khứ đơn: S + V.ed => called Tạm dịch: Tối qua, khi Carol gọi thì tôi đang xem chương trình yêu thích của mình trên tivi.
  • 112. Question 10. _______, I will remind him to call for you. A. when he was turning up B. until he turned up C. after had turned up D. When he turns up Kiến thức: Sự phối hợp các thì (Tương lai & hiện tại) Giải thích: Ta thấy, động từ của mệnh đề chính ở tương lai đơn nên động từ của mệnh đề trạng ngữ chỉ thời gian chia ở hiện tại. Vậy, đáp án D đúng. Tạm dịch: Khi anh ấy đến, tôi sẽ nhắc anh ấy gọi cho bạn.
  • 113. Question 11. Researchers have to _______ the conclusion that your personality is affected by your genes. A. arrived B. come C. got D. reached Kiến thức: Cụm từ cố định Giải thích: Ta có cụm từ cố định: Come to conclusion: đưa ra kết luận Vậy đáp án đúng là B Tạm dịch: Các nhà nghiên cứu phải đưa ra kết luận rằng tính cách của bạn bị ảnh hưởng bởi gen của bạn.
  • 114. Question 12. Closer _______ of the documents revealed a number of interesting and important facts. A. scrutiny B. inception C. peculiarity D. Suspicion Kiến thức: Từ vựng Giải thích: Xét các đáp án : A. scrutiny : xem xét kỹ lưỡng B. inception : khởi đầu C. peculiarity : đặc biệt D. suspicion : nghi ngờ Ta có cụm từ : close scrutiny : xem xét kĩ lưỡng Vậy đáp án đúng là A Tạm dịch: Việc xem xét kỹ hơn các tài liệu đã tiết lộ một số sự kiện thú vị và quan trọng.
  • 115. Question 13. It is claimed that new nuclear power plants designed to be safer than the current ones _______. A. will build B. build C. are soon built D. built Kiến thức: Câu bị động Giải thích: Căn cứ vào nghĩa, “new nuclear power plants” - những nhà máy điện nguyên tử thì động từ phải chia ở thể bị động. Do đó loại A,B, D Vậy đáp án đúng: C Tạm dịch: Người ta cho rằng các nhà máy điện hạt nhân mới được thiết kế để an toàn hơn các nhà máy hiện tại sẽ sớm được xây dựng.
  • 116. Question 14. _______ water resources irresponsibly, the authorities had to deal with water shortages in the region. A. Managed B. Having managed C. To manage D. Being managed Kiến thức : Rút gọn mệnh đề trạng ngữ Giải thích: Khi hai mệnh đề cùng chủ ngữ, và câu muốn nhấn mạnh hành động phía trước được hoàn thành xong trước rồi hành động phía sau mới xảy ra thì chúng ta dùng công thức: Having + V-p2, S+Ved. Đáp án đúng là B. Tạm dịch: Quản lý tài nguyên nước thiếu trách nhiệm, các cơ quan chức năng đã phải đối mặt với tình trạng thiếu nước trong khu vực.
  • 117. Question 15. The harder you study for these exams, _______ you will do. A. the better B. the much C. the best D. the more good Kiến thức: So sánh kép Giải thích: Xét các đáp án: A. the better → cấu trúc so sánh kép: The more/ -er + S + V, the more/ -er + S + V B. the much → sai cấu trúc so sánh kép C. the best → sai cấu trúc so sánh kép D. the more good → sai cấu trúc so sánh hơn (hard là từ 1 âm tiết, good – bất quy tắc) Đáp án đúng là A Tạm dịch: Bạn càng học chăm chỉ cho những bài thi này, bạn làm bài càng tốt.
  • 118. Question 16. John was in Hanoi and wanted to send a parcel to his parents. He asked a local passer-by the way to the post-office. - John: "Can you show me the way to the nearest post office, please?" - Passer-by: "_______" A. Look it up in a dictionary! B. Just round the corner over there. C. There's no traffic near here. D. Not way, sorry. Kiến thức: Tình huống giao tiếp (Đáp lại lời đề nghị) Giải thích: Tình huống giao tiếp: John đang ở Hà Nội và muốn gửi một bưu kiện cho bố mẹ. Anh hỏi đường đến bưu điện một người qua đường địa phương. - John: "Bạn có thể chỉ cho tôi đường đến bưu điện gần nhất được không?" - Người qua đường: "_______" Xét các đáp án: A. Tra từ điển! B. Chỉ quanh góc đằng kia. C. Gần đây không có giao thông. D. Không được, xin lỗi. Dựa vào nghĩa, đáp án đúng là B
  • 119. Question 17. Ann was thanking Kate for coming to dinner. - Ann: “Thank you for taking the time to come here in person.” - Kate: “_______” A. I’d love to come. What time? B. I haven’t got a clue. C. It’s my pleasure. D. That’s all for now. Kiến thức: Tình huống giao tiếp (Đáp lại lời cảm ơn) Giải thích: Tình huống giao tiếp: Ann cảm ơn Kate vì đã đến ăn tối. - Ann: "Cảm ơn bạn đã dành thời gian đến đây trực tiếp." - Kate: ‘_______’ Xét các đáp án: A. Tôi rất muốn đến. Mấy giờ? B. Tôi không có manh mối. C. Đó là niềm vui của tôi. D. Đó là tất cả cho bây giờ.
  • 120. Question 18. A. survival B. industry C. endanger D. commercial Kiến thức: Trọng âm của từ 3 âm tiết Giải thích: A. survival / səˈvaɪvl / (n): Trọng âm rơi vào âm tiết thứ 2 vì từ gốc là động từ và hậu tố không thay đổi trọng âm chính. B. industry / ˈɪndəstri / (n): Trọng âm rơi vào âm tiết thứ nhất vì đuôi –try trọng âm rơi vào âm tiết thứ 3 tính từ cuối. C. endanger / ɪnˈdeɪndʒ/ (adj): Trọng âm rơi vào âm tiết thứ 2 vì từ gốc là danh từ và tiền tố không thay đổi trọng âm chính. D. commercial / kəˈmɜːʃl / (adj): Trọng âm rơi vào âm tiết thứ 2 vì đuôi –cial trọng âm rơi vào âm tiết trước nó. Vậy đáp án đúng là B
  • 121. Question 19. A. cartoon B. answer C. reason D. paper Kiến thức: Trọng âm của từ 2 âm tiết Giải thích: A. cartoon /kɑːˈtuːn/: Từ này trọng âm rơi vào âm tiết thứ hai. Vì theo quy tắc các từ tận cùng bằng đuôi -oon trọng âm nhấn ở chính đuôi này. B. answer /ˈɑːnsə(r)/: Từ này trọng âm rơi vào âm tiết thứ nhất. Vì theo quy tắc trọng âm không rơi vào âm /ə/. C. reason / 'ri:zn /: Từ này trọng âm rơi vào âm tiết thứ nhất. Vì theo quy tắc trọng âm, thì danh từ 2âm tiết trọng âm thường rơi vào âm tiết thứ nhất. D. paper /ˈpeɪpə(r)/: Từ này trọng âm rơi vào âm tiết thứ nhất. Vì theo quy tắc trọng âm không rơi vào âm /ə/. => Phương án A trọng âm rơi vào âm tiết thứ hai, các phương án còn lại trọng âm rơi vào âm tiết thứ nhất.
  • 122. Question 20. A. visited B. played C. hated D. needed Kiến thức : Cách phát âm đuôi -ED Giải thích: – Phát âm là /t/ khi từ có tận cùng bằng các phụ âm vô thanh: /θ/, /p/, /k/, /f/, /s/, /ʃ/, /tʃ/ – Phát âm là /id/ khi từ có tận cùng là các âm: /t/, /d/ – Phát âm là /d/ khi các từ có tận cùng là nguyên âm và các phụ âm hữu thanh còn lại
  • 123. Question 21. A. hand B. save C. take D. face Kiến thức: Cách phát âm của nguyên âm Giải thích: Xét các đáp án: A. hand /hӕnd/ B. save /seiv/ C. take /teik) D. face /feis/
  • 124. Question 22. Emissions from factories and exhaust fumes from vehicles can have detrimental effects on our health. A. beneficial B. neutral C. needy D. harmful Kiến thức: Đồng nghĩa (từ đơn) Giải thích: Xét các đáp án: - beneficial (adj): có lợi - neutral (adj): trung tính, trung lập - needy (adj): nghèo túng, nghèo đói - harmful (adj): có hại - detrimental (adj): có hại, bất lợi cho = harmful Vậy đáp án đúng là D Tạm dịch: Khí thải từ các nhà máy và các phương tiện đi lại có thể tác động có hại đến sức khỏe của chúng ta.
  • 125. Question 23. Dozens of valuable works of art disappeared during shipment to the US. A. authority B. security C. activity D. delivery Kiến thức: Đồng nghĩa (từ đơn) Giải thích: Xét các đáp án: A. authority (n.): chính quyềnB. security (n.): an ninh C. activity (n.): hoạt động D. delivery (n.): sự giao hàng Ta có: shipment: sự giao hàng shipment = delivery Vậy đáp án đúng là D Tạm dịch: Hàng chục tác phẩm nghệ thuật có giá trị đã biến mất trong quá trình vận chuyển đến Mỹ.
  • 126. Question 24. Tom may get into hot water when driving at full speed after drinking wine. A. get into trouble B. stay safe C. fall into disuse D. keep calm Kiến thức: Trái nghĩa (cụm từ hoặc thành ngữ) Giải thích: - get into trouble: gặp rắc rối >< stay safe: giữ an toàn - get into hot water: gặp rắc rối - stay safe: giữ an toàn - fall into disuse: bỏ đi, không dùng đến - keep calm: giữ bình tĩnh Vậy đáp án đúng là B Tạm dịch: Tom có thể gặp rắc rối khi lái xe quá tốc độ sau khi uống rượu.
  • 127. Question 25. Because of her conservative views, the professor never accepts anything not related to traditional values and the status. A. conservational B. progressive C. modern D. economic Kiến thức: Trái nghĩa (từ đơn) Giải thích: Xét các đáp án: A. conservational: bảo tồn B. progressive: tiến bộ C. modern: hiện đại D. economic: kinh tế Ta có: conservative: bảo thủ, cố chấp Vậy: conservative >< progressive Tạm dịch: Vì quan điểm bảo thủ của mình, giáo sư không bao giờ chấp nhận bất cứ điều gì không liên quan đến các giá trị truyền thống và địa vị.
  • 128. Question 26. His wife gave birth to their first child. He understood what true responsibility meant. A. Never has he understood true responsibility before he became a parent himself. B. Were his first child not to be born, he wouldn’t understand true responsibility. C. Hardly had he understood true responsibility when their first child was born. D. Not until he became a parent did he understand what true responsibility meant. Kiến thức: Kết hợp câu – đảo ngữ Giải thích: Tạm dịch: Vợ anh ấy sinh đứa con đầu tiên của họ. Anh ta hiểu được trách nhiệm thực sự là gì. = D. Mãi cho đến khi vợ anh ta sinh đứa con đầu tiên của họ thì anh ta mới hiểu được trách nhiệm thực sự là gì. Cấu trúc: Not until + S + V + O + trợ động từ + S + V (nguyên thể): Mãi cho đến khi … Chọn D Các phương án khác: A. Sai “has” => “had” (hành động trở thành bố đã ở quá khứ => hành động trước khi trở thành bố chia ở thì quá khứ hoàn thành) B. Câu điều kiện loại 2 => sai
  • 129. Question 27. Peter moved abroad for a fresh start. He regrets it now. A. Peter wishes he hadn’t moved abroad for a fresh start. B. If Peter moved abroad for a fresh start, he would regret it. C. Peter regrets not having moved abroad for a fresh start. D. If only Peter had moved abroad for a fresh start. Kiến thức: Kết hợp câu – câu ước Giải thích: Dịch đề bài: Peter chuyển ra nước ngoài cho một khởi đầu mới. Bây giờ anh ấy đang hối hận. A. Peter ước anh ấy đã không chuyển ra nước ngoài cho một khởi đầu mới. (Đúng nghĩa, đúng cấu trúc) B. Nếu Peter chuyển ra nước ngoài cho một khởi đầu mới, anh ấy sẽ hối hận. (Sai nghĩa) C. Peter hối tiếc vì đã không chuyển ra nước ngoài cho một khởi đầu mới. (Sai nghĩa – sự thật là đã chuyển ra nước ngoài) D. Giá như Peter đã chuyển ra nước ngoài cho một khởi đầu mới. (Sai nghĩa – như C)
  • 130. Question 28. The composer Verdi has written the opera Aida to celebrate the opening of the Suez Canal, but the opera was not performed until 1871. A. has written B. to celebrate C. opening of D. was not performed Kiến thức: Lỗi sai – Thì của động từ Giải thích: Xét mốc thời gian “until 1871” thì hành động đều đã xảy ra vào thời điểm xác định trong quá khứ. Nên sai ở đáp án A. Sửa lỗi: has wri en → wrote Tạm dịch: Nhà soạn nhạc Verdi đã viết vở opera Aida để kỷ niệm việc mở kênh đào Suez, nhưng vở opera đã không được biểu diễn cho đến năm 1871.
  • 131. Question 29. Those of us who have a family history of heart disease should make yearly appointments with their doctors. A. who B. should make C. yearly D. their Kiến thức: Lỗi sai – Đại từ nhân xưng/ tính từ sở hữu Giải thích: Những người trong số chúng ta đặt lịch hẹn bác sỹ của chúng ta chứ không thể là đặt lịch hẹn bác sỹ của họ. Vậy đáp án đúng là D Sửa lỗi: their → our
  • 132. Question 30. Modern office buildings have false floors under which computer and phone wires can be lain. A. office buildings B. false floors C. which D. can be lain Kiến thức: Lỗi sai – Từ vựng Giải thích: lay - laid - laid : đặt, xếp thứ gì đó nằm ở vị trí tĩnh lie – lay - lain: tựa lên, nằm nghỉ trên một vị trí bằng phẳng Vậy đáp án đúng là D Sửa: can be lain → can be laid
  • 133. Question 31. It’s possible that Joanna didn’t receive my message. A. Joanna shouldn’t have received my message. B. Joanna needn’t have received my message. C. Joanna mightn’t have received my message. D. Joanna can’t have received my message. Kiến thức: Câu đồng nghĩa – Động từ khuyết thiếu Giải thích: Dịch câu gốc: Có thể là Joanna đã không nhận được tin nhắn của tôi Dịch các phương án. A. Joanna đáng lẽ không nên nhận được tin nhắn của tôi B. Joanna đáng lẽ ra không cẩn nhận được tin nhắn của tôi C. Joanna có lẽ đã không nhận được tin nhắn của tôi. D. Joanna đã không thể nhận được tin nhắn của tôi shouldn’t have P 2 : không nên làm gì trong quá khứ (nhưng đã làm) needn’t have P 2 : không cần làm gì trong quá khứ (nhưng đã làm) mightn’t have P 2 : có thể có lẽ đã không can’t have P 2 : không thể nào đã làm Phù hợp nhất với nghĩa gốc là đáp án C
  • 134. Question 32. She said, "John, I'll show you round my city when you're here." A. She made a trip round her city with John. B. She planned to show John round her city. C. She said to John that she would show John round her city when he was there.. D. She organized a trip round her city for John. Kiến thức: Câu đồng nghĩa – Câu tường thuật Giải thích: Vậy đáp án đúng là C Đề bài: Cô ấy nói: "John, tớ sẽ dẫn bạn đi thăm thành phố của tớ trong lúc bạn ở đây = C. Cô ấy hứa sẽ dẫn John đi thăn thành phố của cô ấy.
  • 135. Question 33. It is over twenty years since I last got in touch with them. A. I can’t help keeping getting in touch with them for over 20 years. B. I haven’t gotten in touch with them for over 20 years. C. I used to get in touch with them for over 20 years. D. I have been getting in touch with them for over 20 years. Kiến thức: Câu đồng nghĩa – Thì của động từ Giải thích: Câu đề bài: Đã hơn hai mươi năm kể từ lần cuối tôi liên lạc với họ. = B. Tôi đã không liên lạc với họ được hơn 20 năm. Công thức: S + haven’t/ hasn’t + Vp2 + for + time = It’s + time + since + S + (last) Vp1
  • 136. Question 34. A. much B. many C. every D. a lot Every day on radio, on TV, and in the newspaper, we hear, see or read about (34) _______ problems in the world, for example, pollution problems. Kiến thức: Đọc điền từ Giải thích: Xét các đáp án: A. much + N-không đếm được B. many + N-số nhiều C. every + N-số ít D. a lot + (of + N-số nhiều/ không đếm được)
  • 137. Question 35. A. released B. launched C. escaped D. freed Air pollution is the first kind. It mostly comes from fumes (35) _______ from motorbikes, cars, airplanes, trains and poisonous gases emitted from factories. Kiến thức: Đọc điền từ Giải thích: Xét các đáp án: A. release (v): thải ra/ phát hành B. launch (v): phóng ra C. escape (v): trốn thoát D. free (v): giải phóng Tạm dịch: Ô nhiễm không khí là loại đầu tiên. Nó chủ yếu đến từ khói thải ra từ xe máy, ô tô, máy bay, tàu hỏa và khí độc thải ra từ các nhà máy.
  • 138. Question 36. A. that B. which C. who D. where Also, waste is dumped anywhere, even in the city (36) _______ many people are living. Kiến thức: Đọc điền từ Giải thích: Xét các đáp án: A. that: Thay thế cho danh từ chỉ người và vật có chức năng làm chủ ngữ và tân ngữ trong mệnh đề quan hệ. B. which: Thay thế cho danh từ chỉ vật, sự vật hiện tượng có chức năng làm chủ ngữ và tân ngữ trong mệnh đề quan hệ. C. who: Thay thế cho danh từ chỉ người có chức năng làm chủ ngữ và tân ngữ trong mệnh đề quan hệ. D. where: Thay thế cho danh từ chỉ nơi chốn có chức năng làm trạng ngữ chỉ nơi chốn trong mệnh đề quan hệ.
  • 139. Question 37. A. However B. So C. Therefore D. But . Many people earn their living from fishing in the sea, and the fish they catch feed many people. (37) _______, the sea has become so polluted from oil spills and factory wastes that the fish are dying. Kiến thức: Đọc điền từ Giải thích: Xét các đáp án: A. However: tuy nhiên B. So: vì vậy C. Therefore: cho nên D. But: nhưng Tạm dịch: Nhiều người mưu sinh bằng nghề đánh bắt trên biển, và những con cá họ bắt được đã nuôi sống nhiều người. Tuy nhiên, biển đã trở nên ô nhiễm do dầu tràn và chất thải của nhà máy đến nỗi cá đang chết dần.
  • 140. Question 38. A. method B. solution C. road D. idea This problem is growing more difficult every day. We must find a good (38) _______ that makes the world a better place to live. Kiến thức: Đọc điền từ Giải thích: Xét các đáp án: A. method: phương pháp B. solution: giải pháp C. road: con đường D. idea: ý kiến Tạm dịch: Vấn đề này mỗi ngày một khó hơn. Chúng ta phải tìm ra một giải pháp tốt để làm cho thế giới trở thành một nơi tốt đẹp hơn để sống.
  • 141. Question 39. The best title for this passage could be ________. A. Vietnam struggling with ageing population B. Stopping the two-child policy in Vietnam C. Raising the retirement age in Vietnam D. How to solve the aging population in Vietnam Kiến thức : Đọc hiểu Giải thích: Tiêu đề tốt nhất cho đoạn văn này có thể là _________. A. Việt Nam đấu tranh với việc già hóa dân số B. Chấm dứt chính sách 2 con ở Việt Nam C. Tăng tuổi nghỉ hưu ở Việt Nam D. Cách giải quyết vấn đề già hóa dân số ở Việt Nam Căn cứ vào thông tin toàn bài: “chính sách 2 con; tăng tuổi nghỉ hưu; cách giải quyết vấn đề già hóa dân số” đều được đề cập trong bài nhưng chưa bao quát toàn bài. => Đáp án A.
  • 142. Question 40. Which statement is probably TRUE according to the information in the paragraph 1? A. In 2017, the elderly take up for one third of the total population, this leads to more concerns about healthcare, welfare and pensions for the elderly. B. In 2015, the two-child policy has been officially tightened and succeeded. C. The government would promote families to have two children to compensate for the ageing population within the next 20 years. D. In the next 15-17 years, Vietnam's economy will need a large labor force to integrate with global economy. Giải thích: Phát biểu nào có lẽ là ĐÚNG theo thông tin trong đoạn văn 1? A. Năm 2017, dân số giả ở Việt Nam chiếm tới 1/3 dân số cả nước, điều này dẫn đến những lo ngại về chăm sóc sức khỏe, phúc lợi xã hội và lương hưu cho người già. B. Năm 2015, chính sách 2 con đã chính thức được thắt chặt và đã thành công. C. Chính phủ khuyến khích các gia đình có 2 con để bù đắp cho việc già hóa dân số trong 20 năm tới. D. Trong 15-17 năm tới, nền kinh tế Việt Nam sẽ cần một lực lượng lao động lớn để hội nhập với nền kinh tế toàn cầu. Thông tin: "in 15-20 years the elderly will account for one third of the total population. This raises concerns about healthcare, welfare and pensions for the elderly at a time when Vietnam is focusing on economic integration and requires a large labor force. Tạm dịch: ...trong 15- 20 năm nữa, người cao tuổi sẽ chiếm một phần ba tổng dân số. Điều này làm tăng mối lo ngại về chăm sóc sức khỏe, phúc lợi và lương hưu cho người cao tuổi tại thời điểm Việt Nam đang tập trung vào hội nhập kinh tế và đòi hỏi một lực lượng lao động lớn.
  • 143. Question 41. The word “its" in paragraph 1 refers to ________? A. two-Child policy B. aging population C. retirement age D. economic integration Giải thích: Từ “its” trong đoạn 1 thay thế cho từ _________. A. chính sách 2 con B. già hóa dân số C. tuổi nghỉ hưu D. hội nhập kinh tế Thông tin: By ending the two-child policy the government expects to make up for the ageing population within the next 20 years. But its effect could be creating an uncontrollable boom in the Vietnamese population. Thông tin: While the policy is beneficial in utilizing the work experience of the elderly while creating savings in the pension budget, it also means fewer job prospects and promotion opportunities for younger generations.
  • 144. Question 42. In the 2rd paragraph, the writer suggests that ________. A. The Ministry of Labor has applied raising the retirement age in May 2017. B. Raising the retirement age can reduce job opportunities for younger generations. C. The elderly whose age is 50 would be dangerous if they continued to work D. Raising the retirement age and stopping two-child policy can be considered as long-term and effective solutions. Kiến thức : Đọc hiểu Giải thích: Trong đoạn 2, tác giả gợi ý rằng _________. A. Bộ Lao động đã áp dụng việc nâng tuổi nghỉ hưu từ tháng 5 năm 2017 B. Nâng tuổi nghỉ hưu có thể làm giảm cơ hội việc làm cho thế hệ trẻ. C. Những người 50 tuổi có thể sẽ gặp nguy hiểm nếu tiếp tục làm việc D. Nâng tuổi nghỉ hưu và dừng chính sách 2 con được coi là những giải pháp lâu dài và hiệu quả. Thông tin: While the policy is beneficial in utilizing the work experience of the elderly while creating savings in the pension budget, it also means fewer job prospects and promotion opportunities for younger generations.
  • 145. Question 43. The word "temporary" in paragraph 2 means _____. A. constant B. permanent C. short-term D. long-term Kiến thức : Đọc hiểu Giải thích: Từ “ temporatory” trong đoạn 2 có nghĩa là ______. A. consistant (a): kiên trì B. permanent (a): lâu dài C. short-term (a): ngắn hạn D. long-term (a): dài hạn Temporatory = short-term Thông tin: Despite these drawbacks, raising the retirement age is still considered by policymakers as one of the key solutions to the ageing population problem in Vietnam. But these are only temporary solutions. Tạm dịch: Bất chấp những hạn chế này, việc tăng tuổi nghỉ hưu vẫn được các nhà hoạch định chính sách coi là một trong những giải pháp chính cho vấn đề dân số giờ ở Việt Nam. Nhưng đây chỉ là những giải pháp tạm thời.
  • 146. Question 44. Which of the following could best reflect the main purpose of the author in the passage? A. To prove that coral reefs are animals. B. To provide the facts about coral reefs. C. To explain that coral reefs are the most diverse ecosystems in the ocean. D. To distinguish coral reefs with other animals. Kiến thức : Đọc hiểu Giải thích: Câu nào sau đây thể hiện chính xác nhất mục đích chính của tác giả trong bài đọc? A. Nhằm chứng minh rằng dải san hô ngầm là động vật. B. Để cung cấp các thông tin thật về dải san hô ngầm. C. Để giải thích rằng dải san hô ngầm là hệ sinh thái đa dạng nhất dưới đại dương. D. Để phân biệt dải san hô ngầm và các động vật khác. Căn cứ vào thông tin toàn bài: Bài đọc cung cấp cho chúng ta toàn bộ thông tin về loài san hô, từ cấu tạo (đoạn 2, 3), nơi sinh sống (đoạn 4) và các loại san hô (đoạn cuối).